0% found this document useful (0 votes)
167 views130 pages

Micro Questions

This document contains a review test with 25 multiple choice questions about bacteria and microbiology. The questions cover topics such as bacterial toxins, sugar transport in bacteria, culture media uses, antibiotic resistance mechanisms, bacterial cell structures, bacteriophages, and bacterial metabolism. The correct answer is provided for each question along with a brief explanation.

Uploaded by

Jack Jenkins
Copyright
© © All Rights Reserved
We take content rights seriously. If you suspect this is your content, claim it here.
Available Formats
Download as PDF, TXT or read online on Scribd
0% found this document useful (0 votes)
167 views130 pages

Micro Questions

This document contains a review test with 25 multiple choice questions about bacteria and microbiology. The questions cover topics such as bacterial toxins, sugar transport in bacteria, culture media uses, antibiotic resistance mechanisms, bacterial cell structures, bacteriophages, and bacterial metabolism. The correct answer is provided for each question along with a brief explanation.

Uploaded by

Jack Jenkins
Copyright
© © All Rights Reserved
We take content rights seriously. If you suspect this is your content, claim it here.
Available Formats
Download as PDF, TXT or read online on Scribd
You are on page 1/ 130

Review Test

Directions: Each of the numbered items or incomplete statements in this section is followed by
­answers or completions of the statement. Select the ONE lettered answer that is BEST in each case.

1.  A bacterial toxin with superantigen activity 7.  Bacteria that synthesize organic compounds
is produced by from inorganic compounds are
(A) Clostridium tetani (A) Heterotrophs
(B) Bordetella pertussis (B) Obligate anaerobes
(C) Escherichia coli (C) Aerobes
(D) Staphylococcus aureus (D) Facultative anaerobes
(E) Vibrio cholerae (E) Autotrophs

2.  Sugar transport into bacteria is frequently 8.  A bacterial structure involved in adherence is
facilitated by (A) Capsule
(A) Acetyltransferase (B) Lipopolysaccharide
(B) Neuraminidases (C) Common pili
(C) Oxidases (D) O-specific side chain
(D) Penicillin-binding proteins (E) Teichoic acid
(E) Phosphotransferases
9.  Aminoglycoside antibiotics are
3.  Thayer-Martin and Martin-Lewis media are (A) Bactericidal for Gram-positive bacteria
used to isolate and identify (B) Inactivated by R-factor
(A) E. coli phosphotransferases
(B) Mycobacteria (C) Mycolic acid synthesis inhibitors
(C) Neisseria (D) Peptidoglycan synthesis inhibitors
(D) Salmonella (E) Items that require bacterial growth for the
(E) Shigella effect

4.  β-lactamases confer antibiotic resistance by 10.  A-B subunit structure as it relates to bacte-
(A) Altering antibiotic permeability rial pathogenesis refers to the structure of
(B) Altering penicillin-binding proteins (A) Bacterial exotoxins
(C) Altering 70S ribosome structure (B) Gram-negative bacteria endotoxin
(D) Modifying cellular RNA polymerase (C) Nucleic acid inhibitor antibiotics
(E) Modifying antibiotic structure (D) Penicillin-binding proteins
(E) Resistance transfer factors
5.  Polymers of N-acetylglucosamine and
N-acetylmuramic acid are found in which of 11.  Which of the following displays the Pasteur
the following structures? effect?
(A) Teichoic acid (A) Heterotrophs
(B) Cell wall (B) Obligate anaerobes
(C) Glycocalyx (C) Aerobes
(D) Lipopolysaccharide (D) Facultative anaerobes
(E) Autotrophs
6.  A phage that is not inactivated by proteases
is called a 12.  Which of the following toxins acts on
(A) Prophage synaptosomes?
(B) Virulent phage (A) E. coli heat-labile toxin
(C) Temperate phage (B) Clostridium tetani exotoxin
(D) Filamentous phage (C) Corynebacterium diphtheriae exotoxin
37
38 BRS Microbiology and Immunology

(D) Pseudomonas aeruginosa exotoxin (C) A replicon


(E) Clostridium perfringens alpha-toxin (D) A lytic bacteriophage
(E) An intron
13.  Superoxide dismutase-containing bacteria
(A) Need superoxide to grow 20.  The expression of the lac operon
(B) Are frequently obligate anaerobes (A) Must be initiated by the binding of an
(C) Grow slowly in the presence of CO2 ­inducer protein
(D) Produce hydrogen peroxide from hydrogen (B) Involves the release of allolactose from a
ion and the superoxide free radical (O2•2) repressor protein
(C) Does not involve the expression of
14.  Lysogenic phage conversion involves ­structural genes
(A) The transformation of a virulent phage to a (D) Necessitates the finding of RNA
lysogenic phage ­polymerase followed by transcription
(B) A change in bacterial phenotype due to
the presence of a prophage 21.  Bacteriophage containing host-cell DNA is
(C) The conversion of a prophage to a temper- involved in which of the following processes?
ate phage (A) Transformation
(D) The incorporation of a prophage into the (B) Conjugation
bacterial chromosome (C) Transduction
(D) Transcription
15.  Bacteria capable of growth in a high salt (E) Recombination
concentration are best isolated in which of the
following media? 22.  The exchange of allelic forms of genes is
(A) Minimal growth media ­involved in which of the following processes?
(B) Complex growth media (A) Transformation
(C) Differential growth media (B) Conjugation
(D) Selective growth media (C) Transduction
(D) Transcription
16.  Bacteria lacking superoxide dismutase are
(E) Recombination
(A) Heterotrophs
(B) Obligate anaerobes 23.  Which of the following processes creates
(C) Aerobes high-frequency recombination donors?
(D) Facultative anaerobes
(A) Transformation
(E) Autotrophs
(B) Conjugation
17.  The regulation of enzyme activity in (C) Transduction
­bacterial cells can (D) Transcription
(E) Recombination
(A) Be coupled to the binding of effector
(F) Translation
molecules
(B) Be controlled by a catabolite activator
24.  Toxins of enterotoxic Gram-negative
­ rotein (CAP)
p
­bacteria are transferred outside of the cell by
(C) Occur via attenuation sequences
(D) Involve inducer molecules (A) ATP-activated pores
(B) GTP-coupled transporters
18.  The plasma membrane (C) PBPs
(A) Contains matrix porins (D) Pili
(B) Includes endotoxin (E) Protein secretion systems
(C) Contains glycocalyx
(D) Contains the enzymes involved in bacte- 25.  A mutation which rarely disrupts gene
rial oxidative phosphorylation product function is a
(A) Deletion
19.  Bacterial antibiotic resistance is frequently (B) Frameshift
conveyed by (C) Insertion
(A) A temperate bacteriophage (D) Nonsense
(B) An R-factor plasmid (E) Nucleotide substitution
Answers and Explanations

1. The answer is D.  Staphylococcus aureus produces an enterotoxin and TSST-1 toxins with
s­ uperantigen activity. Streptococcus pyogenes also produces toxins with this activity.
2. The answer is E.  The transport of sugar into a bacterium frequently involves the transfer of a
phosphate group to the sugar molecule.
3. The answer is C.  These media are variants of chocolate agar and contain antibiotics that in-
hibit many normal respiratory and genital bacteria but allow the growth of Neisseria species.
4. The answer is E.  β-lactamases cleave the β-lactam ring structure that is important for the anti-
bacterial activity of penicillins, cephalosporins, monobactams, and carbapenems.
5. The answer is B.  N-acetylglucosamine and N-acetylmuramic acid are polymerized to form the
peptidoglycan backbone of the cell wall.
6. The answer is A.  A prophage is the intracellular DNA of a phage and is therefore resistant to
protease degradation.
7. The answer is E.  Autotrophic bacteria do not require organic compounds for growth because
they synthesize them from inorganic precursors.
8. The answer is C.  Common pili, adhesins, and the glycocalyx are three bacterial structures that
are involved in adherence.
9. The answer is E.  Bacteria must be actively replicating and synthesizing protein for these
­compounds, which bind to the 30S ribosomal subunit to have their bactericidal effect.
1 0. The answer is A.  Many bacterial exotoxins have an A-B subunit structure in which the
B ­subunit is involved in binding and the A subunit possesses biological activity inside the
a­ ffected cell.
11. The answer is D.  Facultative anaerobes shift from a fermentative to a respiratory metabolism
in the presence of air because the energy needs of the cell are met by consuming less glucose
(Pasteur effect) under respiratory metabolism.
12. The answer is B.  Clostridium tetani exotoxin acts on synaptosomes, thereby causing hyperre-
flexia of skeletal muscles.
13. The answer is D.  Superoxide dismutase is found in aerobic and facultative anaerobic bacteria.
It protects them from the toxic free radical (O2•2) by combining it with a hydrogen ion to form
hydrogen peroxide, which is subsequently degraded by peroxidase.
14. The answer is B.  Lysogenic phage conversion refers to a change in bacterial phenotype result-
ing from the presence of a lysogenic prophage of a temperate phage.
1 5. The answer is D.  A selective growth medium that contains a high salt concentration would
permit bacterial growth.
16. The answer is B.  Superoxide dismutase, which is present in aerobes and facultative anaerobe
organisms, protects them from the toxic O2•2 radical. This enzyme is not present in obligate
anaerobes.
17. The answer is A.  The biochemical activity of an enzyme may be regulated by binding of
­effector molecules or by biosynthetic pathway end-product feedback inhibition. Enzyme
­synthesis may be controlled by inducers, attenuation sequences, or catabolite activator protein.
18. The answer is D.  The plasma membrane contains the enzymes involved in oxidative
phosphorylation.

39
40 BRS Microbiology and Immunology

1 9. The answer is B.  R-factor (resistance) plasmids contain genes for proteins that degrade antibi-
otics or alter antibiotic transport, thus conferring antibiotic resistance. They also carry transfer
genes, which facilitate their intercellular transfer to other genomes.
20. The answer is A.  The transcription of the lac operon is under negative control. Initiation
­depends on the binding of allolactose to a repressor protein. This reaction prevents the
­repressor from binding to the operator region, thus allowing RNA polymerase to bind and
­transcription to proceed.
21. The answer is C.  Bacteriophages containing portions of host-cell DNA can introduce this
­genetic material into new host cells via the process of transduction.
22. The answer is E.  DNA or genetic recombination is the general term used to describe the
­exchange of allelic forms of genes in bacteria or eukaryotic cells.
2 3. The answer is E.  High-frequency recombination donors, which result from the integration of a
fertility (F) factor into chromosomal DNA, are created by recombination.
2 4. The answer is E.  Depending on the bacteria, one of four types of protein secretion systems of
the Gram-negative bacteria is responsible for transporting exotoxins to the outside of the cell.
2 5. The answer is E.  Nucleotide substitution and some missense mutations can be silent and not
affect gene product function.
Review Test

Directions: Each of the numbered items or incomplete statements in this section is followed by
­answers or completions of the statement. Select the ONE lettered answer that is BEST in each case.

1.  A 21-year-old male college student who had (D) Clostridium perfringens
complained of headache and feeling feverish (E) Haemophilus influenzae
the night before is brought this morning to the
emergency department (ED) when his room- 5.  A patient undergoing chemotherapy
mate was unable to rouse him. He had been ­ evelops a cough. Acid-fast stain of his spu-
d
well until yesterday. Vital signs include fever tum shows rods and slightly longer forms,
(39.8°C/103.1°F), tachycardia, and hypotension with some branching; they vary in their acid-
(BP 70/55). Remarkable on physical examina- fast reaction from one area of the slide to the
tion is petechial rash (purpuric in areas) and nu- next. The acid-fast stain was performed by an
chal rigidity with positive Kernig and Brudzinski experienced medical technologist and, when
signs. CSF is cloudy with high protein and low redone, showed the same variation. The growth
glucose. Intracellular, red diplococci are seen on was done aerobically. What is the most likely
Gram stain. What is the most likely genus? agent?
(A) Staphylococcus (D) Mycobacterium (A) Actinomyces
(B) Streptococcus (E) Neisseria (B) Chlamydophila
(C) Chlamydia (C) Mycobacterium avian-intracellulare
(MAI or MAC)
2.  A 24-year-old female presents with dysuria, (D) Nocardia
as well as urinary urgency and frequency. A
urine dipstick test is positive for both leukocyte 6.  A female patient with a new genital lesion
esterase and nitrites. What genus or family is presents to your sexually transmitted disease
noted for the production of nitrites? clinic. She is homeless, has no health insur-
(A) Escherichia (C) Streptococcus ance, and is an intravenous drug user. You
(B) Staphylococcus (D) Vibrio suspect syphilis. Which of these techniques
would be most appropriate to demonstrate
3.  What rapid test commonly used on Gram- treponemes?
negative rods rules out Enterobacteriaceae if (A) Immunological test such as the VDRL
positive? (B) Dark-field microscopy
(A) Catalase (D) Chitinase (C) Acid-fast stain
(B) Coagulase (E) Urease (D) Gram stain
(C) Oxidase (E) Electrophoresis

4.  A patient presents with rapid onset severe 7.  The CSF from a 2-week-old infant with men-
respiratory symptoms. Chest radiographs show ingitis shows rods with tumbling motility. These
a hemorrhagic lymphadenitis. The isolation bacteria are found to be Gram-positive and do
of chains of fairly large, aerobic Gram-positive not form spores. What is the most likely agent?
rods, some of which have started to sporulate (A) Actinomyces
from a patient with this presentation, should (B) Bacillus
raise a major concern of which organism? (You (C) Clostridium
should be able to answer this question from the (D) Corynebacterium
genus alone, although a question might also (E) Listeria
mention that it was nonmotile.)
(A) Actinomyces israelii 8.  Both a 53-year-old farmer and his 21-year-
(B) Bacillus anthracis old son present in August with fever, myalgia,
(C) Campylobacter jejuni and malaise, which they came down with

60
Chapter 3   Important Bacterial Genera 61

within a few hours of each other. The son had (A) Chlamydophila pneumoniae
been home in southern Minnesota for only (B) Influenza virus
3 weeks to help field train two new hunting (C) Klebsiella pneumoniae
dogs. You ask about potential tick bites, and (D) Mycoplasma pneumoniae
the son did have one on him, which was quite (E) Staphylococcus aureus
engorged. Platelets and granulocytes are low (F) Streptococcus pneumoniae
in each man’s blood. You ask one of your
experienced techs to do a Giemsa stain on a 11.  The reagent used to distinguish staphylo-
thick blood smear. He calls, reporting clusters cocci from streptococci is
of cells resembling raspberries in granulo- (A) Hydrogen peroxide
cytes, even though nothing grows in any of (B) Fibronectin
the blood cultures. You realize that the blood (C) Fibrinogen
cultures you set up will not grow and that you (D) Oxidase
have two patients who have infections with
a tick-borne obligate intracellular parasite of 12.  A 14-month-old boy is brought in by his
granulocytes. What genus does the organism parents with fever, fussiness and lethargy,
belong to? and apparent headache. On examination, the
(A) Anaplasma (formerly Ehrlichia) neck is stiff. His parents have not allowed his
(B) Borrelia routine childhood vaccines. Very short Gram-
(C) Chlamydia negative rods are seen in the CSF, so antibiotics
(D) Haemophilus are immediately started. The organism grows
(E) Mycoplasma on chocolate agar but not blood agar. No one
else in the family is ill. What is the most likely
9.  A full-term 6-day-old neonate is brought in ­causative agent?
with a purulent conjunctivitis which the par- (A) Escherichia coli
ents noticed earlier today. On Gram stain of the (B) Haemophilus influenzae type b
purulent exudate, no bacteria are seen. Which (C) Klebsiella pneumoniae
of the following bacteria is most likely the cause (D) Neisseria meningitidis
of the conjunctivitis? (E) Streptococcus pneumoniae
(A) Chlamydia trachomatis
(B) Escherichia coli 13.  A healthy 7-year-old boy who has not
(C) Listeria monocytogenes t­ raveled outside the United States is brought in
(D) Neisseria gonorrhoeae by his parents in June with signs of meningitis.
(E) Streptococcus pneumoniae No bacteria are seen in the Gram stain of the
CSF, and no bacterial capsule material is pres-
10.  An 83-year-old who still lives in her own ent as determined by a series of latex particle
home has developed pneumonia following in- agglutination tests standard to the diagnosis of
fluenza. The Gram stain of her sputa is shown. meningitis. The CSF glucose level is slightly low,
What is the most likely agent? protein is near normal, and white cell count is
less than 500 cells/microL, mainly lymphocytes.
What is the most likely causative agent?
(A) Chlamydophila pneumoniae
(B) Enterovirus
(C) Mycoplasma pneumoniae
(D) Mycobacterium tuberculosis
(E) Treponema pallidum

14.  In any case, a clue indicating that the


causative organism is an obligate intracel-
lular pathogen transmitted by an arthropod
bite should lead you to which of the following
groups of organisms?
(A) Chlamydiae
(B) Enterobacteriaceae
62 BRS Microbiology and Immunology

(C) Rickettsias including Anaplasma (A) Fluorochromes are more specific and used
and Ehrlichia just for Mycobacterium tuberculosis.
(D) Spirochetes including Borrelia (B) IFAs are less specific since they use anti-
burgdorferi body to a different species’ antibody (i.e.,
rabbit antibody to human antibody).
15.  What is the main difference between (C) Fluorochrome staining is less sensitive than
f­ luorochrome staining (e.g., auramine-­ comparable staining with light microscopy.
rhodamine screening for Mycobacterium (D) IFA’s specificity is dependent on the pri-
­tuberculosis) and indirect fluorescent ­antibody mary antibody used; fluorochromes lack
(IFA) staining? the antibody specificity.
Answers and Explanations

1. The answer is E.  Gram-negative bacteria should be described as red or pink, so the
­description fits a Gram-negative diplococcus which is most likely Neisseria. Staph and Strep
would be purple, and neither Chlamydophila nor Mycobacterium will show up on Gram stain.
2. The answer is A.  Since all choices are in italics, all are genus names and you do not need to
look for a family name. Since all Enterobacteriaceae generally produce nitrate reductase, they
all eventually produce a positive dipstick nitrite test. Escherichia is the only choice belonging
to this family. Other members of Enterobacteriaceae that cause urinary tract infections include
Proteus and Klebsiella. Staph and Strep are Gram-positive and Vibrio is a genus of Gram-nega-
tive, comma-shaped bacteria. A positive nitrite test rules out Staph. saprophyticus as the cause
of a urinary tract infection.
3. The answer is C.  Most common Gram-negative rods are oxidase-positive. The major excep-
tions are members of the Enterobacteriaceae. A catalase test (utilizing hydrogen peroxide) is
most commonly used to distinguish Staphylococci (+) from Streptococci (–). (Most aerobes will
be catalase-positive and many obligate anaerobes are catalase-negative.) Coagulase is used to
distinguish Staph. aureus (+) from other medical isolates (coagulase-negative Staph). No medi-
cally important bacteria have chitinase. There are several medically important urease-positive
bacteria, most importantly H
­ elicobacter pylori, Proteus sp., and Ureaplasma urealyticum.
4. The answer is B.  Bacillus is the correct genus. There are two Gram-positive spore-forming
rods. Bacillus is aerobic with one species (Bacillus anthracis), causing hemorrhagic lymphad-
enitis and pulmonary edema. The other genus with bacterial endospores, the Clostridia, do
not grow aerobically. The other Gram-positive organism listed, Actinomyces, often described
as a branching bacterium, is also anaerobic. Campylobacter and Haemophilus are both
Gram-negative.
5. The answer is D. Either Nocardia or MAI is possible from the clinical scenario, but the most
likely agent from the acid-fast stain description, the focus of the question, is Nocardia. Also,
MAI would not grow in 2 days (so some mention of time would be made) and Actinomyces
would only grow anaerobically, which would also be stated. Chlamydophila would not grow
except in tissue culture.
6. The answer is B.  First, notice that the question specifically asks about demonstrating the pres-
ence of treponemes and not making the diagnosis of syphilis. (Also, note that the lesions are
still present, which suggests that it is too early for serological methods to be reliable.) Then,
remember that Treponema pallidum is still only cultured by research labs. Because treponemes
are so thin in cross-section, they do not reliably show up on a Gram stain; thus, dark-field or FA
(not mentioned as a choice) staining would be necessary.
7. The answer is E.  Listeria is the correct answer. Both Bacillus and Clostridium can be elimi-
nated because they are spore-formers. Actinomyces and corynebacteria are both Gram-positive
and nonmotile and are not common causative agents of neonatal meningitis. Listeria has a
tumbling motility when grown in broth, in this case CSF.
8. The answer is A.  Obligate intracellular bacteria transmitted through arthropods would
most likely be the genera: Rickettsia, Orientia, Anaplasma, and Ehrlichia, with the latter two
infecting white cells and Anaplasma causing the described disease (Human granulocytic
anaplasmosis).
9. The answer is A.  Neonatal eye infections are most likely to be Chlamydia trachomatis, which
does not show up on Gram stain. All of the remaining organisms stain well with Gram stain.

63
64 BRS Microbiology and Immunology

1 0. The answer is F.  The most common cause of pneumonia in people over 65 years of age
has generally been S. pneumoniae. Although the other agents also cause pneumonia,
S. ­pneumoniae is the only organism that fits the descriptions of Gram-positive cocci in chains.
It is also quite common following influenza. If the woman had not been vaccinated for influ-
enza, then she also may not have been vaccinated for pneumococcus.
1 1. The answer is A.  The staphylococci are aerobic while the streptococci are aerotolerant anaer-
obes. Streptococci ferment even in the presence of full oxygen and lack catalase. A standard
quick test is the test mixing staphylococci with hydrogen peroxide. The generation of oxygen
bubbles indicates that a Gram-positive coccus is a Staphylococcus rather than a Streptococcus.
A coagulase test using serum to see if fibrinogen is clotted is used to distinguish the coagulase-
positive staphylococci from coagulase-negative staphylococci. Oxidase is not the reagent but
the reactor in the oxidase test.
12. The answer is B. Although Haemophilus influenzae type b is rarely seen in vaccinated chil-
dren younger than 2 years of age, it still occurs in unvaccinated children. The description of a
chocolate agar-positive organism should suggest either Neisseria meningitidis or H. influenzae
type b. The descriptor as a short rod (from CSF) or pleomorphic rod (from culture) suggests
H. i­ nfluenzae type b. Although both Klebsiella pneumonia and Escherichia coli (a cause of
neonatal meningitis) are both Gram-negative rods, neither is likely to cause meningitis in a
healthy child, and, like all other Enterobacteriaceae, will grow on blood agar. Streptococcus is
Gram-positive.
1 3. The answer is B.  None of the bacteria listed as choices reliably show up on Gram stain, and
none of them is likely to cause meningitis in this scenario. It is much more likely to be an
­enterovirus. (This one was just to keep you awake!)
14. The answer is C.  Of the choices, only the chlamydiae and the rickettsia are obligate intracel-
lular organisms. Chlamydiae are spread by direct contact or respiratory droplets, while the
rickettsia (the correct answer) are spread commonly by arthropod vectors. Spirochetes like
B. burgdorferi are not obligate intracellular pathogens and most are also not transmitted by
­arthropod vectors.
15. The answer is D.  Fluorochrome dyes, like auramine-rhodamine screening for M ­ ycobacterium
tuberculosis, lack the specificity of either direct or indirect FA tests b
­ ecause the FA tests use
­antibodies, making them more specific. The fluorochrome dyes are more sensitive than
a ­comparable acid-fast stain on a bright field light microscope because they light up the
­microbes on a black background when viewed with the fluorescent microscope. However, the
use of antibodies in the IFAs makes them highly specific.
Review Test

Directions:  Each of the numbered items or incomplete statements in this section is followed by
­answers or completions of the statement. Select the ONE lettered answer that is BEST in each case.

1.  A 36-year-old man presents with focal A Gram-negative organism is found in unusu-
central nervous system signs. Imaging shows ally high numbers in the pulmonary mucus.
a brain abscess. The dominant organism is Which virulence factor is most important in
an anaerobe normally found as part of the colonization and maintenance of the organism
oral flora. Which of the following best fits that in the lungs?
description? (A) Exotoxin A
(A) Nocardia (B) Pyocyanin (blue-green pigment)
(B) Actinomyces (C) Polysaccharide slime
(C) Mycobacterium (D) Endotoxin
(D) Pseudomonas aeruginosa
5.  From the above case, exotoxin A of the caus-
2.  A 23-year-old man who has recently ative agent most closely resembles the action of
started working on a sheep farm in Nova which other microbial toxin?
­Scotia develops pneumonia shortly after (A) Heat-labile toxin (LT) of Escherichia coli
helping with lambing. His cough produces (B) Shiga toxin
little sputum, and a saline-induced sputum (C) Diphtheria toxin
sample shows no predominant organism (D) Vibrio cholerae toxin
­either with Gram stain or with acid-fast (E) Verotoxin
stain. It is established that he acquired the
­pneumonia from parturition products from 6.  A 36-year-old man who immigrated to
the sheep. Which agent is most likely to be the United States 15 years ago and lived in a
the cause of his pneumonia? crowded resettlement camp before coming to
(A) Rickettsia akari the United States presents with a cough that
(B) Rickettsia typhi has been bothering him for several weeks. He
(C) Rickettsia rickettsii has also lost 10 pounds. A gamma interferon
(D) Coxiella burnetii release blood test is positive. Which of the fol-
(E) Anaplasma phagocytophilum lowing factors is known to be most important
in triggering the granulomatous reaction to
3.  A 3-year-old girl presents with difficulty wall off and contain the infection?
breathing and will not lie down to be exam- (A) Cord factor
ined. You suspect acute bacterial epiglottitis (B) Mycolic acid
and examine the child’s epiglottis, which is (C) Purified protein derivative (PPD)
highly inflamed. Which vaccine are you most (D) Sulfatides
likely to find that the child is missing? (E) Wax D
(A) Diphtheria
(B) Neisseria meningitidis 7.  A 75-year-old patient develops diarrhea
(C) Polio 5 days after starting antibiotic treatment for a
(D) Streptococcus pneumoniae (conjugate serious staphylococcal infection. What is the
vaccine) most likely causative agent?
(E) Haemophilus influenzae (A) Clostridium perfringens
(B) Clostridium difficile
4.  A 22-year-old man with cystic fibrosis (C) Pseudomonas aeruginosa
presents with fever and increasing dyspnea. (D) Shigella sonnei

107
108 BRS Microbiology and Immunology

8.  A 23-year-old woman presents with mild pneumoniae. What is the most important viru-
gastroenteritis a few days after having a variety lence factor?
of sushi at a party. There is no blood or pus in (A) Endotoxin
the stool. Which causative agent is most likely (B) A phospholipase allowing Streptococcus
to have caused this illness? pneumoniae to escape the phagosome
(A) Vibrio cholerae quickly
(B) Vibrio parahaemolyticus (C) Polypeptide capsule
(C) Salmonella typhi (D) Polysaccharide capsule
(D) Shigella sonnei
13.  Which of the following organisms grows in
9.  Yersinia pestis may be transferred by 40% bile?
(A) Dermacentor tick bite (A) Enterococcus faecalis
(B) Human body louse bite (B) Streptococcus pneumoniae
(C) Ixodes tick bite (C) Group B streptococci
(D) Respiratory droplets (D) Viridans streptococci

10.  A patient who had surgery to put in a pace- 14.  A 45-year-old man who recently returned
maker and who states he felt fine for the first from Africa has been febrile for several days
2 months now presents 3 months postopera- and now presents with abdominal pain. His
tively with complaints of malaise and increas- blood cultures grow out Salmonella typhi. What
ing fatigue. He is running a low-grade fever, was the most likely source of his infection?
tires easily, and has worsening heart murmurs. (A) Raw chicken
Which of the following staphylococcal organ- (B) Undercooked hamburger
isms causes subacute bacterial endocarditis (C) Contact with baby goats on a farm and
that generally occurs 2 months or more after then eating without washing hands
heart surgery? (D) A food preparer with bad personal hygiene
(A) Staphylococcus aureus (E) Undercooked pork
(B) Staphylococcus epidermidis
(C) Staphylococcus haemolyticus 15.  A 4-day-old infant girl now showing signs
(D) Staphylococcus saprophyticus of sepsis is brought to the emergency depart-
ment. She was preterm (33 weeks) and born at
11.  A previously healthy 6-month-old boy home to her 16-year-old mom after 22 hours of
presents with upper body weakness. He cannot labor following the rupture of the membranes.
hold his eyes open, pupils do not react, and he A friend helped the mother deliver the baby.
cannot hold his head up. What is the proper What is the best description for the agent most
treatment? likely causing the sepsis if it was acquired dur-
(A) Send him home on amoxicillin and ing labor but prior to delivery? All organisms in
clindamycin (to stop the toxin production the answer choices are Gram-positive, catalase-
quickly) negative cocci found in pairs or short chains.
(B) Give him a dose of equine anti-botulinum (A) Nonhemolytic organisms found as part
immunoglobulin of the normal fecal flora; resistant to bile
(C) Offer monitored supportive care with and optochin; carries a high level of drug
antibiotics and human anti-botulinum resistance
immunoglobulin (B) Alpha-hemolytic diplococci sensitive to
(D) Offer monitored supportive care with hu- both bile and optochin
man anti-botulinum immunoglobulin (C) Beta-hemolytic cocci in chains and carry-
(E) Offer monitored supportive care with no ing Lancefield’s Group B antigen
antibiotics and no antitoxin (D) Alpha-hemolytic cocci in chains; resistant
to bile and optochin
12.  A 78-year-old man presents with a high
fever, cough producing a blood-tinged spu- 16.  A 62-year-old woman presents with signs
tum, and difficulty breathing. Sputum shows of a gastric ulcer. She does not regularly take
an organism consistent with Streptococcus nonsteroidal anti-inflammatory agents. Which
Chapter 4  Bacterial Diseases 109

characteristic appears to play a central role in after knee surgery. The laboratory gives
the organism’s ability to survive transit of the a ­preliminary ­report of a beta-hemolytic,
lumen to colonize the stomach? catalase-­positive, coagulase-positive, Gram-
(A) Phospholipase-C production positive coccus. The most likely causative
(B) Urease production agent is
(C) Microaerophilic lifestyle (A) Moraxella catarrhalis
(D) O antigens (B) Staphylococcus aureus
(C) Staphylococcus epidermidis
17.  A 54-year-old man develops a (D) Streptococcus agalactiae
­pyogenic ­infection along the suture line (E) Streptococcus pyogenes
Answers and Explanations

1. The answer is B. Only Actinomyces is anaerobic; the rest are aerobic. (And, of those men-
tioned, only Actinomyces is part of the normal oral flora. Nocardia can, however, cause brain
abscesses as well, but it is acquired from the environment.)
2. The answer is D.  Coxiella burnetii is a rickettsia-like organism that can be spread via amni-
otic fluid, aerosols, or dust particles. It withstands drying and thus can be transmitted at least
10 miles by the wind.
3. The answer is E.  Epiglottitis is a medical emergency requiring hospitalization. It can be fatal
in 24 hours. Pediatric cases were almost always caused by H. influenzae type b and have been
dramatically reduced by the conjugate vaccine.
4. The answer is C.  Staphylococcus aureus and Pseudomonas aeruginosa are two primary
­pulmonary colonizers that cause pneumonia in patients with cystic fibrosis. (Staphylococcus
is u ­ sually only in young CF patients.) Of the two, Pseudomonas is Gram-negative. Its slime
­material (­alginate) produces the resistance to phagocytic killing and poor penetration of
­antibiotics to the site, which, in conjunction with the antibiotic resistance of Pseudomonas,
make these s­ erious infections.
5. The answer is C. Both Pseudomonas exotoxin A and diphtheria toxin inhibit protein synthesis
through the inhibition of elongation factor (EF-2). Incorrect choices include: Shiga toxin, which
is a cytotoxin, enterotoxin, and neurotoxin. Vibrio cholerae enterotoxin and E. coli labile toxin
(LT) both result in increased cyclic adenosine monophosphate (cAMP).
6. The answer is A.  Mtb’s cord factor helps trigger the Th1 response, which helps contain the
infection.
7. The answer is B.  Clostridium difficile has been shown to be the major causative agent of
­pseudomembranous colitis, which causes diarrhea that most commonly starts after 3 to 4 days
of antibiotic administration.
8. The answer is B.  Vibrio cholerae causes classic cholera, which is not generally mild or self-
limited; Vibrio parahaemolyticus, in contrast, causes a relatively mild gastroenteritis. It is also
found in raw fish. Sal. typhi is the causative agent of typhoid. Shigellae infections are always
invasive and generally will have a little pus in the stool.
9. The answer is D.  Most transmission in the United States is from an infected flea bite (a choice
not given in the question). The other route of transmission is through respiratory droplets from
patients who have developed pneumonic emboli and pneumonia.
10. The answer is B.  Staphylococcus epidermidis is ubiquitous as part of the normal flora. Organ-
isms are introduced into the host during invasive procedures. Staph. aureus is more likely to be
acute, with high fever and damage developing more quickly.
11. The answer is D.  Clostridium botulinum found in household dust or honey was ingested by
the baby and the spores germinated in her GI tract because her normal flora was not sufficient
to suppress the germination. It is the vegetative cells that produce the botulinum toxin. Antibi-
otics disrupt normal flora, prolonging the disease, but administration of human antitoxin can
dramatically reduce the length of the hospital stay.
12. The answer is D.  The Gram-positive organism Streptococcus pneumoniae contains no
­endotoxin. It is not phagocytosed in the immunologically naive, eliminating choice B. It is the
­capsule that is considered the most important virulence factor.

110
Chapter 4  Bacterial Diseases 111

13. The answer is A.  Enterococci can be differentiated by their reactivity with group D antiserum,
bacitracin resistance, and growth in 40% bile or pH 9.6.
14. The answer is D.  Sal. typhi has only human hosts.
15. The answer is C.  If the mother is young and has had multiple sexual partners, she is more
likely to be colonized with Group B streptococci. If the labor is prolonged after rupture of the
membranes, the baby is more likely to be infected. And, since she delivered before her due date
and had her baby at home, she was not screened for Group B streptococci and did not receive
intrapartum antibiotics to prevent infection of the baby. The other descriptions belong to:
(A) Enterococcus, (B) Strep. pneumoniae, and (D) Viridans strep.
16. The answer is B.  A major survival and virulence factor of Helicobacter pylori is urease, which
neutralizes stomach acid to allow the organism to survive to reach the tissue.
17. The answer is B.  Of the answer choices, only streptococci and staphylococci are
­Gram-positive. The streptococci are catalase-negative and staphylococci are catalase-positive.
Of the two staphylococci, Staphylococcus aureus is the beta-hemolytic, coagulase-positive
organism.
Review Test

Directions: Each of the numbered items or incomplete statements in this section is followed by ­answers
or completions of the statement. Select the ONE lettered answer that is BEST in each case.

1.  Clinical viral disease 6.  Linear, single-stranded DNA is the genetic
(A) Is most frequently due to toxin material of
production (A) Caliciviruses
(B) Usually follows virus infection (B) Flaviviruses
(C) Can result without infection of host cells (C) Papillomaviruses
(D) Is associated with target organs in most (D) Parvoviruses
disseminated viral infections
7.  Host-cell tRNAs are involved in the genome
2.  The eclipse period of a one-step viral replication of
­multiplication curve is defined as the period of (A) Influenza A virus
time between the (B) Retroviruses
(A) Uncoating and assembly of the virus (C) Respiratory syncytial virus
(B) Start of the infection and the first (D) Rhinovirus
­appearance of extracellular virus
(C) Start of the infection and the first 8.  An RNA virus that has a nuclear phase to its
a­ ppearance of intracellular virus replication process is
(D) Start of the infection and uncoating of
(A) Coronavirus
the virus
(B) Rhabdovirus
(C) Retrovirus
3. HTLVs
(D) Togavirus
(A) Are associated with leukemias
(B) Are defective RNA tumor viruses 9.  Negri bodies are associated with
(C) Carry tyrosine protein kinase
oncogenes (A) Cytomegalovirus infections
(D) Synthesize early proteins that interact (B) Herpes simplex virus infections
with p53 Rb (C) Rabies virus infections
(D) Rubella virus infections
4.  Amantadine inhibits
10.  Persistent virus infections
(A) Influenza A and B virus hemagglutinin
binding activity (A) Are usually confined to the initial site of
(B) Influenza A virus M2 protein activity infection
(C) Influenza A and B virus neuraminidase (B) Are preceded by acute clinical
activity disease
(D) Influenza B virus RNA-dependent RNA (C) Elicit a poor antibody response
polymerase activity (D) May involve infected carrier
individuals
5.  Passive immunization is available for
p
­ rotection from 11.  A killed virus vaccine is
(A) Influenza A virus (A) Jeryl Lynn mumps vaccine
(B) Hepatitis A virus (B) Enders measles vaccine
(C) Parainfluenza type 2 virus (C) Salk poliovirus vaccine
(D) Rubella virus (D) Oka varicella-zoster vaccine

141
142 BRS Microbiology and Immunology

12.  The first viral-induced defense mechanism 19.  Disinfection of day care center play tables
in a nonimmune individual is the with 70% ethanol is least likely to affect the
(A) Generation of cytotoxic T lymphocytes ­viability of
(B) Production of interferon (A) Cytomegalovirus
(C) Synthesis of lymphokines (B) Parainfluenza virus
(D) Synthesis of neutralizing (C) Respiratory syncytial virus
antibodies (D) Rotavirus

13.  Localized viral disease 20.  The nanogram level of antigen in serum is
(A) Is a major feature of congenital viral detected by
infections (A) Dot blot tests
(B) Is associated with a pronounced (B) Enzyme-linked immunosorbent assay
viremia (C) Fluorescent antibody staining
(C) Can be associated with carrier (D) Protein-protein hybridization tests
individuals
(D) May have systemic clinical features such 21.  A virus that infects and lyses p
­ rogenitor
as fever erythroid cells causing aplastic crises in
­patients with hemolytic anemia is
14.  Lymphotropic and macrophage (A) California encephalitis virus
t­ rophic designation is important in the (B) Epstein-Barr virus
­pathogenesis of (C) Parvovirus B19
(A) Cytomegalovirus (D) Yellow fever virus
(B) Herpes simplex virus
(C) Human immunodeficiency virus 22.  A viral protein that is thought to i­ nduce
(D) JC virus t­ umors by binding to a cellular tumor
­suppressor protein is
15.  Viral oncogenes are present in (A) Adenovirus E1A
(A) JC virus (B) Epstein-Barr nuclear antigen proteins
(B) Human T-cell lymphotropic virus (C) Hepatitis B virus e protein
type 1 (D) Human immunodeficiency virus gag protein
(C) Rous sarcoma virus
(D) Simian virus 40 23.  Viruses whose genomes have a messenger
(positive-sense) polarity are
16.  Dane particles are associated with (A) Adenoviruses
(A) Hepatitis A virus (B) Papovaviruses
(B) Hepatitis B virus (C) Paramyxoviruses
(C) Hepatitis C virus (D) Polioviruses
(D) Hepatitis E virus
24.  Antiviral nucleoside analogs
17.  The exchange of homologous segments of (A) Are effective only against replicating
RNA between two different influenza type A viruses
viruses is called (B) Include foscarnet
(A) Complementation (C) Inhibit replicases
(B) Genetic reassortment (D) May block viral penetration
(C) Phenotypic masking
(D) Phenotypic mixing 25.  A commercial vaccine consisting of virion
subunits prepared by recombinant technology
18.  The Monospot test is based on exists for
(A) Destruction of Downey cells (A) Hepatitis B virus
(B) Heterophile antibodies (B) Rabies virus
(C) Syncytia inhibition (C) Rotavirus
(D) Viral capsid antigen antibodies (D) Varicella-zoster virus
Answers and Explanations

1. The answer is D.  Many viral infections are asymptomatic or subclinical. Clinical disease,
however, is often associated with viral replication in target organs during disseminated viral
infections.
2. The answer is C.  The period of time between the adsorption and penetration of the virus until
the first appearance of intracellular virus is the eclipse phase.
3. The answer is A.  Human T-lymphotropic virus type 1 (HTLV-1) is associated with adult T-cell
leukemia; HTLV-2 is implicated in human hairy cell leukemia.
4. The answer is B.  Amantadine binds to the M2 protein of influenza A virus, which inhibits this
ion pore and prevents uncoating of the virus.
5. The answer is B.  A commercially available human immune globulin preparation is available
for pre-exposure and postexposure prophylaxis for hepatitis A virus.
6. The answer is D.  Parvoviruses have linear, single-stranded DNA, while papovaviruses have
­circular, double-stranded DNA. Caliciviruses and flaviviruses are RNA viruses.
7. The answer is B.  Host-cell tRNAs act as primers for the synthesis of retrovirus DNA by reverse
transcriptase.
8. The answer is C.  The reverse transcriptase of retroviruses makes a DNA copy of the genomic
RNA. This DNA must be integrated into the host-cell DNA in the nucleus for the remaining
steps in the replication process to occur.
9. The answer is C.  Negri bodies are intracytoplasmic inclusion bodies found in rabies virus-
infected neurons and are important in the diagnosis of infected animals.
10. The answer is D.  Some persistent virus infections, such as serum hepatitis caused by hepatitis
B virus, involve carrier individuals who may or may not have clinical signs of the disease.
11. The answer is C.  Although many of the childhood vaccines like measles, mumps, and
­chickenpox contain live, attenuated virus, the Salk poliovirus vaccine contains killed virus.
12. The answer is B.  The production of interferons that induce the synthesis of antiviral
­replication proteins in neighboring cells occurs before the appearance of any other viral-­
induced immune defense mechanisms.
13. The answer is D.  Although localized infections are not associated with pronounced viremia,
they can have clinical features similar to viremic systemic infections.
14. The answer is C.  Strains of human immunodeficiency virus are classified as lymphotropic or
macrophage trophic depending on their preferred site of latency.
15. The answer is C.  Viral oncogenes are found in many RNA tumor viruses. Both Rous sarcoma
virus and human T-cell lymphotropic virus type 1 are RNA tumor viruses, but only Rous
­sarcoma virus carries an oncogene (v-src).
16. The answer is B.  The spherical virion of hepatitis B virus is called the Dane particle.
17. The answer is B.  Genetic reassortment is the name given to the process whereby homologous
pieces of RNA are exchanged between two different strains of influenza viruses replicating in
the cell.
18. The answer is B.  IgM antibody produced in response to most Epstein-Barr virus infections
­agglutinates sheep and beef erythrocytes and forms the basis for the Monospot test used in
­diagnosing infectious mononucleosis caused by EBV.

143
144 BRS Microbiology and Immunology

19. The answer is D.  Since rotavirus is a naked virus, it is least susceptible to inactivation by
70% alcohol, a lipid solvent disrupting the envelope of the three enveloped viruses.
20. The answer is B.  Enzyme-linked immunosorbent assay (ELISA) is the most sensitive method
of detecting antigens in the serum.
21. The answer is C.  The target cells of human parvovirus B19 are progenitor erythroid cells;
­infections in patients with hemolytic anemia can be serious.
22. The answer is A.  In permissive cells, adenovirus E1A protein is involved in the replication
­process, but in nonpermissive cells it can bind to cellular tumor suppressor protein p110Rb
and inactivate its normal cellular function, which results in cellular transformation.
23. The answer is D.  The genetic material of poliovirus is single-stranded RNA, which can
be translated into a large polyprotein that is subsequently cleaved into the individual viral
proteins.
24. The answer is A.  Nucleoside analogs inhibit viral replication by inhibiting viral DNA synthesis
or function; they do not affect RNA replicases or block penetration.
25. The answer is A.  Both the Recombivax-HB and Engerix-B vaccines for protection from
­hepatitis B virus contain the virus surface antigen prepared from yeast using recombinant
DNA technology.
Review Test

Directions: Each of the numbered items or incomplete statements in this section is followed by
­answers or completions of the statement. Select the ONE lettered answer that is BEST in each case.

1.  A florist presents with a subcutaneous s­ ymptoms. What would you expect to find in a
l­ esion on the hand, which she thinks resulted KOH of skin scrapings?
from a jab wound she received while she was (A) Clusters of yeastlike cells and short curved
­making a sphagnum moss-wire frame for a septate hyphae
floral wreath. The nodule has ulcerated and (B) Hyphae with little branching but ­possibly
not healed d ­ espite use of antibacterial cream, with some hyphae breaking up into
and a new nodule is forming above the original arthroconidia
­lesion. What is most likely to be an appropriate (C) Filariform larvae
­treatment for this infection? (D) Budding yeasts with some pseudohyphae
(A) Oral itraconazole or potassium iodide and true hyphae
(B) Miconazole cream (E) Large budding yeast cells with broad bases
(C) Cortisone cream on the buds and thick cell walls
(D) Oral griseofulvin
(E) Penicillin 5.  A severely neutropenic patient presents with
pneumonia. Bronchial alveolar fluid shows
2.  Although hard to find in the above men- dichotomously branching (generally with acute
tioned nodule, what form would be present in angles), septate hyphae. What is the most likely
the tissue? causative agent?
(A) Lots of hyphae (A) Aspergillus
(B) Long, branching hyphae with acute (B) Cryptococcus
angles (C) Candida
(C) Yeasts with broad-based buds (D) Malassezia
(D) Cigar-shaped to oval yeasts (E) Rhizopus
(E) Yeast with multiple buds (mariner’s
wheel) 6.  What is a mass of fungal filaments called?
(A) Pseudohyphae
3.  A patient presents with paranasal swelling (B) Hyphae
and bloody exudate from both his eyes and (C) Mycelium
nares, and he is nearly comatose. Necrotic (D) Septum
­tissue in the nasal turbinates show nonseptate (E) Yeast
hyphae consistent with Rhizopus, Mucor, or
Absidia (phylum Zygomycota, class Phycomy- 7.  A premature infant on intravenous nutrients
cetes). What is the most likely compromising and high-lipid fluids has developed septicemia
condition underlying this infection? that cultures out on blood agar only when over-
laid with sterile olive oil. What is the most likely
(A) AIDS causative agent?
(B) Ketoacidotic diabetes
(C) Neutropenia (A) Aspergillus
(D) B-cell defects (B) Candida
(E) Chronic sinusitis (C) Cryptococcus
(D) Malassezia
4.  A patient presents with a circular, itchy, (E) Sporothrix
i­ nflamed skin lesion that is slightly raised; it
is on his left side where his dog sleeps next 8.  A filamentous fungus subunit is a
to him. His dog has had some localized a­ reas (A) Coenocyte
of hair loss. The patient has no systemic (B) Hypha

181
182 BRS Microbiology and Immunology

(C) Mycelium pseudohyphae. What is the causative agent of


(D) Septum this dermatophytic look-alike?
(E) Yeast (A) Candida
(B) Trichosporon
9.  To treat a patient with a life-threatening (C) Trichophyton
fungal infection, you choose an antifungal (D) Malassezia
drug that causes pore formation in the fungal (E) Microsporum
­membrane and actually kills the cells. Which
drug would this be? 14.  A recent immigrant from rural Brazil
(A) Amphotericin B ­ resents with a swollen face and extremely
p
(B) Griseofulvin poor dental hygiene, including loss of an
(C) Ketoconazole adult tooth, which appears to be the focus
(D) Miconazole of the c­ urrent i­ nfection. There are two open
(E) Nystatin ulcers on the ­outside of the swollen cheek.
Small y­ ellow “grains” are seen in one of the
10.  A 15-year-old dirt-bike rider visit- ulcers. Gram stain shows purple-­staining fine
ing s­ outhern California the first time has ­filaments. What is the most likely disease?
­developed pneumonia. The causative ­organism (A) Actinomycotic mycetoma
has e­ nvironmental form that consists of (B) Chromomycosis
­hyphae that break up into arthroconidia, (C) Eumycotic mycetoma
which ­become airborne. What is the agent? (D) Sporotrichosis
(A) Aspergillus fumigatus (E) Paracoccidioidomycosis
(B) Blastomyces dermatitidis
(C) Coccidioides immitis 15.  A patient who is a recent immigrant
(D) Histoplasma capsulatum from a tropical, remote, rural area with no
(E) Sporothrix schenckii medical care is now working with a group of
­migrant crop harvesters. He has a large, raised,
11.  Which of the following drugs inhibits ­colored, cauliflower-like ankle lesion. Darkly
e­ rgosterol synthesis, is important in t­ reating ­pigmented, yeastlike sclerotic bodies are seen
Candida fungemias, and is used orally to in the tissue biopsy. Which of the following is
­suppress relapses of cryptococcal meningitis in the most likely diagnosis?
AIDS patients? (A) Actinomycotic mycetoma
(A) Amphotericin B (B) Chromoblastomycosis
(B) Fluconazole (C) Eumycotic mycetoma
(C) Griseofulvin (D) Sporotrichosis
(D) Echinocandins (E) Tinea nigra
(E) Nystatin
16.  A premature baby, now 4 days old, has
12.  A patient has splotchy hypopigmentation ­ eveloped a white coating on her buccal
d
on the chest and back with only slight itchiness. ­mucosa extending onto her lips. It appears to be
What is most likely to be seen on a KOH mount painful. What is the most likely causative agent?
of the skin scraping? (A) Actinomyces
(A) Yeasts, pseudohyphae, and true (B) Aspergillus
hyphae (C) Candida
(B) Filaments with lots of arthroconidia (D) Fusobacterium
(C) Clusters of round fungal cells with short, (E) Microsporum
curved, septate hyphae
(D) Darkly pigmented, round cells with sharp 17.  Which of the following stains allows
interior septations ­ ifferentiation of fungus from human tissue
d
(E) Cigar-shaped yeasts by staining the fungus a pink-red color?
(A) Calcofluor white stain
13.  A patient has a dry, scaly, erythematous (B) Gomori methenamine-silver stain
penis. Skin scales stained with calcofluor white (C) Periodic acid-Schiff stain
show fluorescent blue-white yeasts and a few (D) Hematoxylin and eosin stain
Chapter 8   Fungal Diseases 183

18.  A normally healthy 8-year-old boy from (C) Coccidioides immitis


Florida is visiting friends on a farm in Iowa (D) Histoplasma capsulatum
during the month of July. He presents on July (E) Sporothrix schenckii
28 with a fever, cough, and lower respiratory
symptoms (no upper respiratory tract symp- 21.  What is the scientific name for a fungal
toms). He has been ill for 4 days. His chest cross wall?
sounds are consistent with pneumonia, so a (A) Coenocyte
chest radiograph is obtained. The radiograph (B) Hypha
shows small, patchy infiltrates with hilar (C) Mycelium
­adenopathy. His blood smear shows small, (D) Septum
nondescript yeast forms inside monocytic cells. (E) Yeast
What is the most likely causative agent?
(A) Aspergillus fumigatus 22.  A noncompliant, human immunode-
(B) Blastomyces dermatitidis ficiency virus (HIV)-positive ­patient has
(C) Coccidioides immitis been complaining of a stiff neck and a
(D) Histoplasma capsulatum ­severe ­headache. The headache was initially
(E) Pneumocystis jiroveci ­lessened by analgesics, but the analgesics
are no longer effective. His current CD4+
19.  Which of the following is a polyene antifun- count is 180/mm3. He is not on any
gal agent used for many life-threatening fungal ­prophylactic drugs. What is the most likely
infections? causative agent?
(A) Amphotericin B (A) Aspergillus
(B) Griseofulvin (B) Cryptococcus
(C) Itraconazole (C) Candida
(D) Miconazole (D) Malassezia
(E) Nystatin (E) Sporothrix

20.  A logger undergoing chemotherapy for 23.  Which of the following features differenti-
cancer has developed pneumonia and skin ates fungal cells from human cells?
­lesions. Biopsy of the skin lesions demonstrates (A) 80S ribosomes
the presence of large yeasts with thick cell walls (B) Presence of an endoplasmic reticulum
and broad-based buds. What is the most likely (C) Ergosterol as the major membrane sterol
causative agent? (D) Enzymes that allow them to use carbon
(A) Aspergillus fumigatus dioxide as their sole carbon source
(B) Blastomyces dermatitidis (E) Presence of chloroplasts
Answers and Explanations

1. The answer is A.  This is a classic case of lymphocutaneous sporotrichosis in which a g­ ardener
or florist is infected via a puncture wound. The drug of choice is either itraconazole or
­potassium iodide (administered orally in milk). Topical antifungals are not effective, and the
­cortisone cream would probably enhance the spread of the disease. Griseofulvin localizes in
the keratinized tissues and would not halt the subcutaneous spread of this infection. Penicillin
would have no effect because Sporothrix is not a bacterium.
2. The answer is D.  This is a classic case of lymphocutaneous sporotrichosis. Sporothrix schenckii
is dimorphic; the tissue form is cigar-shaped yeasts, but they are hard to find by histology.
3. The answer is B.  Zygomycota are aseptate fungi that cause serious infections, primarily in
ketoacidotic diabetic patients and cancer patients. Fungal infections common in AIDS patients
include Candida infections (ranging from oral thrush early to fungemias later), cryptococcal
meningitis, and disseminated histoplasmosis and coccidioidomycosis. Severely neutropenic
patients are most likely to have invasive Aspergillus infections.
4. The answer is B.  The case is ringworm acquired from a dog. In tissue, any of the dermato-
phytes would show hyphae and arthroconidia. Pityriasis versicolor would have the clusters of
yeasts with short, septate, curved hyphae (spaghetti and meatballs appearance). A filariform
larvae would only be characteristic of dog hookworm, which is usually acquired from walking
barefoot where there are dog feces. It would not be acquired from sleeping with the dog, and
would not cause hair loss in the dog. Choice D describes Candida, which does not fit the case.
Choice E would describe Blastomycosis, which is highly unlikely.
5. The answer is A.  Aspergillus spores are commonly airborne. Invasive infections with
­Aspergillus are controlled by phagocytic cells. In severe neutropenia, risk of infection is high.
6. The answer is C.  A mycelium is a mass of hyphae (fungal filaments).
7. The answer is D.  Malassezia furfur is a lipophilic fungus that is found on skin. It causes
­fungemia, primarily in premature infants on high-lipid intravenous supplements.
8. The answer is B.  The fungal subunit, called a hypha, is a filamentous structure with or without
cross walls (septae).
9. The answer is A.  Although both amphotericin B and nystatin are polyenes, only a­ mphotericin
B is used systemically. The imidazoles inhibit ergosterol synthesis, and griseofulvin, which
localizes in the keratinized tissues, inhibits the growth of dermatophytes by inhibiting microtu-
bule assembly.
10. The answer is C.  Coccidioides immitis is found in desert sand, primarily as arthroconidia and
hyphae.
11. The answer is B.  Fluconazole is an imidazole; all imidazoles inhibit ergosterol synthesis.
­Fluconazole has become the mainstay in the treatment of serious Candida infections, and it is
used to prevent relapse of fungal CNS infections in compromised patients. Amphotericin B and
nystatin both bind to ergosterol and create membrane pores, causing cell leakage and death.
Echinocandins inhibit the fungal cell wall synthesis. Griseofulvin is not used against Candida
as it may make the infection worse.
12. The answer is C.  Malassezia furfur is seen in tissues as clusters of round fungal cells with
short, curved septate hyphae (spaghetti and meatballs appearance) and is the causative agent
of pityriasis or tinea versicolor; M. furfur overgrowth causes pigmentation disturbances.

184
Chapter 8   Fungal Diseases 185

13. The answer is A.  Candida may cause skin infections that resemble some dermatophytic infec-
tions. The patient described in the question has Candida balanitis. In tinea cruris, the penis is
not usually involved.
1 4. The answer is A.  The disease syndrome is lumpy jaw, which is a form of mycetoma. The loca-
tion of the lesions and presenting signs seen in this patient suggest actinomycotic mycetoma,
a bacterial infection caused by the Actinomyces part of the gingival crevices flora. (Students:
You needed a nonfungal question!) Yeasts will also stain Gram-positive. Remember that
­Actinomyces is a Gram-positive anaerobic bacterium that is not acid-fast.
15. The answer is B.  The finding of dematiaceous (dark), yeastlike sclerotic bodies that have sharp
planar division lines and the clinical presentation are both characteristic of chromoblastomy-
cosis. Tinea nigra would show dematiaceous hyphae in flat palmar or plantar lesions.
16. The answer is C.  The disease described is thrush, and it is caused by Candida.
17. The answer is C.  Calcofluor white stain, Gomori methenamine-silver stain, and periodic
acid-Schiff stain are all differential stains, but only the periodic acid-Schiff stain turns fungi a
pink-red color. The hematoxylin and eosin stain turns fungi a pink-red color also but does not
differentiate between the fungi and human tissue, so it is not a correct answer.
18. The answer is D.  Histoplasma and Blastomyces are both endemic in Iowa (central United
States bordering the Mississippi River), but only Histoplasma fits the description of a facultative
intracellular parasite circulating in the reticuloendothelial system.
19. The answer is A.  Amphotericin B, a polyene, is the most effective treatment for many
­life-threatening fungal infections. Nystatin, also a polyene, is used topically or orally, but is not
absorbed.
20. The answer is B.  Blastomyces has a double refractile wall and buds with a broad base of
­attachment to the mother cell. The environmental association appears to be rotting wood.
21. The answer is B.  The cross wall of a hypha is called a septum or septation.
22. The answer is B.  Cryptococcus, an encapsulated yeast, is the major causative agent of menin-
gitis in patients with AIDS.
23. The answer is C.  Ergosterol is the major fungus membrane sterol, and its presence is impor-
tant in chemotherapy of fungal infections. For example, amphotericin B binds to ergosterol,
producing pores that leak out cellular contents, killing the fungus. Imidazole drugs inhibit the
synthesis of ergosterol. Both fungi and humans have 80S ribosomes and endoplasmic reticu-
lum. Fungi are heterotrophic rather than autotrophic and thus cannot use carbon dioxide as
their carbon source; instead, fungi break down organic carbon compounds. Fungi are also not
photosynthetic.
Review Test

Directions: Each of the numbered items or incomplete statements in this section is followed by
­answers or completions of the statement. Select the ONE lettered answer that is BEST in each case.

1.  A biology graduate student who recently (C) Hymenolepis nana


visited a tropical region of Africa presents with (D) Dipylidium caninum
new visual impairment and the sensation that (E) Taenia solium
something is moving in her eye. She tells you
that she is concerned because she had been 4.  Which of the following protozoans is free
warned about eye disease transmitted by black l­ iving and is such that acquisition does not
flies. When in Africa, she was in a river area, generally indicate fecal contamination?
and despite her best efforts she received a lot of
(A) Acanthamoeba
black fly bites. She also has some subcutaneous
(B) Dientamoeba fragilis
nodules. If her infection was acquired by black
(C) Entamoeba histolytica
fly bite, what is the most likely causative agent?
(D) Entamoeba coli
(A) Ancylostoma braziliense (E) Giardia
(B) Dracunculus medinensis
(C) Loa loa 5.  A 26-year-old woman with uncomplicated
(D) Onchocerca volvulus malaria who was treated initially with chloro-
(E) Wuchereria bancrofti quine now has relapsed. What is the reason
for a chloroquine-treated case of Plasmodium
2.  A woman who imports food from Mexico vivax relapsing?
and spends several months per year in
­rural Mexico had to have a compound leg (A) P. vivax has a significant level of chloro-
fracture pinned and set in Mexico and has quine resistance.
returned 3 days later. She now has signs of (B) P. vivax has a persistent erythrocytic
acute ­appendicitis and is taken to surgery in stage.
­Houston. When her appendix is removed, it is (C) P. vivax has a persistent exoerythrocytic
found to contain a light-colored, 20.5-cm-long stage (hypnozoite).
roundworm as well as bile-stained, knobby (D) Chloroquine is not one of the drugs of
eggs ­consistent with Ascaris. How did she choice.
­acquire this infection?
6.  How is Leishmania donovani transmitted?
(A) Ingestion of water containing filariform
larvae (A) Anopheles mosquito bite
(B) Skin penetration by filariform larvae (B) Black fly bite
(C) Skin penetration by rhabditiform larvae (C) Culex mosquito bite
(D) Ingestion of food contaminated with (D) Sandfly bite
the eggs (E) Skin penetration by trauma
(E) Inhalation of dust carrying the cysts
7.  How is Schistosoma haematobium
3.  A patient whose major source of protein is transmitted?
smoked and cooked fish develops what a­ ppears (A) Ingestion of raw or undercooked snail,
to be pernicious anemia. What parasite is noted frog, or snake
for causing a look-alike vitamin B12 ­anemia (B) Invasion of filariform larvae from soil
in certain genetically predisposed infected (C) Handling aquatic birds
individuals? (D) Standing or swimming in contaminated
(A) Echinococcus granulosus water
(B) Diphyllobothrium latum (E) Tsetse fly bite

201
202 BRS Microbiology and Immunology

8.  An untreated AIDS patient (CD41 count of (D) Taenia saginata


180 cells/mm3) from southern California has (E) Toxocara canis
developed a progressively severe headache
and mental confusion, along with ataxia and 12.  How is Clonorchis sinensis (Chinese liver
retinochoroiditis. Focal lesions are present fluke) most likely transmitted to humans?
on a computed tomography scan of his brain. (A) Fish ingestion
No mucocutaneous lesions are found. He has (B) Mosquito bite
been living under a bridge for the past 2 years. (C) Swimming or water contact
His level of immunoglobulin G (IgG) to the (D) Rare beef ingestion
infectious agent is high. What is the most likely (E) Mango fly (Chrysops)
explanation for how this current infection
started? 13.  A 48-year-old subsistence farmer from rural
(A) Earlier exposure to pigeons Brazil dies of heart failure. His autopsy shows a
(B) Earlier exposure to desert sand greatly enlarged heart. What was the vector for
(C) Reactivation of bradyzoites in cysts from the most likely infectious agent that may have
an earlier infection been responsible for his death?
(D) Recent exposure to cat feces (A) Ixodes tick
(E) Recent exposure to bats (B) Mosquito
(C) Reduviid bug
9.  Which of the following is the tapeworm (D) Sandfly
­acquired from eating undercooked pork? (E) Tsetse fly
(A) Dipylidium spp.
(B) Echinococcus granulosus 14.  A 16-year-old man who recently returned
(C) Taenia saginata from camping in Canada presents with fatty
(D) Taenia solium diarrhea and acute abdominal pain following
(E) Trichinella spiralis many meals. How does the most likely agent
cause the diarrhea?
10.  What roundworm is most likely to be (A) Coinfection with bacteria
t­ ransmitted by ingestion of food or water (B) Enterotoxin production
­contaminated with feces? (C) Suction disk attachment
(A) Ascaris lumbricoides (D) Tissue invasion leading to an
(B) Enterobius vermicularis ­inflammatory response and prostaglandin
(C) Necator americanus production
(D) Taenia saginata
(E) Toxocara canis 15.  Which of the following protozoans is
t­ ransmitted primarily by the motile trophozoite
11.  What roundworm is transmitted by form?
f­ ilariform larvae that are found in the soil and (A) Balantidium coli
penetrate the skin? (B) Entamoeba histolytica
(A) Dracunculus medinensis (C) Giardia lamblia
(B) Enterobius vermicularis (D) Taenia solium
(C) Strongyloides stercoralis (E) Trichomonas vaginalis
Answers and Explanations

1. The answer is D.  Onchocerca volvulus causes river blindness and is transmitted by the bite of a
black fly. The patient may be able to detect movement of the parasite in the eye.
2. The answer is D. Fertilized Ascaris eggs released in feces may contaminate food or water,
which is then consumed. Ascaris does not attach to the intestine but maintains its position by
mobility. The worm may become hypermotile (e.g., during febrile periods, anesthetic use, or
antibiotic use) and may migrate into the appendix or bile duct.
3. The answer is B.  Diphyllobothrium is the tapeworm associated with anemia. It is transmitted
in fish found in cool lake regions.
4. The answer is A.  Acanthamoeba is a free-living organism with a sturdy cyst stage that is found
in dust. A common way of acquiring Acanthamoeba infections in the United States is through
homemade saline solutions for soft contact lenses. Giardia may be from animal contamination
of water (rather than human) but is still not probably truly free living.
5. The answer is C. Both Plasmodium ovale and Plasmodium vivax may have resting liver
forms, which are very slow to develop into schizonts with merozoites and proceed onto the
­chloroquine-sensitive erythrocytic stages after treatment is over. (It is not over with P. ovale or
P. vivax unless you also treat with primaquine phosphate, which kills the liver stages.) (Papua,
New Guinea, and Indonesia now have chloroquine-resistant Plasmodium vivax.)
6. The answer is D.  All leishmaniae are transmitted by sandflies.
7. The answer is D.  All schistosomes are transmitted by skin penetration from standing or
­swimming in contaminated water. Remember that snails are intermediate hosts.
8. The answer is C.  The most likely disease in this case is encephalitis with focal lesions. Because
the patient has high levels of immunoglobulin G, the current infection is likely a reactivation
of an earlier infection; therefore, recent exposures (choices D and E) can be eliminated. Expo-
sure to pigeons suggests cryptococcosis, which is often a reactivational infection. However, in
cryptococcosis antibody levels are rarely monitored, and there is no mention of India ink stain
or capsular polysaccharide in the cerebrospinal fluid, which are the major diagnostic methods.
In addition, based on the patient’s symptoms, the infection is more likely to be encephalitis
rather than meningitis or meningoencephalitis; also, retinochoroiditis is usually not present in
cryptococcosis. The retinochoroiditis and lack of mucocutaneous lesions makes infection with
Coccidioides less likely. Reactivation of toxoplasmosis is most likely.
9. The answer is D.  If you answered Trichinella spiralis, you fell for a typical testing “bait and
switch.” T. spiralis is the pork roundworm and Taenia solium is the pork tapeworm. Dipylidium
caninum is the common tapeworm of both cats and dogs. It may be transmitted by ingestion of
fleas harboring cysticercoid larvae. Transmission to humans usually occurs when crushed fleas
harboring the disease are transmitted from a pet when it licks a child’s mouth.
10. The answer is A.  Ascaris lumbricoides is transmitted via the fecal-oral route. Enterobius is
most likely transmitted via contaminated hands, clothing, or bedding. Necator enters by skin
­ enetration. Taenia is not a roundworm. Toxocara is most commonly acquired from eating
p
­fecally contaminated dirt or soil.
1 1. The answer is C.  Strongyloides stercoralis is a type of hookworm (also a roundworm). The
filariform larvae of S. stercoralis are acquired when walking barefoot or sitting on the ground.
Dracunculus medinensis (the guinea worm) is acquired by drinking water with copepods
­containing the larvae. Filtration of all drinking water through clean sari silk or T-shirt material
has reduced the incidence of new cases dramatically and may allow its eradication. (For those

203
204 BRS Microbiology and Immunology

who are infected with D. medinensis, adults in subcutaneous nodules are slowly removed by
rolling them out on a pencil.) Toxocara canis and Toxocara cati are acquired most commonly
by pica, the ingestion of inert material; in this case, dirt or sand with animal feces. Taenia
­saginata is a flatworm. Enterobius (pinworm) eggs are ingested.
12. The answer is A.  Raw, undercooked, smoked, or pickled freshwater fish are the most common
route of transmission of Clonorchis sinensis. You should be able to answer this question from
the general information in the preceding chapter without any specifics. The question told you
it was a fluke, and so you know that water was involved in transmission; however, it is not a
­Schistosoma sp., so it has to be ingestion of aquatic plant or animal and in this case it is fish.
13. The answer is C.  The case of the Brazilian farmer is a classic description of heart failure from
chronic Chagas’ disease, which is caused by Trypanosoma cruzi. T. cruzi is transmitted by
reduviid bugs (cone-nose bugs or kissing bugs) that defecate as they bite. Scratching the bite
spreads the trypanosome into the bite site, initiating the infection.
14. The answer is C.  In this case, Giardia lamblia is the causative agent. G. lamblia is carried by
muskrats and beavers, which is why it can be picked up in pristine northern lakes, such as
those found in Canada. Attachment of numerous Giardia via their ventral sucking disks in the
duodenal-jejunal area leads to malabsorption diarrhea and temporary lactose intolerance.
1 5. The answer is E.  Protozoans transmitted by the fecal-oral route are transmitted in the cyst
form, which survives stomach acid. Only the sexually transmitted Trichomonas vaginalis is
transmitted in the motile form. Taenia solium is not a protozoan, but a flatworm.
Review Test

Directions: Each of the numbered items or incomplete statements in this section is followed by
­answers or completions of the statement. Select the ONE lettered answer that is BEST in each case.

1.  One week postdelivery, an infant boy (C) Nocardia asteroides


­ ecomes extremely irritable and continuously
b (D) Pseudomonas aeruginosa
rubs his right eye, which contains a mucopuru-
lent exudate. An examination of the eye shows 5.  There is an outbreak of gastrointestinal
papillae on the conjunctiva. No bacteria are ­ isease involving six children at a day care
d
seen in the exudate. The most likely causative center. The children all have symptoms of
infectious agent is fever, watery stool, and vomiting. R ­ ecent
(A) Candida albicans ­activities at the center involved p
­ laying
(B) Chlamydophila trachomatis with animals including kittens, turtles,
(C) Cytomegalovirus birds, and hamsters. What types of bacteria
(D) Herpes simplex virus are most likely to be o­ bserved in
the stool?
2.  A 58-year-old man presents at your o
­ ffice (A) Gram-positive cocci
complaining of extreme pain in his right (B) Gram-negative cocci
­testicle. Physical examination shows consid- (C) Gram-positive rods
erable swelling. A small amount of urethral (D) Gram-negative rods
exudate could be aspirated. Microscopic
­examination and Gram staining showed the 6.  A 75-year-old man is brought by his
presence of bacteria. Those bacteria most ­ aughter to the community health center.
d
likely are He appears confused and has a stiff neck
(A) Gram-negative cocci and ­severe headache. She indicates that the
(B) Gram-negative diplococci ­previous week he had a runny nose and other
(C) Gram-positive cocci symptoms consistent with sinusitis. What
(D) Gram-positive rods ­organism would you expect to find in his cere-
brospinal fluid?
3.  A 22-year-old woman presents with (A) Coxsackie virus
s­ ymptoms of vaginitis, including a yellowish (B) Haemophilus influenzae
discharge. She claims to have not been ­sexually (C) Neisseria meningitis
active for 18 months. When KOH is added to (D) Streptococcus pneumoniae
the discharge, a “fishy” odor appears. She is
most likely infected with 7.  While on winter semester break, a
(A) Candida albicans 2­ 1-year-old male college student appears at
(B) Chlamydophila trachomatis his family physician’s office with several skin
(C) Gardnerella vaginalis ­lesions on his right arm. They consist of a
(D) Herpes simplex virus 2 ­central pustule within a raised erythematous
area. He reports that the only recent unusual
4.  A known alcoholic appears at the emergency activities that have involved skin exposure
department with chest pain, a 40°C/104°F have been hot tub parties at his friend’s
­fever, chills, and a productive cough. He has no cabin during the previous two weekends.
history of chronic pulmonary disease. There What is the most likely causative infectious
are red blood cells and encapsulated Gram- agent?
negative rods in the sputum. The symptoms are (A) Pseudomonas aeruginosa
most likely caused by (B) Streptococcus pyogenes
(A) Haemophilus influenzae (C) Staphylococcus aureus
(B) Klebsiella pneumoniae (D) Papillomavirus
235
236 BRS Microbiology and Immunology

8.  In July, you are called to a rural nursing 9.  An 18-month-old female infant is
home where several of the residents have brought to the emergency department by
­developed a gastrointestinal disease c­ onsisting her mother. The mother reports an abrupt
of fever, vomiting, and bloody stool. Lab onset of s­ ymptoms consisting of continual
­analysis of the stool shows the presence of ­crying, high fever (40°C/104°F), and a cough
PMNs and Gram-negative rods. Epidemiologi- ­producing rusty ­colored sputum. What would
cal discussions with the residents indicate they you ­expect a chest x-ray and lab findings
all shared the same raw milk brought to one to show?
resident by his farmer son. The most likely (A) Lobar pattern and Gram-positive cocci
cause of the infection is (B) Lobar pattern and Gram-negative cocci
(A) Campylobacter jejuni (C) Bronchopneumonia pattern and
(B) Clostridium perfringens Gram-positive cocci
(C) Staphylococcus aureus (D) Bronchopneumonia pattern and
(D) Vibrio parahaemolyticus Gram-negative rods
Answers and Explanations

1. The answer is B.  This case of neonatal conjunctivitis is most likely caused by Chlamydophila
trachomatis since no bacteria are present and papillae rather than follicles are observed. The
mother was probably the source of the infection.
2. The answer is C. Both Staphylococcus epidermidis and various Enterobacteriaceae can cause
epididymitis in men over the age of 35 years, but Gram-negative rods is not a choice; therefore,
Gram-positive cocci is correct.
3. The answer is C.  Gardnerella vaginalis and Candida albicans are the non-STD choices.
Gardnerella vaginalis is correct because it produces the fishy odor when KOH is added to the
discharge.
4. The answer is B.  Klebsiella pneumoniae is associated with pneumonia in alcoholics. The other
Gram-negative rod (Pseudomonas aeruginosa) is associated with chronic pulmonary disease in
the immunocompromised.
5. The answer is D.  The most likely bacteria to cause gastrointestinal disease with the symptoms
and circumstances provided is a species of Salmonella, which are Gram-negative rods.
6. The answer is D.  The symptoms of confusion and severe headache point to a case of bacterial
meningitis. With preceding sinusitis in an elderly man, Streptococcus pneumoniae is the most
likely cause. The other two bacteria are more often observed in infants or young adults.
7. The answer is A.  Pseudomonas aeruginosa is associated with an infection known as
“­whirlpool” or “hot tub” folliculitis. Inappropriate care of whirlpools, hot tubs, and swimming
pools can allow this organism to grow and enter the skin through small breaks.
8. The answer is A.  The symptoms and lab analysis of the stool are consistent with
­Campylobacter jejuni infection. The raw milk was most likely the source of the infection.
9. The answer is A.  The most common cause of CAP in infants and elderly is S ­ treptococcus
p
­ neumoniae. The symptoms in this case are consistent, and the rusty colored sputum is
­suggestive of this bacteria. It is a Gram-positive cocci with a lobar pneumonia on a chest x-ray.

237
3
Microbiology

03_USMLE-STEP1_ch03_039-088.indd 39 9/17/18 3:30 PM


40 CHAPTER 3 Microbiology

CASE 1
A 19-year-old university student presents with eye pain to the campus nurse. He has been pulling all-nighters for
his upcoming exams. While trying to study, he often finds himself falling asleep with his contact lenses still in; he
rinses them with tap water occasionally. As finals week ends, he is scratching his eyes often because they feel dry and
painful, as if there are foreign bodies in them. He also notices that his eyes are red and that he tears up frequently.
These symptoms have occurred gradually during the week. He has no associated upper respiratory symptoms, and
no recent sexual activity.

What is the most likely diagnosis?


Keratitis, caused by the free-living ameba Acanthamoeba, is the most likely diagnosis. Conditions to consider in the
differential diagnosis include herpes simplex virus (HSV), herpes zoster virus, other viruses, and bacterial or fungal
infection. However, HSV keratitis would present acutely and progress rapidly to photophobia, decreased visual
acuity, and dendritic ulcer formation, which are not seen in this patient. Likewise, a bacterial or other viral keratitis
would tend to a more-acute presentation, while a fungal keratitis might present similarly but is comparatively rare.
Bacterial orbital cellulitis would be an ophthalmologic emergency.

What are the risk factors for developing this condition?


The number one risk factor for Acanthamoeba infection is extended wearing of contact lenses. Inadequate
disinfection of the lenses with tap water or homemade saline solution and wearing lenses while swimming or
showering can also predispose contact lens wearers to this infection. Acanthamoeba organisms are common in
soil, air, and water and are resistant to chlorine.

What are other symptoms and complications of this condition?


Unlike bacterial keratitis, keratitis from Acanthamoeba takes days or weeks to cause symptoms. The initial
symptoms are usually redness and a feeling of a foreign body in the eye. Blurring of vision may also be present.
Over time, this progresses to pain, lid edema, and conjunctival infection. If untreated, increased intraocular
pressure, cataracts, and even loss of vision can develop.

How is this condition diagnosed?


The diagnosis is made by slit-lamp examination of the eye, which shows thickened epithelium and rough corneal
nerves. A characteristic ring on the cornea may also appear approximately 6 weeks after initial infection. Corneal
scraping or biopsy reveals irregular polygonal Acanthamoeba cysts.

What is the treatment for this condition?


Initial treatment consists of topical antimicrobials such as miconazole and neomycin for several months. If the
infection has been left untreated (ie, at the corneal ring stage), surgery, such as corneal debridement and possibly
corneal transplantation, is usually required.

What populations are at risk for systemic infection with this organism?
Patients with significant immunosuppression, such as those with lymphoproliferative disorders, patients on
chronic steroids, patients receiving chemotherapy, and patients with AIDS may develop systemic infections
with Acanthamoeba and other free-living amebae. The central nervous system is frequently involved (termed
granulomatous amebic encephalitis). These patients may present with changes in mental status, headache, and
stiff neck. They can also develop cranial nerve palsies, ataxia, and hemiparesis. Treatment is urgent in such cases,
and the mortality rates are high.

03_USMLE-STEP1_ch03_039-088.indd 40 9/17/18 3:30 PM


Microbiology CHAPTER 3 41

CASE 2
A 63-year-old man with no past medical history presents to the physician with jaw pain that began 1 month ago. On
examination, the right side of the jaw is asymmetric to the left side of the jaw. Incision and drainage is done under
local anesthesia, and a yellow discharge is noticed and collected. A fluid sample of the discharge is collected for
Gram stain and culture. The test reveals a gram-positive rod that forms long, branching filaments.

What are the possible bacterial microorganisms found in this sample?


Actinomyces and Nocardia both fit this description. Although they somewhat resemble fungi on Gram stain, they
are both bacteria, not fungi. They appear as characteristic gram-positive rods with long, branching filaments.

How are these two microorganisms differentiated?


Actinomyces israelii is a non-acid-fast filamentous, anaerobic organism (see Figure 3-1) and has characteristic “sulfur
granules.” Nocardia, on the other hand, is modified-acid-fast positive and an obligately aerobic organism (see
Figure 3-2).

FIGURE 3-1. Actinomyces on Gram stain. (Reproduced FIGURE 3-2. Gram-positive aerobic Nocardia asteroi-
courtesy of Centers for Disease Control and Prevention/ des slide culture revealing chains of bacteria among
Dr. Lucille Georg.) aerial mycelia. (Reproduced courtesy of Centers for
Disease Prevention and Control/Dr. Lucille George.)

What are the other obligate aerobic and anaerobic organisms?


The most common aerobic organisms are Nocardia, Pseudomonas, Mycobacteria, and Bacillus species (mnemonic:
“Nagging Pests Must Breathe”).
Some common anaerobic organisms are Clostridium, Bacteroides, and Actinomyces species (mnemonic: “Can’t
Breathe Air”).

After paging the intern, the pathologist learns the sample was drained from an oral abscess. Now which of
these two microorganisms is more likely?
Actinomyces is more likely because it is part of the normal oral microbiota and frequently forms abscesses in the
mouth or gastrointestinal tract after trauma. Actinomyces often forms abscesses that drain though sinus tracts.
Nocardia is an opportunistic infection seen primarily in compromised hosts, and most often results in pulmonary
symptoms due to lung abscesses or, rarely, central nervous system symptoms due to brain abscesses.

If this microorganism were found in a sputum sample that stained weakly acid fast, what could be inferred
about the patient’s immune status?
Nocardia is most often found in immunocompromised patients. The diagnosis may be delayed if this organism
is not considered; it may grow too slowly to be detected on routine sputum culture. It may be found on fungal
or Acid-Fast Bacilli (AFB) cultures, but the medium of choice for detection of Nocardia is Buffered Charcoal Yeast
Extract (BCYE) agar.

What is the treatment for each of these infections?


Nocardia is most commonly treated with trimethoprim-sulfamethoxazole, though speciation and antimicrobial
susceptibility testing is recommended. Actinomyces is treated with penicillin G. A mnemonic is SNAP
(Sulfonamides for Nocardia, Actinomyces for Penicillin).

03_USMLE-STEP1_ch03_039-088.indd 41 9/17/18 3:30 PM


42 CHAPTER 3 Microbiology

CASE 3
A 25-year-old man is brought to the ED by ambulance after a motor
vehicle collision. He is lucid but has severe bleeding from his leg. His wife
is with him and reports that the patient is generally healthy, although he
had several bouts of “lung and ear infections” as a child. He suffers from
periodic bouts of diarrhea, so he has been trying a gluten-free diet without
relief. He was referred to a gastroenterologist last week, but he has not
seen her yet. He starts to become pale and less responsive even with the
application of a tourniquet, so the decision is made to transfuse him with
a unit of whole blood. He is given 1 unit of type-matched RBCs after he is
typed and crossed for blood products. Soon afterward he develops a red,
itchy rash over most of his body (see Figure 3-3) and begins to develop
difficulty breathing and hypotension. On chest radiograph, his lungs are
clear, and the bleeding from his leg has stopped. On exam, his abdomen
is non-acute, and he has reassuring heart tones.
FIGURE 3-3. (Reproduced, with permission, from
Sussman G, et al. Allergy, Asthma, and Clinical Im-
munology. 2015;11(1):7.)

What is the likely cause of this patient’s repeated infections and reaction to the blood transfusion?
This patient is having an anaphylactic reaction. IgA is a common component in blood products. This patient
likely has hereditary IgA deficiency and therefore has developed IgG antibodies against IgA. He is particularly
susceptible to gastrointestinal infections, especially giardiasis, for which secretory IgA plays an important
protective role. IgA deficiency can occur as an isolated syndrome or may involve concurrent IgG deficiency, which
increases the risk of sinopulmonary infections.

What is the next step in management of this condition?


Because of the patient’s severe anaphylactic reaction to the transfused blood products, it is imperative to
discontinue transfusion and administer an epinephrine injection. Epinephrine counteracts the bronchospasm and
vasodilation that is causing his respiratory difficulty and decreasing blood pressure.

What is the cause of the patient’s milk allergy?


In the absence of intestinal IgA, large proteins are more likely to enter the bloodstream whole. An IgG antibody
reaction to these proteins can then cause an allergic reaction. (This is different from lactose intolerance, which is
not a true allergy and involves a deficiency of lactase.) For the same reasons, patients with IgA deficiency are at
increased risk of developing antibodies against wheat proteins and thus celiac disease. IgA is found on the mucosa
of the gastrointestinal tract.

What are the stages in B-cell development that lead up to IgA secretion?
Pluripotent stem cells first differentiate into lymphoid stem cells, then to pro-B cells, then to pre-B cells. Pre-B
cells contain the IgM (mu) heavy chains intracellularly but no surface IgM. The next step is formation of immature
or naive B cells that express surface IgM. After stimulation by antigen, the immature cells can mature into
IgM-secreting cells or, with CD4+ T-cell stimulation (CD40 ligand-CD40 receptor activation), can class switch to
express IgG, IgA, or IgE antibodies. After class switching, the cells can undergo affinity maturation to select for
antibodies with higher binding affinities for the antigen and subsequently form plasma cells that secrete the
specialized antibodies. IgA exists in pairs, IgG and IgE exist as single units, and IgM can exist in pentamers.

03_USMLE-STEP1_ch03_039-088.indd 42 9/17/18 3:30 PM


Microbiology CHAPTER 3 43

CASE 4
A 43-year-old local craftsman who makes garments from the hides of goats visits his physician because over the past
few days he has developed disturbing black lesions on his hands and arms (see Figure 3-4). This lesion is not painful,
but he is alarmed by its appearance. He is afebrile and his physical examination is unremarkable.

FIGURE 3-4. (Reproduced with permission from USMLE-Rx.com.)

What is the most likely diagnosis?


Cutaneous anthrax, caused by Bacillus anthracis. The skin lesions are painless and dark or charred ulcerations
known as black eschar (as shown in Figure 3-4). Cutaneous anthrax lesions frequently exhibit edema out of
proportion to the size of the lesion, which can make lesions on the face or neck particularly dangerous. Cutaneous
anthrax is classically transmitted by contact with the hide of a goat or other animal at the site of a minor open
wound.

How will the causative microorganism appear on Gram staining?


B anthracis is a gram-positive spore-forming rod. The spores are resistant to many chemical disinfectants, heat,
ultraviolet light, and drying and are therefore a feared agent of biological warfare. Unlike other bacteria, B anthracis
has a poly-peptide capsule containing D-glutamate, which can sometimes be seen on Gram or India ink staining
from tissue (but not from culture).

What is the other common spore-forming microorganism?


Clostridium species are the other gram-positive spore-forming bacteria. Bacillus and Clostridium species can be
differentiated by their ability to neutralize oxygen free radicals. Bacillus species (like the other aerobic bacteria)
have catalase and superoxide dismutase, enzymes that can neutralize oxygen free radicals and therefore survive
in aerobic environments. Clostridium species do not have these enzymes and are therefore obligate anaerobic
microorganisms.

What other medically important species are included in this genera?


While there are numerous Bacillus (and related genera) species, the most important medically is Bacillus cereus. This
organism is far less virulent compared to Bacillus anthracis. B cereus causes food poisoning when the spores survive
the cooking process and enter the gastrointestinal system. The most common associated food is warm rice. Nausea
and vomiting is seen within the first 1-5 hours and caused by a preformed toxin. Diarrhea is seen 8 to 20 hours
later. B cereus can also rarely cause systemic infections.

What is the other primary manifestation of this infection?


B anthracis also causes pulmonary anthrax, or wool-sorters’ disease. In this condition, inhaled anthrax spores reach
the alveoli, where they are taken up by macrophages and carried to mediastinal lymph nodes. This can result in
mediastinal hemorrhage and a bloody pleural effusion. X-ray of the chest reveals a widened mediastinum. The
organism makes its way quickly to the bloodstream to cause a frequently fatal systemic illness.

How is this infection treated?


The treatment of anthrax generally involves a fluoroquinolone, such as ciprofloxacin.
44 CHAPTER 3 Microbiology

CASE 5
A 49-year-old woman from Indonesia presents with diffuse, crampy abdominal pain that has persisted for the
previous 4 days. She has had no bowel movements since the pain started and has noticed a weight loss of about
4.5 kg (10 lb) over the past month. She had a screening colonoscopy 3 months before presentation, which was
negative. CT of the abdomen reveals an inflamed gallbladder and an irregular mass in the second portion of the
duodenum. Stool sample reveals rough-surfaced eggs. Complete blood count and liver function test results are as
follows:
White blood cell (WBC) count: 14,000/mm3 Alanine transaminase (ALT): 27 IU/L
Platelet count: 250,000/mm3 Alkaline phosphatase: 210 IU/L
Albumin: 3.2 g/dL Bilirubin, total: 4.0 mg/dL
Aspartate transaminase (AST): 29 IU/L Bilirubin, direct: 3.7 mg/dL

What is the most likely diagnosis?


Ascariasis, caused by Ascaris lumbricoides, a nematode (roundworm) found in the southern United States and
tropical climates. Ascariasis is the most common helminthic infection worldwide. Eosinophilia is a classic finding
in helminth infections and is due to the increased need for eosinophilic release of major basic protein, a key
antiparasitic factor.

What tests can be used to confirm the diagnosis?


Analysis of a stool sample shows eggs with a knobby, rough surface (see Figure 3-5).

FIGURE 3-5. Ascariasis egg on stool ova and parasite


test. (Reproduced courtesy of the Centers for Disease
Control and Prevention.)

What are the treatments for this condition?


As with many nematode infections, mebendazole or albendazole is the drug of choice. The benzimidazoles work
by disrupting helminthic microtubule synthesis, which weakens cell structure.

What are some other Nematodes (roundworms)?


Enterobius vermicularis (pinworm) is a roundworm that is transmitted by fecal-oral route. This roundworm infects
the intestines and causes anal pruritus. Classically, this organism is seen in pediatric patients with itching of the
bottom. This is diagnosed by visualizing the ova with a “tape test.” Mebendazole is often used to treat the disease
and should be given to the whole family.
Trichinella spiralis is another nematode that is transmitted by fecal-oral route but can also be transmitted by eating
undercooked pork. This nematode can infect the intestine, and then the larval forms infect the body through the
bloodstream. From the bloodstream, the worm travels to the muscle, where it causes inflammation of the muscle.
The disease is called “trichinosis” and is characterized by fever, nausea, vomiting, and myalgia. Treatment is with
mebendazole or albendazole plus prednisone.

03_USMLE-STEP1_ch03_039-088.indd 44 9/17/18 3:30 PM


Microbiology CHAPTER 3 45

CASE 6
A 54-year-old man with a history of tobacco
use and chronic obstructive pulmonary
disease (COPD) presents to the ED because
of severe shortness of breath. The patient
has been steroid-dependent for his COPD
for approximately a year but developed
hemoptysis 1 week ago. He was started
on empiric antibiotics and underwent
bronchoalveolar lavage, which revealed
the presence of fungal elements with
45-degree branching septate hyphae (see
Figure 3-6).

FIGURE 3-6. (Reproduced with permission from USMLE-Rx.com.)

What is the most likely diagnosis?


The hemoptysis and pulmonary symptoms, along with the acute-angle branching septate hyphae found on
lavage (and visible in Figure 3-6), indicate Aspergillus infection. Aspergillus is an opportunistic infection, secondary
to the patient’s chronic immunosuppression from corticosteroid use. Mucormycosis most commonly presents
with sinusitis, black nasal discharge, and craniofacial invasion, but may also cause pulmonary disease, but lavage
findings would indicate 90-degree branching nonseptate hyphae.

What patient population is at risk for this condition?


This infection is most common in patients with neutropenia, and those with type 1 diabetes mellitus ketoacidosis
and other forms of acidosis. Neutropenia may be secondary to hematologic malignancy, chemotherapy,
immunosuppressive therapy, solid organ transplant, or HIV infection.

What is x-ray of the chest likely to show?


Aspergillus can appear as a “fungus ball” or a circular mobile lung mass within preexisting cavitary lesions in
the lungs. This form of Aspergillus infection is called an aspergilloma and tends to be seen in patients with
pre-existing pulmonary disease. In neutropenic patients, an x-ray of the chest is likely to show dense infiltrates,
often with wedge-shape infarcts.

The patient is treated with voriconazole. What is the drug’s mechanism of action?
Voriconazole is one of the azole antifungal agents. Azoles primarily work by inhibiting the lanosterol 14-alpha-
demethylase enzyme. This enzyme converts lanosterol to ergosterol, which is an important component in the
synthesis of fungi’s cellular membrane. The lack of ergosterol increases the permeability of the cell membrane,
causing lysis of fungi.

If, instead of COPD, this patient had a history of severe asthma, to what type of fungal infection would he be
most susceptible?
Allergic bronchopulmonary aspergillosis is an IgE-mediated hypersensitivity reaction to Aspergillus spores.
The hyperactive inflammatory response in the airways of asthmatics predisposes them to bronchospasm and
pneumonitis in response to an otherwise benign inoculation of Aspergillus spores.
46 CHAPTER 3 Microbiology

CASE 7
A 41-year-old man presents to the ED complaining of the sudden onset of weakness, nausea, vomiting, and blurred
vision. On physical examination, he has fixed, dilated pupils and a decreased gag reflex. When asked, he admits that
he often eats food that he has canned himself. The patient is admitted to the hospital for further monitoring.

What is the most likely diagnosis?


Botulism, resulting from ingestion of the botulinum toxin made by the gram-positive, spore-forming bacteria
Clostridium botulinum.

How does the toxin work?


Acetylcholine is normally released by motor neurons into the neuromuscular junction, where it binds to
muscarinic receptors on the motor endplate of the muscle fiber. This binding depolarizes the membrane and
subsequently contracts the muscle. Botulinum toxin acts on the presynaptic neuron by cleaving the SNARE
proteins within the neuron. As a result, acetylcholine cannot be released from inside the neuron and into the
neuromuscular junction. The result is a flaccid paralysis, or inability to contract. The binding is irreversible, and it
takes approximately 6 months for new synapses to form.

What is the typical course in adult patients with this condition?


Ingestion of the botulinum toxin in food usually causes symptoms within 12–36 hours. The first symptoms
are gastrointestinal distress (eg, cramps and nausea), due to enteric nervous system dysfunction, followed by
neurologic symptoms. The first nerves affected are the cranial nerves, causing blurred vision, decreased eye
movements, and a decreased gag reflex. The paralysis is symmetric and descending. Autonomic nerves can also
be affected, resulting in ileus, urinary retention, and orthostatic hypotension. Respiratory muscles can also be
affected, necessitating ventilator support.

What is the differential diagnosis for this presentation?


The major differential includes Guillain-Barré syndrome, myasthenia gravis, and Lambert-Eaton syndrome.
Unlike botulism, the paralysis seen in Guillain-Barré syndrome is due to a postinfectious demyelination of
alpha motor neurons and is ascending. The most common infection leading to Guillain-Barré syndrome is
Campylobacter jejuni. Myasthenia gravis is an autoimmune condition caused by antibodies created against the
muscarinic acetylcholine receptor. Patients with this condition have muscle weakness after prolonged muscle
use, classically at the end of the day, and typically have more gradual onset of illness. Lambert-Eaton syndrome
is a paraneoplastic anti–calcium channel antibody syndrome that causes muscle weakness that improves with
prolonged muscle use.

How can this toxicity be acquired?


In adults, it is acquired most commonly from ingestion of preformed toxin in contaminated canned foods (usually
home canned). In infants, ingestion of bacterial spores found in honey can result in toxicity referred to as “floppy
baby syndrome” due to intestinal overgrowth with C botulism.

What other organism in this genus causes paralysis?


Clostridia tetani is a gram-positive organism that creates an exotoxin that can cause paralysis. However, this
paralysis is opposite to Clostridia botulinum. The paralysis is spastic and classically presents with trismus and risus
sardonicus. The toxin cleaves the SNARE protein for neurotransmitter just like botulism toxin does. This toxin blocks
GABA and glycine instead of acetylcholine. The TdaP vaccine is given to prevent this disease. Treatment includes
diazepam, which is a muscle relaxer, and with antitoxin.

03_USMLE-STEP1_ch03_039-088.indd 46 9/17/18 3:30 PM


Microbiology CHAPTER 3 47

CASE 8
A 57-year-old patient with diabetes presents to her physician with a white, flaky, adherent substance on the skin
under her breasts. Her last Hgb A1c was 10.2, which was done 3 months ago. Patient was prescribed home insulin.
Upon questioning, the patient admits to not being compliant with her daily injections of insulin. She has had
worsening symptoms of urinary frequency and thirst. Her complaints with the skin began 2 weeks ago.

What is the most likely diagnosis?


The fungus Candida albicans can result in systemic or superficial fungal infection (candidiasis). Skinfold infection,
vaginitis (yeast infection), and oral thrush are common manifestations of local candidiasis and present as a white,
flaky, cheesy exudate on the affected surface.

Where is the microorganism that causes this condition normally found?


C albicans is part of the normal microbiota of mucous membranes of the gastrointestinal tract, respiratory tract,
and women’s genital tract. Overgrowth, due to alteration of normal microbiota in women taking antibiotic
therapy or patients who are immunocompromised, causes candidiasis.

What laboratory tests can help confirm the diagnosis?


A potassium hydroxide preparation (KOH mount) is
used for skin or tissue scrapings. Pseudohyphae and
budding yeast (see Figure 3-7) are observed in the tissues.
Pseudohyphae are seen in culture at 20°C (68°F). Germ
tube formation in the laboratory distinguishes C albicans
and other Candida species. For (rare) systemic disease
(eg, invasive candidiasis found primarily in neutropenic
patients), blood cultures are positive for the fungus.

What populations other than immunocompromised


patients are at risk for serious forms of this condition?
Intravenous drug users are at higher risk for candidal
endocarditis. The fungi enter the skin by the needle
and affect the heart. Hospitalized patients with medical
devices (intravascular and urinary catheters) who
receive broad-spectrum antibiotic therapy are at risk for
bloodstream and urinary tract Candida infections. Patients
that have had recent antibiotic treatment may have
FIGURE 3-7. Candida albicans. (Reproduced courtesy of
vaginal discharge that is due to candida. This is because the Centers for Disease Control and Prevention/Dr. Gordon
the normal microbiota of the vaginal tract is disrupted due Roberstad.)
to the antibiotics.

What are the treatments for this condition?


Oral fluconazole or nystatin is used for superficial infections. Fluconazole, an echinocandin, or amphotericin B can
be used for systemic infections.

03_USMLE-STEP1_ch03_039-088.indd 47 9/17/18 3:30 PM


48 CHAPTER 3 Microbiology

CASE 9
A 49-year-old woman who recently immigrated to the United States from Nicaragua presents to the clinic with
difficulty swallowing, constipation, and abdominal pain. She says her last bowel movement was more than a week
ago. Physical examination reveals tachycardia and a distended abdomen. An electrocardiogram (ECG) shows Mobitz
type I heart block.

What is the most likely diagnosis?


Chagas disease, or American trypanosomiasis, caused by the protozoan Trypanosoma cruzi.

What is the vector of the responsible protozoan?


The vector is the reduviid bug, also known as the “kissing bug”
because the bite is painless (see Figure 3-8). It can also be
acquired orally via food or juice contaminated with the vector.

What is the classic sign associated with the acute form of


this condition?
The Romaña sign is painless, unilateral periorbital edema and
conjunctivitis that results from acute Chagas disease. This sign
is typical but not sensitive for acute T cruzi infection.

Where in the world is this condition commonly found? FIGURE 3-8. Triatoma infestans (Reduviidae family,
Chagas disease is commonly found in the southern United the “kissing bug, ” “assassin bug, ” or “cone-nose
bug”), a vector for Chagas disease. (Reproduced cour-
States, Mexico, and Central and South America (ie, only in the tesy of the Centers for Disease Control and Prevention/
Western hemisphere). Rarely, it is transmitted in the southern Donated by the World Health Organization, Geneva,
United States. Switzerland.)

What is the pathophysiology of this condition?


This woman is experiencing chronic Chagas disease, which is most often characterized by heart block, ventricular
tachycardia, and dilated cardiomyopathy. Dilatation of the esophagus and colon (megaesophagus and
megacolon) can cause difficulty swallowing and constipation. The acute phase of the disease can be characterized
by a swollen red area called a chagoma at the parasite’s site of entry into the host, rarely accompanied by fever and
acute systemic symptoms, myocarditis, or meningoencephalitis. In endemic areas, the acute phase is seen more
frequently in children.

What is the treatment for this condition?


Nifurtimox and benznidazole are used to treat acute cases. In chronic illness, the data suggest that treatment of
patients with cardiac disease may slow progression prior to development of congestive heart failure. Antiparasitic
therapy does not alter established pathology in the cardiovascular or GI systems. For chronic heart disease,
supportive measures for congestive heart failure, antiarrhythmics to prevent recurrent ventricular tachycardia, and
pacemaker implantation for heart block are used. For gastrointestinal disease, dilation of the esophageal sphincter,
changes in diet, the use of laxatives and/or enemas, and in some cases eventual resection of the megacolon are
used.

What other disease is caused by the protozoan genus that causes this condition?
The protozoa Trypanosoma gambiense and Trypanosoma rhodesiense cause African sleeping sickness. This illness is
characterized by lymphadenopathy, recurrent fevers due to antigenic variation, somnolence, and eventually coma.
It is transmitted by the tsetse fly, whose bite is painful.

03_USMLE-STEP1_ch03_039-088.indd 48 9/17/18 3:30 PM


Microbiology CHAPTER 3 49

CASE 10
A 24-year-old American man is traveling in rural India during the monsoon season. Over the course of a few hours,
he develops severe watery diarrhea. In the next 30 hours, he has approximately one episode per hour of liquid stools
that appear clear with small white flecks of mucus. He also has occasional episodes of vomiting. He quickly becomes
lethargic and generally ill with crampy abdominal pain but is afebrile. He rehydrates himself aggressively during the
illness, and the symptoms resolve within approximately 48 hours.

What is the most likely diagnosis? FIGURE 3-9. Gram-


This patient has cholera, a potentially fatal dehydrating stained specimen of
illness caused by Vibrio cholerae. This microorganism is Vibrio cholerae, re-
vealing the presence
a gram-negative, curved, motile, polar flagellated rod of single polar fla-
(see Figure 3-9). Symptomatic cholera usually manifests gellum. (Reproduced
in epidemics, and it is endemic to developing regions in courtesy of Centers for
Africa, Asia, South and Latin America, and recently the Disease Control and
Middle East. Prevention.)

What is the primary differential diagnosis?


Watery diarrhea induced during travel within a foreign country makes the noninvasive enterotoxigenic Escherichia
coli (ETEC) infection the primary differential diagnosis (“traveler’s diarrhea”). However, ETEC diarrhea generally is
not as voluminous as the diarrhea induced by cholera and is not associated with white mucus flecks.

How does the microorganism involved in this condition exert its effect on the gastrointestinal tract?
V cholerae is ingested through fecally contaminated water. It secretes an exotoxin (cholera toxin) that binds to
the surface of intestinal epithelium. This toxin ADP-ribosylates adenylyl cyclase, thus increasing levels of cyclic
adenosine monophosphate (cAMP) within the intestinal mucosa (see Figure 3-10). E coli produces a toxin that
has a similar effect on the enterocyte. This causes increased chloride secretion and decreased sodium absorption,
leading to a massive secretory loss of fluids and electrolytes.

ADP-R Enterocyte

cAMP Protein Water and


Active kinase electrolytes
subunit
Cholera
toxin Binding Gs Adenylate
subunit
Water, chloride,
protein cyclase Gut lumen bicarbonate

Diarrhea

FIGURE 3-10. cAMP induction by exotoxins of Vibrio cholerae and Escherichia coli. (Reproduced
with permission from USMLE-Rx.com.)

What are the clinical manifestations of this condition?


The hallmark of cholera is rice-water stools, so described because the small white flecks of mucus resemble grains
of rice in the voluminous, watery fluid. The onset of this diarrhea typically occurs 1–3 days after infection. Vomiting
and abdominal cramping is common, but fever is rare because V cholerae itself is noninvasive and thus remains in
the gastrointestinal tract. Many infections are asymptomatic, but severe cholera can lead to extreme dehydration
that can cause death within hours due to the excretion of electrolytes leading to renal failure, arrhythmias from
hypokalemia, metabolic acidosis from bicarbonate loss, and hypovolemic shock.

What is the treatment for this condition?


Oral rehydration solution (ORS), which has reduced mortality rates from 50% to < 1%. A typical preparation of
ORS contains glucose, potassium chloride, sodium chloride, and sodium bicarbonate. Glucose facilitates sodium
absorption from the gut, which allows for the concurrent absorption of water. Antibiotics are of limited use in
stopping the diarrhea, although early use of doxycycline can reduce the volume of diarrhea and decrease the
duration of bacteria excretion by 1 day.
50 CHAPTER 3 Microbiology

CASE 11
A 3-year-old boy is brought to his pediatrician with a fever, tachypnea, and a cough productive of rusty sputum.
He has a history of recurrent lung and skin infections. He has had several fungal infections of his skin, as well as a
staphylococcal abscess that formed where he scraped his arm. An x-ray of the chest shows a normal thymic shadow
but some hilar lymphadenopathy. Further questioning of the parents reveals a maternal male cousin who died at
5 years of age from severe pneumonia and a maternal uncle who has severe pulmonary disease and two surgeries
for intracranial fungal infections.

What is the most likely diagnosis?


The most likely diagnosis is chronic granulomatous disease (CGD), an X-linked inherited immunodeficiency
syndrome. This patient is predisposed to bacterial and fungal infections. The normal thymic shadow suggests
normal T-cell maturation, which effectively rules out a diagnosis of severe combined immunodeficiency or
DiGeorge syndrome. The strong family history of male involvement on the maternal side indicates an X-linked
hereditary condition and suggests CGD as the likely diagnosis in this patient.

Infections with which organisms could be particularly severe and problematic in this patient?
Patients with CGD are at risk for serious infections with catalase-positive bacteria, including Staphylococcus aureus,
Aspergillus species, and Burkholderia cepacia.

Why are patients with this condition especially susceptible to catalase-positive organisms?
Reduced nicotinamide adenine dinucleotide phosphate (NADPH) oxidase, which is required for production of
reactive oxygen species, is deficient in patients with CGD. The radicals are used by neutrophils during the oxidative
burst to kill engulfed organisms. In the absence of the killing ability of the deficient neutrophils, the primary
method of host immunity is containment of the offending organism, leading to numerous granulomas.

What laboratory test can confirm this diagnosis?


The nitroblue tetrazolium test can detect the presence of a respiratory burst in neutrophils. In normal individuals
the test is positive, but in patients with CGD the test is negative because the superoxide free radical is not
produced.
A more modern test for CGD is the dihydrorhodamine 123 test, where dihydrorodamine (DHR) is ingested by the
neutrophil. DHR is then oxidized by the cell and produces a product that can be seen when run through the flow
cytometry.

What medical treatments are available for this condition?


Infections must be treated aggressively with appropriate antimicrobials. Trimethoprim-sulfamethoxazole can be
used as long-term prophylaxis. In addition, interferon-α, an immunomodulator, is used in patients with CGD.

What therapy or procedure provides a definitive cure for this patient?


Bone marrow transplantation provides a source of functional neutrophils with the ability to create oxygen free
radicals to effectively kill organisms engulfed by phagocytosis.

What are some other immunodeficiencies?


Severe combined immunodeficiency (SCID) is a type of X-linked disease if there is a defective IL-2R γ chain. SCID
can be autosomal recessive where there is an adenosine deaminase deficiency. This condition presents with
opportune infections, fungal infections, and protozoal infections. Thrush and chronic diarrhea can also be seen. A
thymic shadow is absent, unlike in the patient in this case. Treatment is a bone marrow transplant.
Wiskott-Aldrich syndrome is an X-linked disease; there is a mutation in the WAS gene where T cells are unable to
reorganize actin cytoskeleton.

03_USMLE-STEP1_ch03_039-088.indd 50 9/17/18 3:30 PM


Microbiology CHAPTER 3 51

CASE 12
An 85-year-old man is hospitalized for community-acquired pneumonia. He is treated with moxifloxacin and over the
next week he feels that he is slowly recovering. On hospital day 10, he develops a low-grade fever, watery diarrhea,
and lower abdominal pain.

What is the most likely diagnosis?


Antibiotic-associated colitis or pseudomembranous colitis caused by Clostridium difficile superinfection
(or overgrowth). C difficile is a gram-positive, spore-forming anaerobe. It should be noted that most
antibiotic-associated diarrhea (without fever) is osmotic, resulting from decreased carbohydrate digestion
secondary to a loss of gut microbiota. However, C difficile infection will present with fever and leukocytosis, and
can be a very serious complication of prolonged antibiotic use.

What are the manifestations of this condition?


Approximately 20% of hospitalized patients are
asymptomatically colonized with Clostridium difficile. Patients
with disease usually present with a low-grade fever, watery
diarrhea, lower abdominal pain, leukocytosis, and a recent
history (within 10 weeks) of antibiotic use. In severe cases,
peritonitis (inflammation of the peritoneum) can result
from microperforation of the diseased colon. These patients
present with signs of peritonitis such as rebound tenderness
and involuntary guarding. On colonoscopy, they likely
have pseudomembranes on the colon, which are raised
yellow-white plaques created by the C difficile toxins (see
Figure 3-11). Risks include ileus and toxic megacolon, which
can grossly perforate and cause death. Emergent colectomy
is indicated and can be a lifesaving procedure if performed in
a timely manner.

FIGURE 3-11. Pseudomembrane in C difficile infection. (Re-


produced with permission from Abe I, et al. Acute fulminant
pseudomembranous colitis which developed after ileostomy
closure and required emergent total colectomy: A case report. J
Med Case Rep. 2012;6:130.)

What population of patients is susceptible to this condition?


Infection is most often seen in elderly hospitalized patients. C difficile produces resistant spores, which are
commonly found on hospital objects and on the hands of health care workers. Common alcohol-based hand
sanitizers are ineffective at eliminating C difficile spores. C difficile colonizes the gastrointestinal (GI) tract (usually
the colon) after the normal gut microbiota is killed or altered by antibiotics. The antibiotics most commonly
associated with this disease are the penicillins, cephalosporins, quinolones, and clindamycin.

What is the pathophysiology of this disease?


Once it has colonized the GI tract, C difficile releases toxins (toxins A and B) that destroy intestinal epithelial cells.
A new, more virulent strain of this bacterium that produces a binary toxin in addition to toxins A and B is often
associated with the use of fluoroquinolones.

How is this condition diagnosed and treated?


Definitive diagnosis can be made with a cytotoxicity assay, an enzyme-linked immunosorbent assay for
C difficile toxin A, or polymerase chain reaction. First-line treatment is with oral metronidazole or vancomycin. Fecal
transplantation is an emerging therapy that aims to replenish the missing gut microbiota in patients with
C difficile overgrowth by introducing normal fecal bacteria from a healthy patient. Several well-controlled studies
reveal a high success rate of this procedure, and it is emerging as standard of care for recurrent or refractory
C difficile infections.

03_USMLE-STEP1_ch03_039-088.indd 51 9/17/18 3:30 PM


52 CHAPTER 3 Microbiology

CASE 13
A 5-year-old girl is brought to the clinic with a 3-month history of worsening vision and behavioral difficulty in school.
She immigrated to the United States from Guatemala 2 years ago with her mother and a younger sibling. Her mother
received no prenatal care and, through a translator, reports that the patient was delivered without complication at
home. As an infant, the girl had a “wart-like” perioral maculopapular rash and three or four recurrent right-sided ear
infections. Physical examination reveals that the girl is in the 30th percentile for weight and the 35th percentile for
height. Additional observations include fundi that are notable for nummular keratitis, prominent notching of her
upper two incisors and molars, and outward bowing of the tibia bilaterally.

What is the most likely diagnosis?


Congenital syphilis. This infection is one of the so-called TORCHeS infections (Toxoplasmosis, Other infections,
Rubella, Cytomegalovirus, Herpes simplex virus, Syphilis), the most common causes of congenital infection.

What is the causative microorganism in this condition?


Treponema pallidum.

What symptoms are commonly found in patients with this condition?


Congenital syphilis is a cause of hydrops fetalis, or stillbirth due to fluid accumulation in the fetus. If the newborn
survives, it can develop various abnormalities, including the classic facial anomalies (tooth abnormalities known as
Hutchinson incisors and mulberry molars, saddle nose, frontal bossing, and short maxilla), as well as recurrent ear
infections and interstitial keratitis, leading to vision problems.

In the newborn, what tests can help confirm the diagnosis?


Serum rapid plasma reagin (RPR) test: Umbilical cord blood may show false-positive results because of maternal
titers but remains the best screening tool for detecting syphilis infection. Serum and CSF (cerebrospinal fluid) can
be tested by the Venereal Disease Research Laboratory (VDRL) testing. VDRL, like RPR, detects anticardiolipin
antibodies that are produced by patients with syphilis. The serum VDRL is used for screening purposes. VDRL
testing from CSF samples is used to detect central nervous system involvement of the disease, known as
neurosyphilis. Other CSF findings such as pleocytosis (increased number of cells) and elevated protein levels also
suggest infection. The presence of anticardiolipin antibodies is not specific to syphilis and can return false-positive
results in patients with Epstein-Barr virus, systemic lupus erythematosus, rheumatoid arthritis, and other
autoimmune or inflammatory conditions. For that reason, confirmatory serological testing using a treponemal test
(TP-PA; various EIA/CIA methods) is recommended to confirm RPR or VDRL positive cases.

How does infection with this organism present in an adult?


Syphilis in adults is a sexually transmitted infection. Primary syphilis presents as a macule or papule that becomes
an ulceration that usually occurs on the genitals but may be in the oral cavity. Secondary syphilis occurs 4–11
weeks afterwards and involves mucous membranes and skin. A non-itchy maculopapular rash presents on the
extremities or trunk. Tertiary syphilis occurs years after initial infection. Without treatment, an infected person
still has syphilis even though there are no signs or symptoms. It remains in the body, and it may begin to damage
the internal organs, including the brain, nerves, eyes, heart, blood vessels, liver, bones, and joints. Gummas occur
anywhere on the body and are inflammatory lesions (see Figure 3-12). Symptoms may involve the central nervous
system, including general paralysis, tabes dorsalis, dementia, or Argyll Robertson pupils. Syphilis has the potential
to inflame the vasa vasorum of the aorta, which results in aortic aneurysm.

FIGURE 3-12. Tertiary syphilis. Gumma of the nose due to a


long-standing tertiary syphilitic Treponema pallidum infection. (Re-
produced courtesy of Centers for Disease Control and Prevention/J.
Pledger.)

03_USMLE-STEP1_ch03_039-088.indd 52 9/17/18 3:30 PM


Microbiology CHAPTER 3 53

CASE 14
A pathologist is performing an autopsy on a 56-year-old university
professor who suffered a rapid demise from an undiagnosed
neurologic disease. Approximately 1 year previously, the patient
presented to a psychiatrist with symptoms of psychosis. Shortly
thereafter, his symptoms advanced to include unsteadiness and
involuntary movements, and the patient ultimately became
immobile and unable to speak. A sample of brain tissue is shown
in Figure 3-13.

FIGURE 3-13. (Reproduced courtesy of the Centers for


Disease Control and Prevention/Teresa Hammett.)

What is the most likely diagnosis?


Creutzfeldt-Jakob disease (CJD) is a prion disease characterized by rapidly progressive dementia with ataxia,
myoclonus, and death within 1 year.

What are the classic brain histology findings in this condition?


On histology, prion disease presents with dramatic neuronal loss with numerous vacuoles in the gray matter
resembling a porous sponge (see Figure 3-13). This is therefore known as spongiform encephalopathy.

How does the causative agent in this condition differ from other pathogens?
Prions do not contain RNA or DNA; they are composed only of abnormally folded proteins.

How is this condition transmitted?


Disease can be transmitted by central nervous system (CNS) tissue containing prions (transmission has been
seen secondary to corneal transplants, ingestion, and implantable electrodes or other intracranial surgery with
contaminated instruments as prions are not destroyed by routine autoclaving). Prion disease can also be inherited,
but most cases are sporadic.

What other condition is associated with this type of pathogen?


Prions cause two acquired degenerative CNS diseases in humans: CJD and kuru, a slowly progressive, fatal disease
formerly found among tribes in Papua, New Guinea, who practice cannibalism, as well as several extremely rare
heritable prion diseases.

How does the structure of normal prions differ from that of pathologic prions?
Normal prions have α-helix conformations, whereas pathologic prions are composed of an abnormal isoform
of β-pleated sheets. The new structure renders them undegradable, and buildup leads to neuronal toxicity.
Further, the abnormal prions cause normal prions to change conformation into β-pleated sheets, leading to the
contagiousness of the disease.

03_USMLE-STEP1_ch03_039-088.indd 53 9/17/18 3:30 PM


54 CHAPTER 3 Microbiology

CASE 15
A 32-year-old man with a medical history of HIV presents to the ED with complaints of worsening headache, fever,
and a stiff neck. Lumbar puncture is performed, and analysis reveals an elevated opening pressure, increased protein,
and decreased glucose level. Gram staining of the spinal fluid reveals budding yeast; an antigen test is positive.

What is the most likely diagnosis?


Cryptococcal meningitis is the most common fungal cause of meningitis and is prevalent among patients with AIDS.

What microorganism causes this disease?


Cryptococcus neoformans is a heavily encapsulated yeast. It is found only as a yeast; it is not a dimorphic
microorganism.

How does this microorganism cause illness?


C neoformans is found in pigeon droppings and in soil. When inhaled, the yeast causes a local infection in the
lung; this infection is commonly asymptomatic but can result in pneumonia. Hematogenous spread to the central
nervous system (CNS) results in meningitis. As in other common causes of meningitis, the capsule is thought to be
an important virulence factor for gaining access into the CNS.

What laboratory tests can help confirm the diagnosis?


Tests for the polysaccharide capsular antigen are the diagnostic
procedures of choice; a lateral-flow immunoassay is now the test of choice.
The microorganism can also be cultured on Sabouraud agar. India ink
stains the heavy polysaccharide capsule and reveals budding yeast (see
Figure 3-14) but is insensitive and no longer standard of care.

FIGURE 3-14. India ink stain of Cryptococcus


neoformans. (Reproduced courtesy of the
Centers for Disease Control and Prevention/
Dr. Leanor Haley.)

CD4+ cell counts are typically at or below what level when infection
with this microorganism occurs?
C neoformans usually infects severely lymphopenic patients with CD4+ cell counts < 100 cells/mm3.

What is the treatment for this condition?


Patients who are not immunocompromised can be treated sufficiently with amphotericin B and flucytosine for the
meningitis. Patients with AIDS require long-term suppression with fluconazole after induction with amphotericin B
and flucytosine. In these patients, long-term suppression may be stopped if the patient responds to highly active
antiretroviral therapy (HAART) and has repeated measurements demonstrating high CD4+ counts.

What cerebrospinal fluid findings are expected in this condition?


Like viral meningitis, fungal meningitis has an elevated WBC count with a lymphocytic predominance. However, all
other laboratory results mimic those of bacterial meningitis: increased opening pressure, increased protein, and
decreased glucose in cerebrospinal fluid.

03_USMLE-STEP1_ch03_039-088.indd 54 9/17/18 3:31 PM


Microbiology CHAPTER 3 55

CASE 16
A 42-year-old man who immigrated from Mexico 5 years ago
presented to the ED with new-onset seizures. CT of the head
(see Figure 3-15) reveals several calcified regions and cystic
masses but no solid mass lesion or evidence of bleeding. A
complete blood count reveals mild anemia and a WBC count
of 78,000/mm3 with 12% eosinophils.

FIGURE 3-15. (Reproduced with permission from Rooper AH,


et al. Adams & Victor’s Principles of Neurology, 9th ed. New York:
McGraw-Hill, 2009: Figure 32-8.)

What is the most likely diagnosis?


Cysticercosis, caused by Taenia solium (pork tapeworm), is a cestode (tapeworm) infection. When the tapeworm
invades the brain, it forms small nonpurulent abscesses. The disease is known as neurocysticercosis and is
responsible for the majority of adult-onset seizures in developing nations.

How does the organism involved in this condition cause illness?


Ingestion of undercooked pork introduces larvae from pig muscle into the human gastrointestinal (GI) system.
These larvae mature in the small intestine into adult tapeworms. Eggs from the adult worms are released into the
feces. Ingestion of these eggs via the fecal-oral route from a tape worm–infected human host allows eggs to enter
the GI tract, where they develop into larvae. The larvae then penetrate the intestinal wall and migrate into the
blood and tissues, encysting in various organs, including the brain, as seen in Figure 3-15.

What signs and symptoms are associated with this condition?


Infection with the tapeworm is usually asymptomatic or can cause mild abdominal discomfort. Cysticercosis can
be found anywhere in the body, unfortunately, including the brain and eye, leading to seizures, focal neurological
symptoms, and blindness.

What tests can help confirm the diagnosis?


Intestinal infection is revealed by eggs in stool. Cysticerci can be observed on imaging of the head when
cysticercosis occurs in the brain. X-ray may reveal calcified cysticerci in other parts of the body, such as muscle.
Serological testing for T solium antibody can complement imaging in diagnosis.

What are the treatments for this condition?


Praziquantel is used for cysticercosis, and albendazole with steroids is used for neurocysticercosis. In addition,
steroids and anticonvulsants may be given for decreasing complications of neurocysticercosis. Asymptomatic
patients with only calcified cysts are rarely treated.

What are the other cestodes?


Diphyllobothrium latum is a cestode transmitted by ingestion from freshwater fish that rarely causes vitamin
B12-deficient macrocytic anemia. Echinococcus granulosus eggs are ingested from dog feces and can cause liver
cysts.

03_USMLE-STEP1_ch03_039-088.indd 55 9/17/18 3:31 PM


56 CHAPTER 3 Microbiology

CASE 17
A 48-year-old woman who recently underwent a lung transplant presents to the ED with a fever and slight
cough. She states that she had some difficulty breathing; however, she has not had chest pain. She is currently
on immunosuppressive therapy for the transplant. An x-ray of the chest is taken and shows bilateral interstitial
infiltrates. The patient undergoes a bronchoscopic biopsy (see Figure 3-16).

FIGURE 3-16. (Reproduced with permission from USMLE-Rx.com.)

What is the most likely diagnosis?


Pneumonia due to cytomegalovirus (CMV) infection. CMV is a double-stranded linear virus in the family
Herpesviridae. Infected cells have intranuclear inclusions, and on histology have an “owl’s-eye” appearance, as seen
in the center of Figure 3-16.

What is the presentation of this condition?


In most people, CMV infection is asymptomatic. In those with symptoms, it usually presents with a mononucleosis-
like syndrome, which includes pharyngitis, cervical lymphadenopathy, fever, lethargy, and, less often,
splenomegaly. Unlike the mononucleosis syndrome seen with Epstein-Barr virus, the monospot test is negative.
CMV can be transmitted by direct contact, blood transfusions, organ transplantation, breast milk, sexual contact,
and vertically (ie, mother to fetus). It is one of the TORCHeS infections (Toxoplasmosis, Other infections, Rubella,
Cytomegalovirus, Herpes simplex virus, Syphilis).

What populations are at risk for complications of this condition?


The populations most at risk are those with decreased cellular immunity, such as patients with AIDS and organ
transplants (especially bone marrow and lung transplants). The main complication in the transplantation
population is CMV pneumonia. The main complication in the AIDS population is CMV retinitis, which usually
presents when the CD4+ cell count is < 50 cells/mm3. In both populations, prophylactic ganciclovir can be given.

How does this condition present in patients infected congenitally?


In patients congenitally infected with CMV, the complications include petechiae, jaundice, microcephaly,
microsomia, retinitis, neurologic abnormalities, and deafness. At-risk fetuses are those whose mothers have
a primary infection, which is seen with positive IgM levels (the IgG levels could be low or high). Mothers with
negative IgM and positive IgG likely have a prior infection and are unlikely to transmit from mother to fetus.

What is the treatment for this condition?


Although most immunocompetent patients do not need treatment, the treatment is ganciclovir, a nucleoside
analog. This drug requires activation by viral kinase, which phosphorylates the drug and allows it to inhibit CMV
DNA polymerase. Acyclovir is not effective against CMV.

What are the most common adverse effects of the treatment for this condition?
Ganciclovir is more toxic than acyclovir. Adverse effects include leukopenia, neutropenia, thrombocytopenia, and
renal toxicity.
Microbiology CHAPTER 3 57

CASE 18
A 36-year-old woman who has recently returned from Southeast Asia presents to her physician with sudden-onset
fever, severe muscle pain in her back and extremities, and recent joint pain in her knees. Examination reveals an
erythematous macular rash that covers her face and body.

What is the most likely diagnosis?


This woman is likely experiencing dengue fever, also known as “breakbone fever” because of the severe joint and
muscle pain associated with it.

What is the vector for this condition?


The vector is the Aedes aegypti mosquito. These mosquitoes are diurnal and live near cities. They are most
commonly found in pools of stagnant water. This distinguishes them from malaria-carrying Anopheles mosquitoes,
which are nocturnal and are typically less populous near urban areas. Once a rare disease in the Western
hemisphere, dengue fever began to reappear in the 1970s, when bans on pesticides such as DDT allowed these
mosquitoes to thrive. The same vector can also carry yellow fever and chikungunya.

Which microorganism causes this condition?


Dengue fever is a disease caused by a positive, single-stranded RNA virus of the Flaviviridae family. This family also
includes St. Louis encephalitis virus, Japanese encephalitis virus, hepatitis C virus, and West Nile virus.

How does this condition differ from yellow fever?


Yellow fever virus is also a member of the Flaviviridae family and has a similar endemic region and transmission
as dengue fever virus. However, yellow fever presents with high fever, black vomit, jaundice, and acute hepatic
necrosis and is not associated with severe joint and muscle pain.

After recovering from this condition, will the patient be immune to it in the future?
The dengue fever virus has four serotypes. The patient will develop lasting immunity to the serotype of the virus
with which she was infected but not to the remaining three serotypes. This means that she could contract dengue
fever four times in all.

Infection with a different serotype of this virus poses what potential complications?
The most serious complications of dengue fever are dengue hemorrhagic fever (DHF) and dengue shock syndrome
(DSS), both of which can be fatal. These conditions are characterized by bleeding (often from the gastrointestinal
tract or from mucosa); petechiae, ecchymoses, or purpura; thrombocytopenia; fluid leakage (manifested as pleural
effusions, ascites, or hemoconcentration); and shock. Such complications most frequently occur in patients who
have already been infected with another serotype of the virus. One theory underlying this phenomenon, termed
antibody-dependent enhancement, proposes that antibodies from previous infections permit increased viral
replication upon reinfection with a different serotype. This has also hindered the development of a vaccine since the
vaccine must provide adequate protection against all four serotypes or it could put the patient at risk for DHF/DSS.

03_USMLE-STEP1_ch03_039-088.indd 57 9/17/18 3:31 PM


58 CHAPTER 3 Microbiology

CASE 19
A 7-year-old girl who recently immigrated to the
United States from Africa is brought to her primary
care physician because of a sore throat and fever of
38.3°C (101°F). Physical examination reveals a grayish
membrane covering her pharynx (see Figure 3-17) as
well as cervical lymphadenopathy.

FIGURE 3-17. (Reproduced courtesy of the Centers for Disease


Control and Prevention.)

What is the most likely diagnosis?


Diphtheria caused by the toxin-producing, gram-positive Corynebacterium diphtheriae. The pathognomonic
findings are the gray pharyngeal pseudomembranes on physical exam, as seen in Figure 3-17.

How does the microorganism involved in this condition cause this presentation?
Exotoxin A is an enzyme that blocks protein synthesis by inactivating elongation factor EF-2 by ribosylating
adenosine phosphate. This results in decreased mRNA translation and protein synthesis. (Pseudomonas toxin
has a similar mechanism.)

What growth media are used to identify the microorganism involved in


this condition?
Potassium tellurite agar and Loeffler coagulated blood serum media are
used to isolate this microorganism. C diphtheriae is a gram-positive rod (see
Figure 3-18). In culture, it often appears in clumps described as “Chinese
characters.”

FIGURE 3-18. Corynebacterium diphtheriae.


(Reproduced courtesy of the Centers for Dis-
ease Control and Prevention.)

Which vaccine would have prevented this child’s condition?


The inactivated form, or toxoid, is a component of the Diphtheria, Tetanus, and acellular Pertussis (DTaP) vaccine.
Children in the United States are required to have the DTaP vaccine by the age of 15–18 months, with a booster
shot given between 4 and 6 years of age. Recent immigrants, especially children, frequently do not have up-to-
date vaccinations.

What is the treatment for this condition?


Careful airway management to prevent respiratory compromise is essential. Antitoxin can inactivate circulating
toxin that has not yet reached its target tissue. Penicillin or erythromycin can be given to prevent further bacterial
growth and exotoxin release, thus making the patient noncontagious. The patient should receive cardiac
monitoring with ECG and telemetry to monitor for myocarditis. The patient likewise needs treatment for heart
failure or arrhythmia, and monitoring of neurologic function for motor deficits.

03_USMLE-STEP1_ch03_039-088.indd 58 9/17/18 3:31 PM


Microbiology CHAPTER 3 59

CASE 20
A 32-year-old man who recently immigrated from Cambodia
presents to his physician with extreme swelling of his legs
(see Figure 3-19) and scrotum. The skin associated with the
swollen areas is thick and scaly. The patient admits to an
episode of fever associated with enlarged inguinal lymph
nodes some time ago, but he did not think much of it.

FIGURE 3-19. (Reproduced courtesy of Centers for Disease


Control and Prevention.)

What is the most likely diagnosis?


Elephantiasis is caused by the nematode (roundworm) Wuchereria bancrofti and (in Asia) Brugi malayi and timori.

How does the organism involved in this condition cause illness?


The organism is transmitted by the bite of a female mosquito. Larvae are released into the bloodstream and travel
to the lymphatics of the lower extremities and genitals, where they mature. Approximately 1 year later, adult
worms, which reside in lymph nodes, trigger an inflammatory response.

What signs and symptoms are associated with this condition?


Inflammation resulting from the presence of adult worms causes fever and swelling of lymph nodes. Repeated
infections cause repeated bouts of inflammation, resulting in fibrosis around the dead adult worms in the lymph
nodes. This fibrosis can obstruct lymphatic drainage and lead to scaly skin and edema (as seen in Figure 3-19).

What test can help confirm the diagnosis?


Blood smears reveal larvae (microfilariae). Because larvae usually emerge at night, drawing blood in the evening
is preferred. Consult the CDC Yellow Book for details on filarial species present in different countries and the best
time to collect specimens.

What is the treatment for this condition?


Diethylcarbamazine is effective in killing the larvae but is not as effective against the adult worms. Adding
doxycycline improves activity against the adults and may decrease clinical pathology.

03_USMLE-STEP1_ch03_039-088.indd 59 9/17/18 3:31 PM


60 CHAPTER 3 Microbiology

CASE 21
A previously healthy 24-year-old man visits his physician
complaining of significant weight loss, flatulence, and
foul-smelling stools. He reports feeling fatigued since his
return from Peru 3 months previously and has suffered
abdominal cramping and intermittent loose, nonbloody
stools since then. The patient’s stool ova and parasite studies
demonstrated characteristic trophozoites on two occasions
(see Figure 3-20). He was prescribed a course of drug therapy
and warned that consumption of alcohol during treatment
could lead to nausea and vomiting.

FIGURE 3-20. (Reproduced courtesy of the Centers for Dis-


ease Control and Prevention/Dr. Mae Melvin.)

What is the most likely diagnosis?


Giardiasis due to Giardia lamblia infection. Giardia appears as both a flagellated, motile, binucleated trophozoite
(as seen in Figure 3-20) and as a round Tetranucleate cyst. Flatulence, foul-smelling stools, and chronic watery
diarrhea in a patient with a recent travel history or exposure to well water is characteristic of Giardia infection. The
ova and parasite test that reveals “smiley face” trophozoites is diagnostic.

What is the differential diagnosis for this condition?


Entamoeba histolytica can cause a similar spectrum of symptoms but might present with bloody diarrhea instead
of watery diarrhea. Both Cryptosporidium and Cyclospora infections can produce similar symptoms. Infections with
enterotoxigenic Escherichia coli, Vibrio cholerae, and Campylobacter jejuni can also cause watery diarrhea, but the
onset in these cases is generally acute and tends to resolve within a few days. Inflammatory bowel disease and
gluten enteropathy can also be associated with chronic diarrheal illnesses, though the travel history suggests an
infectious etiology.

What is the treatment for this condition?


Metronidazole is the agent used to treat giardiasis. Concurrent alcohol use with metronidazole produces a
disulfiram-like effect (disulfiram is occasionally prescribed to discourage alcohol consumption in situations of
alcohol addiction). Metronidazole interferes with the action of aldehyde dehydrogenase in ethanol metabolism,
which increases serum acetaldehyde levels and thus leads to nausea, vomiting, flushing, thirst, palpitations,
vertigo, and chest pain.

What is the mechanism of action of the medication used to treat this condition?
Metronidazole is effective specifically against anaerobic microorganisms. It diffuses across the cell membrane
of microorganisms and is reduced in the mitochondria of obligate anaerobes to cytotoxic intermediates. These
intermediates cause DNA strand breakage and generate free radicals that consequently damage the cell.
Furthermore, the reduction of metronidazole creates a concentration gradient that leads to further uptake of the
drug.

What are other uses of this medication?


Metronidazole is used to treat Clostridium difficile (anaerobe) infection in pseudomembranous colitis, amebic
dysentery, bacterial vaginitis, and Trichomonas vaginitis and as a component of triple therapy for Helicobacter pylori
eradication. Broadly, it is effective against most anaerobic bacteria and various protozoa.

03_USMLE-STEP1_ch03_039-088.indd 60 9/17/18 3:31 PM


Microbiology CHAPTER 3 61

CASE 22
A 22-year-old woman presents to the ED in labor. She states that she has not received any prenatal care, and this
is her first pregnancy. While in triage for the ED, she has a normal spontaneous vaginal delivery of a boy. The baby
appears normal at birth, but after 12 hours, he seems lethargic. He starts to become tachypneic, his blood pressure
drops, and his hands and feet begin to feel cold and clammy.

What infectious agents are most frequently responsible for neonatal sepsis?
Group B streptococci (GBS), Escherichia coli, and Listeria monocytogenes are common causes of sepsis, pneumonia,
and meningitis in newborns. GBS often colonizes the vaginal flora of women and can be transmitted vertically
during vaginal delivery. Most women are tested by a vaginal swab at 37-38 weeks. If they are not tested during this
period, they are tested immediately prior to delivery. This patient’s lack of prenatal care, vaginal delivery, and onset
soon after birth make GBS sepsis a likely diagnosis.

What is the next step in identifying the causative agent?


In Gram staining of a blood sample, GBS appear as gram-positive cocci, L monocytogenes appears as motile
gram-positive rods, and E coli appears as gram-negative rods.

How did the infant become infected?


These bacteria can spread through the placenta or be acquired from the birth canal during delivery. The mother
may be infected or colonized but asymptomatic. However, pregnant and postpartum women are also at risk for
GBS urinary tract infection or chorioamnionitis.

What prenatal testing is routinely performed to reduce the infant’s risk of this infection in the birth canal?
If the patient receives good prenatal care, cultures of the mother’s vagina and rectum are performed between
35 and 37 weeks of gestation to determine whether she is colonized with GBS.

What treatment is initiated if prenatal testing is positive?


Treatment of GBS in infected mothers or newborns involves the use of intrapartum antibiotics such as penicillin.
In mothers who are colonized vaginally or rectally, but who are not actively infected, intrapartum penicillin is
recommended. If the status of the mother is unknown, the mother is prophylactically given penicillin while she is
in labor. If the patient is allergic to penicillin, the patient should be desensitized to penicillin, then given penicillin.

If the baby develops meningitis from this organism, what cerebrospinal fluid findings are expected?
In bacterial meningitis, the cerebrospinal fluid may show bacteria on Gram stain. In addition, the WBC count is
elevated, primarily with neutrophils; the protein level is elevated; and the glucose level is reduced.

03_USMLE-STEP1_ch03_039-088.indd 61 9/17/18 3:31 PM


62 CHAPTER 3 Microbiology

CASE 23
A 3-year-old boy is brought to the pediatrician by his
mother because she is worried her son is not eating and
drinking. He began to refuse solid foods 2 days ago, and
he has only been drinking small sips. Today, the mother
noticed a rash on the child’s hands, feet, and face (see
Figure 3-21). His temperature is 100°F. She noticed other
children at the day care seem to have the same thing.
She is starting to notice symptoms in her other young
children at home. She is worried he has picked up herpes
from a relative that has a history of cold sores. FIGURE 3-21. (Reproduced with permission from Di B, et al. Virol J
2014;11:157.

What is the most likely diagnosis?


This is a case of hand, foot, and mouth disease, caused by coxsackie A virus. This syndrome presents with a tender
rash (see Figure 3-21) on the palms, soles, and often the buttocks and painful vesicles on the oral mucosa. This
patient’s avoidance of solid food strongly suggests involvement of the oral mucosa.

What other microorganisms are included in this family that caused this condition?
The Picornaviridae are a family of single-stranded positive-sense RNA viruses. The members of this family cause
a wide array of illness, possibly because of the high virulence of positive-sense single-stranded RNA, which can
be directly translated into protein products by host ribosomes. These can cause a wide variety of encephalitis,
meningitis, and cardiomyopathy, as more of the extreme infections from these. Members of the Picornaviridae
family include:
• Poliovirus
• Echovirus
• Hepatitis A virus
• Coxsackie viruses
• Rhinovirus

What other conditions can this microorganism cause?


Herpangina, which presents with sore throat, red vesicles on the back of the throat, pain with swallowing, and
fever. Herpangina is a mild, self-limited disease that presents in children and usually results in complete recovery.
Less commonly, coxsackie A virus can cause petechial and purpuric rashes, which may also have a hemorrhagic
component.

What illnesses may be caused by the group B coxsackie viruses?


The coxsackie B virus may cause aseptic meningitis, myocarditis, pericarditis, dilated cardiomyopathy, orchitis, and
epidemic pleurodynia (fever, headache, spasms of the chest wall muscles, and pleuritic pain). Nephritic syndrome
may also occur after a coxsackie B virus infection.

What is the differential diagnosis for a rash of the palms and soles?
Other than Coxsackie A virus, Rocky mountain spotted fever caused by Rickettsia rickettsii, secondary Syphilis, and
Kawasaki disease (vasculitis) commonly present with a rash on the palms and soles (mnemonic: Kawasaki CARS).

03_USMLE-STEP1_ch03_039-088.indd 62 9/17/18 3:31 PM


Microbiology CHAPTER 3 63

CASE 24
An 18-year-old woman presents to the clinic with itching
and dysuria. She also has a fever and headache. When
asked, she says that she recently became sexually active
with a new partner. She states that she was recently tested
for “some sexually transmitted diseases” at her annual
exam with her gynecologist, 3 weeks ago. She was positive
for chlamydia at the time and was treated before she
had the encounter with a new partner. She does not take
birth control and does not use any protection from STDs.
The patient’s sexual partner denied any STDs. She never
received an HPV vaccination as a child. She has recently
started using a new douche, and she thinks this could be
causing these lesions. Physical examination reveals tender
inguinal lymphadenopathy and red, pustular, painful
vesicles on her labia majora (see Figure 3-22). FIGURE 3-22. (Reproduced courtesy of SOA-AIDS Amsterdam.)

What is the most likely diagnosis?


Herpes simplex virus type 2 (HSV-2). The pathognomonic findings for herpes infections are painful vesicles. Both
HSV-1 and HSV-2 can cause genital herpes (as shown in Figure 3-22), but more than 80% of genital lesions are due
to HSV-2.

What are the characteristics of this pathogen?


HSV-2 is a member of the Herpesviridae family, which are double-stranded DNA viruses and include: HSV-1,
varicella-zoster virus (VZV), Epstein-Barr virus (EBV), cytomegalovirus, human herpesvirus-6, and human
herpesvirus-8. They can be recognized by multinucleated giant cells on Tzanck smear and by eosinophilic
intranuclear inclusions.

What is the differential diagnosis of painful genital lesions?


Chancroid caused by Haemophilus ducreyi infection. Lymphogranuloma venereum caused by Chlamydia
trachomatis and granuloma inguinale caused by Klebsiella granulomatis also cause painful genital lesions.

What is the typical course of this infection?


HSV-2 is transmitted by direct contact of the virus with mucosal surfaces or open skin surfaces. It can also be
transmitted from mother to newborn during delivery. Approximately 80% of infected patients are asymptomatic.
The primary infection often presents with constitutional symptoms such as fever, headache, malaise, and myalgia.
Later, genital vesicles may appear that can rupture and leave behind painful ulcers. Other genital symptoms
include itching and tender inguinal lymphadenopathy. Like other viruses in the family, HSV-2 becomes latent and
can be reactivated. Triggers for reactivation include fever, trauma, emotional stress, sunlight, and menstruation.
Upon reactivation, there is often a viral prodrome that involves tenderness, pain, and burning at the future site of
vesicle eruption. The lesions last 4–15 days before crusting over and re-epithelializing. It is important to remember
herpes encephalitis for test questions involving a patient with personality changes and a meningitis-like
presentation. Lesions typically originate in the temporal lobe.

Where does this species remain latent?


In the peripheral nervous system ganglia. HSV-1 tends to remain latent in the trigeminal ganglion, reactivating and
causing oral herpes or “cold sores.” HSV-2 and VZV tend to remain latent in the dorsal root ganglia of the sensory
afferents. This gives rise to the pathognomonic dermatomal distribution of reactivated zoster infections.

What is the treatment for this condition?


The treatment for HSV-2 is acyclovir, a nucleoside analog that acts by inhibiting viral DNA polymerase when it is
phosphorylated by viral thymidine kinase. However, because efficacy requires viral thymidine kinase activity, any
herpesvirus lacking a functional thymidine kinase will be resistant. Although there is suppressive treatment with
acyclovir, there is no cure for herpes. Herpes is a chronic life-long infection. It can be important during pregnancy
because if the mother has an active infection at time of delivery, she should have a C-section instead of a vaginal
delivery.

03_USMLE-STEP1_ch03_039-088.indd 63 9/17/18 3:31 PM


64 CHAPTER 3 Microbiology

CASE 25
A 9-year-old girl is brought to a university pediatric
clinic for a well-child exam. During the history,
you find that the family recently immigrated from
Guatemala. Per her growth chart, the girl is small for
her age, and she is not very active. She is not caught up
on her vaccinations currently, and her mother would
like her to be caught up. She has not received any
vaccinations previously. For the last year, she has had
some trouble breathing and gastrointestinal issues
such as chronic constipation. Physical examination
reveals a small girl with a thin, scaphoid abdomen.
In laboratory studies, she is found to be anemic and
protein deficient. The mother relays that the child
has been eating a varied diet, so she is surprised that
she is protein deficient. The mother has hepatitis
B, and she is worried she may have passed it to the
child. Relevant laboratory findings are as follows:
Hematocrit: 36%
Mean corpuscular volume: 73 fL
WBC count: 11,000/mm3 FIGURE 3-23. (Reproduced with permission from USMLE-Rx.com.)
Differential: 3
 5% segmented cells, 1% bands,
33% lymphocytes, 21% eosinophils
Figure 3-23 shows stool testing using wet mount.

What is the most likely diagnosis?


Hookworm, or nematode, infection. The findings of eosinophilia and microcytic anemia with recent immigration
from an endemic area are highly suggestive of this condition.

What is the next step in confirming the diagnosis?


Stool ova and parasite tests can confirm the presence of characteristic small, round eggs (as seen in Figure 3-23)
and occasional worms approximately 1 cm in size. Stool ova and parasite tests can also be used to delineate the
species of helminth.

What are the other species in this genus?


Ancylostoma duodenale, Necator americanus, and Ancylostoma braziliense are the most common hookworms. Of
the three, N americanus is the most commonly tested, and it causes the classic gastrointestinal symptoms and
microcytic anemia seen in this patient. A braziliense can manifest as a condition known as cutaneous larva migrans,
in which the larva migrate to the subcutaneous tissue and create pruritic, serpiginous tracts underneath the skin.

What other helminth is known to cause anemia?


Diphyllobothrium latum, a tapeworm, causes vitamin B12 deficiency leading to a macrocytic anemia.

How does this infection cause disease in humans?


Percutaneous infection occurs generally through the soles of the feet and is acquired commonly from sandboxes.
The larvae pass into the lungs, and 8–21 days later they cross the pulmonary vasculature and enter the airways.
They ascend to the pharynx and are swallowed. This swallowing of the worms often leads to a chronic cough. By
the time they reach the small intestine, the larvae have become adult worms. The adults “hook” onto the mucosa
and feed on the host’s blood with the help of an orally secreted factor X inhibitor. This results in the microcytic
anemia. The females produce eggs that are passed through the stool and deposited in the soil.

What are the treatments for this condition?


Since hookworm is a helminthic infection, mebendazole and albendazole are the first-line agents. These agents
disrupt helminthic microtubule synthesis, leading to structural weakening and death of helminthic cells. Pyrantel
pamoate can be used as a second-line agent.
Microbiology CHAPTER 3 65

CASE 26
A 46-year-old woman visits her physician in January complaining of “feeling poorly.” She has had fever, chills, muscle
aches, dry cough, and sore throat with no improvement from taking over-the-counter medication for the past few
days. She works as a secretary at a primary care office and says many patients have been coming to the office with
these symptoms. She has been having some diarrhea as well. She is worried that it is “something serious” because
she received an influenza vaccine this year. Physical examination reveals small, tender cervical lymphadenopathy,
swollen nasal mucosa, and an erythematous pharynx.

What is the most likely diagnosis?


She is most likely infected with the influenza virus, an orthomyxovirus.

What are the defining structural features of this class of microorganisms?


Orthomyxoviruses are helical, enveloped, negative, single-stranded RNA viruses with eight segments. Their
primary virulence factors are hemagglutinin and neuraminidase. Hemagglutinin aids in the viral entry into host
cells whereas neuraminidase aids in progeny release from infected host cells. The isotypes of these two proteins
determine the virulence of each particular strain of virus and are the targets of the influenza vaccine. Figure 3-24
shows the general structure of the influenza virus and the location of the neuraminidase and hemagglutinin.

Hemagglutinin Neuraminidase
PB1, PB2, PA
RNA
1 PB2
2 PB1
3 PA
4 HA
5 NP
6 NA
7 M1 + M2
8 NS1 + NS2

M2 matrix protein

Lipid bilayer
M2 ion channel

FIGURE 3-24. The general structure of the influenza virus and the location of neur-
aminidase and hemagglutinin. (Reproduced with permission from USMLE-Rx.com.)

The patient has had a similar infection in the past. Why is her immune system not protecting her from this
illness?
The isotype of hemagglutinin and neuraminidase is constantly changing because of a phenomenon known as
antigenic drift. This is the result of random small mutations that cause changes in the antigenic structure of the
virus. These mutations result in antigen structures that are only partially recognized by the host immune system.

What characteristic of this microorganism’s genome makes deadly epidemics possible?


Influenza A virus infects diverse species including birds, horses, and swine; by contrast, influenza B and influenza
C infect only humans. With its segmented genome, influenza A can swap segments of RNA between animal and
human strains (a process known as reassortment), leading to new human strains with novel surface antigens not
recognized by the immune system. This type of change is termed antigenic shift and was responsible for the
“swine flu” or hemagglutinin isotype 1 and neuraminidase isotype 1 (H1N1) influenza A virus outbreak in 2009.

What pharmacologic agents can be used as prophylaxis and treatment against this infection?
Amantadine and rimantadine block viral penetration by inhibiting the M2 protein responsible for uncoating and
can be used to treat influenza A infection. However, these drugs are rarely used anymore because of the high levels
of resistance that have developed against them. Instead, zanamivir and oseltamivir (neuraminidase inhibitors) are
used to treat both influenza A and influenza B infections. These agents are most effective if started within 48 hours
of symptom onset. The influenza vaccine should be given in October or November before the start of flu season. It
takes approximately 2 weeks for the body to make antibodies to the viruses. The vaccine generally has four strains
chosen by the Centers for Disease Control and Prevention.
66 CHAPTER 3 Microbiology

CASE 27
A 43-year-old man with HIV infection presents to
the HIV clinic with multiple purple-red plaques and
papules distributed across his skin (see Figure 3-25). He
has a history of medication noncompliance. He reports
that he has stopped taking his HARRT therapy because
he had been feeling better. He has noticed some
increased diarrhea and blood on the toilet paper when
he wiped. He has had hemorrhoids before, but he does
not feel the typical itching that he usually feels with
them. The patient says he feels fine and denies fever,
chills, malaise, or headache. He has not been using any
illicit drugs, and he has been practicing safe sex. He has
had “some type of fungus” in his lungs before, and he
was hospitalized 2 years ago for it. A complete blood
count reveals his CD4+ cell count is 180 cells/mm3. FIGURE 3-25. (Reproduced courtesy of the National Cancer Institute.)

What is the most likely diagnosis?


This is Kaposi sarcoma, an angiogenic neoplasm prevalent in HIV-positive patients. Kaposi sarcoma is caused by
human herpesvirus-8 (HHV-8), a member of the Herpesviridae family. Members of this family are DNA viruses with
a double-stranded, linear genome in an enveloped, icosahedral capsid.

What important alternative diagnosis must be ruled out?


An important alternative diagnosis for such skin lesions in HIV patients is bacillary angiomatosis (BA), which
typically presents with systemic symptoms such as fever, chills, and malaise. However, because BA is caused by
Bartonella henselae, cat-scratch fever, it is non-neoplastic and can readily be treated with antibiotics.

How does the microorganism cause the characteristic discolored skin lesions?
HHV-8 has a tropism for endothelium cells and is thought to induce vascular endothelial growth factor, which
causes irregular vascular channels to develop in the skin. RBCs extravasate into these spaces, causing the
characteristic purple-red skin lesions as seen in Figure 3-25.

What other diseases are associated with this microorganism?


Kaposi sarcoma is not limited to the skin; it can affect other mucosal surfaces such as the gastrointestinal tract,
oral mucosa, lungs, lymph nodes, and other visceral organs. This can lead to gastrointestinal bleeding. HHV-8 also
infects B lymphocytes and has been linked to body-cavity B-cell lymphoma (a non-Hodgkin lymphoma subtype)
and to Castleman disease (a lymphoproliferative disorder that may progress to lymphoma).

What other patient population is at increased risk for developing this infection?
Transplantation patients, who, like patients with HIV, are chronically immunosuppressed, have a higher incidence
of infection than the general public. Middle-aged men of Mediterranean descent can also be affected by Kaposi
sarcoma, and it is not related to immunosuppression.

What are the treatments for this condition?


Daunorubicin or doxorubicin is the treatment of choice. Both cause DNA breaks by two mechanisms:
(1) intercalating into the DNA double helix and (2) creating oxygen free radicals that damage DNA. A major adverse
effect of their use, however, is cardiotoxicity. In HIV-positive patients, the first goal is to boost immunity by starting
highly active antiretroviral therapy (HAART), which often leads to improvement of the disease.

At a CD4+ cell count of < 200 cells/mm3, what preventative measures should be taken?
The patient should be started on trimethoprim-sulfamethoxazole therapy for Pneumocystis jiroveci pneumonia and
toxoplasmosis prophylaxis.

03_USMLE-STEP1_ch03_039-088.indd 66 9/17/18 3:31 PM


Microbiology CHAPTER 3 67

CASE 28
A 64-year-old man with a history of smoking and well-controlled diabetes mellitus presents to the ED with a 3-day
history of low-grade fevers, mild diarrhea, and nonproductive cough. He works as a maintenance worker in a local
apartment complex, and he states that many of the tenants have been in the hospital with a “lung infection.” Workup
includes a Gram stain of sputum, which shows prominent polymorphonuclear leukocytes but no microorganisms.
X-ray of the chest reveals diffuse, patchy bilateral infiltrates. Silver stain of the specimen is shown in Figure 3-26.
Relevant laboratory findings are as follows:
Hemoglobin: 14 g/mL
Sodium: 128 mEq/L Bicarbonate: 17 mEq/L
Hematocrit: 40% Blood urea nitrogen: 16 mg/dL
Chloride: 100 mEq/L Glucose: 110 mg/dL
Platelets: 200,000/mm3 Creatinine: 1.2 mg/dL
Potassium: 4.2 mEq/L Urinalysis: 2+ proteinuria; no glucose, ketones,
WBC count: 15,000/mm3 or blood

FIGURE 3-26. (Reproduced courtesy of the Centers for Disease Control


and Prevention/Dr. William Cherry.)

What is the most likely diagnosis?


Legionnaires disease, an infection caused by the gram-negative rod Legionella pneumophila. Any patient
presenting with diarrhea and pneumonia-like symptoms has Legionella until proven otherwise. The patient will
also have low sodium with suspected pneumonia, which raises suspicion for Legionella. This patient has evidence
of interstitial pneumonia, which makes the diagnosis even more likely. Legionella does not Gram stain well, but it
stains well with silver stain (see Figure 3-26). It grows only on charcoal yeast extract medium with iron and cysteine
added.

What is the differential diagnosis for atypical pneumonia?


The common differential diagnosis for atypical pneumonia is first and foremost viral infection, followed by
Chlamydia, Mycoplasma, or Legionella infection.

What test can help confirm the diagnosis?


Urinary Legionella antigen test can establish the diagnosis. Legionella is unique in that it is the only form of
community-acquired pneumonia that can be diagnosed with a urine test.

What risk factors does the patient have for developing this condition?
The patient’s history of diabetes and smoking predisposes him to Legionella infection. Given his occupation as a
maintenance man, he likely works with air conditioning systems. As this microorganism grows in infected water
sources, the patient’s occupation places him at risk.

What are the treatments for this condition?


Legionella responds best to antibiotics that can achieve a high intracellular concentration, such as macrolides
(eg, erythromycin, clarithromycin, and azithromycin) and tetracyclines. Legionella produces β-lactamase, so
cephalosporins and penicillins are ineffective.

03_USMLE-STEP1_ch03_039-088.indd 67 9/17/18 3:31 PM


68 CHAPTER 3 Microbiology

CASE 29
A 41-year-old woman who is a recent immigrant
from Mexico presents to a local clinic complaining of
“white spots” on her body. She says she first noticed
the lesions about 1 month ago and thought they
were from the sun, but they have gradually increased
in number and have not improved despite her new
job indoors. Physical examination reveals multiple,
asymmetrically distributed, hypopigmented lesions
on the patient’s arms, abdomen, back, and feet (see
Figure 3-27). The lesions are sharply demarcated,
with raised, erythematous borders and atrophic,
scaly centers. The lesions are anesthetic, and there
is no hair growth within any of the hypopigmented
areas. Biopsy of the lesions demonstrates granuloma
formation within the dermal nerves of the forearm.

Under what conditions does the causative


microorganism grow?
Both lepromatous and tuberculoid leprosy are
caused by Mycobacterium leprae, an acid-fast bacillus
that cannot be grown in vitro. M leprae is an obligate
intracellular bacillus that, like other mycobacteria,
contains mycolic acid in its cell wall. M leprae grows
FIGURE 3-27. (Reproduced courtesy of Centers for Disease Control
best in cooler temperatures (eg, skin, peripheral
and Prevention/Dr. Andre J. Lebrun.)
nerves, testes, upper respiratory tract).

How does this patient’s condition differ from a more severe form?
This patient has tuberculoid leprosy, which is largely confined to the skin (hypopigmented macules as seen in
Figure 3-27) and peripheral nerves. Cell-mediated immunity is intact, and patients’ T cells recognize M leprae
(positive lepromin skin test). Lepromatous leprosy holds a much worse prognosis because patients have ineffective
cell-mediated immunity (negative lepromin skin test). Skin lesions and nerve involvement are much more
extensive than in the tuberculoid form, and there may be involvement of the testes, upper respiratory tract, and
anterior chamber of the eye.

What is the treatment for this condition?


Both tuberculoid and lepromatous leprosy can be treated with a course of oral dapsone. The tuberculoid form is
reliably cured by a short course of this medication. Patients with lepromatous leprosy have an exceptionally high
bacterial load and may require an extended or even lifelong course of chemotherapy. Alternate therapies for
leprosy include rifampin or a combination of clofazimine and dapsone.

What are the adverse effects of treatment?


Dapsone can cause agranulocytosis, so patients should be monitored initially with weekly or biweekly complete
blood counts. Rifampin can turn body fluids such as sweat, tears, and urine a red-orange color. This can startle
patients and clinicians into thinking that the urine is bloody if they are not counseled before commencing therapy.

03_USMLE-STEP1_ch03_039-088.indd 68 9/17/18 3:31 PM


Microbiology CHAPTER 3 69

CASE 30
The mother of a 1-week-old girl calls her pediatrician because the infant has been fussy all morning. The infant’s
temperature is 103°F, and the mother is asked to bring the infant to the hospital. The workup includes cerebrospinal
fluid (CSF) analysis, hematology studies, and cultures. Empiric antibiotic therapy is initiated. Later, upon microscopic
examination of the CSF, microorganisms with tumbling end-over-end motility are visualized.

What is the most likely diagnosis?


Meningitis due to Listeria monocytogenes infection. This microorganism, identifiable by its classical tumbling
motility, is a gram-positive rod and a common cause of meningitis in newborns and the elderly (see Table 3-1).

Table 3-1. What Are Some Examples of Live-Attenuated Vaccines vs Killed?


Organism Newborn Infant/Child Teen/Adult Elderly
Streptococcus pneumoniae X X X
Neisseria menigitidis X X
Enterovirus X X
Escherichia coli (gram-negative rod) X X
Listeria X X
Group B Streptococcus X

How is the pathogen in this condition transmitted?


L monocytogenes is transmitted through ingestion of unpasteurized dairy products such as milk, cheese, ice cream,
deli meat, and most recently, hummus.

What microorganisms should empiric antibiotic therapy target?


Group B streptococcus, Escherichia coli, and L monocytogenes are the most common causes of sepsis and bacterial
meningitis in infants younger than 1 month of age. Therefore, empiric therapy should be aminopenicillin
or vancomycin for gram-positive infection and aminoglycosides, antipseudomonal penicillins, or third- or
fourth-generation cephalosporins for gram-negative infection.

How does this microorganism evade the host immune response?


L monocytogenes is a facultative intracellular bacterium able to survive in the macrophages of neonates and
immunosuppressed patients. In an immunocompetent host, activation of macrophages destroys phagocytosed
Listeria.

What other population is at particular risk for developing the same infection?
Pregnant patients are at increased risk of developing a serious illness from Listeria known as granulomatosis
infantiseptica. It can cause various complications to the mother and baby from premature rupture of membranes
and intrauterine fetal demise. The elderly and immunocompromised are also at increased risk for this infection.

03_USMLE-STEP1_ch03_039-088.indd 69 9/17/18 3:31 PM


70 CHAPTER 3 Microbiology

CASE 31
A 30-year-old woman presents to the clinic with abdominal pain, a low-grade fever, and a sensation of abdominal
fullness. She says the symptoms have been going on for some time and have been gradually worsening. On physical
examination she appears jaundiced with notable scleral icterus. She says she is originally from South America. She
mentions that she breeds and trains sheep dogs for a living. Based on radiographic findings, surgery is performed
and a biopsy specimen is shown in Figure 3-28.

What is the most likely diagnosis?


The patient most likely has a hydatid cyst, which is a
liver cyst due to Echinococcus infection.

How is this infection transmitted?


Echinococcus is a tapeworm that is transmitted by food
or water contaminated with feces containing eggs from
the tapeworm. Infection is not endemic to the United
States and so is most commonly seen in immigrants or
those with a travel history to endemic areas. The sheep
dog is an intermediate host, while humans serve as
dead-end hosts for these organisms.

What is the typical presentation of this infection?


Echinococcus causes slow-growing cysts in the liver,
as seen in Figure 3-28. As a result, symptoms are
often gradual in onset and include abdominal pain,
cough, low-grade fever, a sense of abdominal fullness,
hepatomegaly, and obstructive jaundice. Leakage of
cysts can cause flushing and urticaria, whereas rupture
can cause anaphylaxis and death. Other organs that
can be involved include the lungs and the brain. In
the lungs, the presentation includes chronic cough,
dyspnea, hemoptysis, and pleuritic chest pain. In the
brain, presentation includes headache, dizziness,
increased intracranial pressure, and hydrocephalus.

How is this condition diagnosed?


On x-ray of the abdomen, a rim of calcification around
the cyst is able to distinguish hydatid cyst from amebic
and pyogenic cysts. However, the diagnosis usually
FIGURE 3-28. (Reproduced courtesy of Dr. Yale Rosen.)
cannot be made with radiology alone and requires
an enzyme-linked immunosorbent assay (ELISA). In
addition, 25% of patients have eosinophilia.

What is the treatment for this condition?


The treatment for Echinococcus infection is usually surgical and involves aspiration of cyst contents followed by
excision. However, during drainage, the interventional radiologist or surgeon must be careful not to rupture the
cyst as this can lead to anaphylaxis. Therefore, many physicians prefer to inject formalin or ethanol into the cyst to
kill the organism before aspirating. In some cases, therapy with a combination of albendazole and mebendazole is
sufficient.

What other organisms are classified as cestodes?


Diphyllobothrium latum is a cestode transmitted by ingestion from freshwater fish that causes vitamin B12-deficient
macrocytic anemia. D latum is treated with praziquantel. Taenia solium larvae are ingested from undercooked
pork and can cause calcified cysts in various organs, including the brain (cysticercosis or neurocysticercosis).
Cysticercosis is treated with praziquantel, whereas neurocysticercosis is treated with albendazole.

03_USMLE-STEP1_ch03_039-088.indd 70 9/17/18 3:31 PM


Microbiology CHAPTER 3 71

CASE 32
A 49-year-old man presents to the ED after a syncopal episode. He said he had just stood up from urinating, and he
felt dizzy and fell down. He thought he might have hit his head, so his boss made him go to the ED. He denies any
chronic health problems and states that he stays fit by walking several miles through the local park every day. He
recently returned from a camping trip to Vermont. Physical examination shows bradycardia and a 12-lead ECG is
ordered (see Figure 3-29). On review of systems, the patient states that he has had low-grade fevers over the past few
days. Upon physical exam, the resident notices an area of induration in his left groin surrounded by an erythematous
ring.
FIGURE 3-29. (Reproduced, with permission, from Kasper D,
et al. eds. Harrison’s Principles of Internal Medicine, 19th ed. New
York, NY: McGraw-Hill; 2014.)

What is the most likely diagnosis?


Third-degree atrioventricular (AV) block secondary to Lyme disease. This condition is caused by Borrelia
burgdorferi, a gram-negative spirochete that is poorly grown in culture and too small to be seen under regular light
microscopy. Fluorescence may be used to visualize the corkscrew-shaped bacterium. However, the diagnosis is
usually made clinically, supported by serology. Early local infection may present with a bull’s-eye rash (erythema
chronicum migrans) after several days. The early-disseminated stage may develop as early as a few days later and
presents with cardiac conduction abnormalities (Lyme carditis), cranial nerve palsies (especially cranial nerve VII),
and meningitis. Up to 43% of patients with Lyme carditis develop complete heart block. The diagnosis of Lyme
disease is usually clinical, and PCR tests can be inaccurate. If there is suspected infection, even in children, the
treatment of choice is doxycycline. If a tick is found soon after a hike through the woods, there does not need to be
prophylaxis. The tick needs 36 hours of latching to spread the disease.

What other conditions can cause this condition?


Damage to the heart’s conduction system by fibrosis, ischemia, cardiomyopathy, myocarditis, or iatrogenic
damage (eg, after valve replacement) may cause complete heart block. Digitalis, calcium channel blockers, and
β-blockers may produce a temporary conduction abnormality.

What is the route of infection?


Lyme disease is an arthropod-borne infection. The Ixodes tick transmits B burgdorferi. Mice and deer are reservoirs
for the disease. These ticks are most often found in wooded areas in the Northeastern United States.

What is the prognosis for this patient?


The prognosis is good. The conduction abnormalities secondary to Lyme carditis are self-limited and short-lived
and often resolve within days to weeks. It is uncommon for residual conduction abnormalities to persist after the
infection has been cleared.

What is the immediate treatment for this patient?


The patient’s ECG demonstrates bradycardia at a rate of 40/min. The episode of syncope indicates that cerebral
perfusion is inadequate at this heart rate. Consequently, transvenous pacing may be initiated, but a permanent
pacemaker is not needed. Antibiotic treatment for Lyme carditis consists of intravenous ceftriaxone until the PR
interval is < 300 ms, at which point oral antibiotics may be initiated. The same regimen applies to Lyme disease
with neurological features. Doxycycline or amoxicillin is used for primary infection and Lyme arthritis only.

03_USMLE-STEP1_ch03_039-088.indd 71 9/17/18 3:31 PM


72 CHAPTER 3 Microbiology

CASE 33
While doing a rotation in Ghana, a medical student encounters a patient who has been having nearly continuous
high-grade fevers with occasional chills and sweats. The fevers seem to be cycling between incredibly high then
back to normal over the day. Physical examination reveals a palpable spleen. The medical student remembers a
lecture from parasitology and wants to do a test he heard about before, so he places a drop of the patient’s blood in
a copper sulfate solution, which reveals anemia. Over the next few days, while waiting for medication to arrive, the
patient’s level of consciousness waxes and wanes, and he is somnolent at times.

What is the most likely diagnosis?


Malaria due to Plasmodium falciparum. The symptoms give clues as to the species. The patient’s altered mental
status is consistent with a diagnosis of P falciparum malaria, since this is the only strain that commonly has cerebral
involvement. This patient’s continuous fever and irregular chills and sweats are also characteristic of
P falciparum malaria. Early in infection, irregular fevers are common in all types of malaria, but the fever can
become periodic in well-established cases of non-falciparum disease. For example, P vivax and P ovale cause
episodes of fever, chills, and sweats every 48 hours. With P malariae, these episodes occur every 72 hours.
Splenomegaly is a common finding in malaria due to work hypertrophy from increased RBC breakdown. This is
differentiated from Trypanosoma brucei, which would not have the splenic involvement, but it would have fever,
headache, fatigue, muscle aches, and swollen lymph nodes. The tsetse fly spreads Trypanosoma brucei (African
Sleeping Sickness), which is endemic to Africa as well. Do not be tricked into thinking malaria is always the answer.

What phase of the microorganism’s life cycle results in the development of anemia?
RBC lysis occurs during the erythrocytic cycle, when the products of asexual replication inside the RBCs (the
merozoite form) are released. The immune response to the merozoites and resulting cytokine release is responsible
for the fever, chills, and sweats. If left untreated, malaria can migrate to the brain, and there it blocks the capillaries
and blood vessels in the brain, leading to coma and death. It can also cause severe anemia and hypoglycemia.

What are the likely peripheral blood smear


(PBS) findings?
A PBS is likely to show ring-shaped trophozoites
inside the RBCs (see Figure 3-30), and there may
be several trophozoites per RBC. Schizonts,
the large, multinucleated cells formed from
the trophozoite by multiple cycles of nuclear
division, may be visible in the erythrocytes in
non-falciparum malaria but are very rarely seen
in falciparum disease. Outside the RBCs, oblong
gametocytes, diagnostic for P falciparum, may
also be visible.

FIGURE 3-30. Malaria shown in the peripheral smear shows ring formation
within the erythrocyte. (Reproduced with permission from USMLE-Rx.com.)

What is the treatment of choice for this condition?


Chloroquine is the drug of choice in the few areas where there is no resistance. Its major mode of action against
Plasmodium is inhibition of the enzyme responsible for polymerizing heme. This results in the accumulation of free
heme, which is toxic to the protozoan. Quinidine in combination with doxycycline or pyrimethamine/sulfadoxine
is commonly used as first-line treatment for chloroquine-resistant P falciparum. Other effective drugs include
mefloquine and atovaquone-proguanil.

What conditions provide protection again this condition?


Sickle cell trait and glucose-6-phosphate dehydrogenase (G6PD) deficiency both protect against malaria. The
hypothesis is that increased fragility of the erythrocytes in these diseases does not allow for Plasmodium species to
effectively replicate.
Microbiology CHAPTER 3 73
CASE 34
A mother brings her 14-year-old son to the pediatrician because the child has been experiencing flu-like symptoms
and conjunctivitis for the past 3 days. The child is pale and febrile at 39.9°C (102.7°F), and his respiratory rate is
25/min. His buccal mucosa has multiple blue-gray spots (see Figure 3-31), and he has a maculopapular rash (see
Figure 3-32). His mother states that the rash started on his face but has spread to his torso. The physician notes that
the skin lesions blanch with pressure. The mother states that she usually uses naturopathic home remedies for her
family’s illnesses, and she has never vaccinated her family.

FIGURE 3-31. (Reproduced courtesy of the Centers for Dis- FIGURE 3-32. (Reproduced courtesy of the Centers for Dis-
ease Control and Prevention.) ease Control and Prevention/Heinz F. ­Eichenwald, MD.)

What is the most likely diagnosis?


This child has measles, one of the most transmissible viral infections. Measles is caused by an RNA virus that is a
member of the family Paramyxoviridae. Transmission occurs by respiratory droplets, and the incubation period is
approximately 10 days. Two of this patient’s symptoms are pathognomonic of measles: Koplik spots on the buccal
mucosa (see Figure 3-31) and blanching rash (see Figure 3-32) with cephalocaudal spread.

What should be considered in the differential diagnosis?


Flu-like symptoms may also be caused by infection with rhinoviruses, parainfluenza, influenza, adenovirus, or
respiratory syncytial virus. Common causes of rash include Mycoplasma pneumoniae, human herpesvirus-6, rubella,
Rocky Mountain spotted fever, scarlet fever, or a drug reaction. However, none of these rashes have cephalocaudal
spread and none feature Koplik spots.

What are potential neurologic sequelae of this condition?


Approximately 1:1000 patients with measles develop encephalitis, which is rapidly fatal in 15% of cases and leads
to permanent neurological damage in 25% of patients. Infection with Morbillivirus can trigger acute disseminating
encephalomyelitis, an autoimmune attack on the central nervous system, which is lethal in up to 20% of cases.
Survivors often suffer intellectual disability and epilepsy. Approximately 7–10 years after a measles infection,
subacute sclerosing panencephalitis may develop; this infection, although rare, is almost always fatal. This often
presents as sudden encephalitis of an unvaccinated teenager.

What tests can confirm the diagnosis?


Anti-measles IgM is seen in patient serum approximately 48 hours after the onset of the rash. A mucosal biopsy
may demonstrate Warthin-Finkeldey cells (multinucleated giant cells with inclusion bodies in the nucleus and
cytoplasm).

What is the treatment for this condition?


There is no treatment for active infection; care is limited to supportive measures. Measles is a reportable disease,
and the patient must be placed in isolation. If other children in the family have not received the measles-mumps-
rubella vaccine, it is likely that they will develop measles because this virus is highly transmissible.

03_USMLE-STEP1_ch03_039-088.indd 73 9/17/18 3:31 PM


74 CHAPTER 3 Microbiology

CASE 35
A 19-year-old college sophomore presents to the university health center with a 7-day history of sore throat,
headache, and fatigue. He has a temperature of 37.7°C (99.9°F). Physical examination reveals enlarged, tender
cervical lymph nodes in both the anterior and posterior cervical chain. The spleen is found to protrude 5 cm under
the costal margin with inspiration. Upon examination of his oropharynx, gray-green tonsillar exudate is noted. He
was originally given amoxicillin by an urgent care provider, but he stated this caused him to break out in a rash.

What is the most likely diagnosis?


Infectious mononucleosis is most frequently caused by Epstein-Barr virus (EBV), a member of the Herpesviridae
family, but it can be also caused by the cytomegalovirus.

What should be considered in the differential diagnosis?


Several diseases present with symptoms similar to mononucleosis. However, streptococcal infection of the
oropharynx is usually not associated with splenomegaly, and cytomegalovirus pharyngitis tends to be mild if
clinically apparent at all. Low-grade fever, lymphadenopathy, and splenomegaly may also be seen in lymphoma.
Lymphoproliferative disorders generally do not present with tonsillar exudate.

What are the PBS findings?


Infectious mononucleosis leads to lymphocytosis. The WBC count is often elevated (12,000–18,000/mm3)
with more than 50% lymphocytes. Up to 10% “atypical” lymphocytes containing large amounts of cytoplasm
may be seen. However, these findings may also be seen in other infections (eg, cytomegalovirus, rubella, and
toxoplasmosis), in some malignancies, and as a result of drug reactions.

Which malignancies are associated with this infection?


Burkitt lymphoma is endemic to Africa and primarily affects children. The disease is a B-cell lymphoma and
often presents with a tumor of the jaw. This tumor often has a “starry sky” appearance under light microscope.
Nasopharyngeal carcinoma is one of the most common cancers in southern China, and evidence supports EBV as
its primary causative agent.

What treatments are available for this condition?


Symptomatic treatment, usually with nonsteroidal anti-inflammatory drugs, is usually used. Although EBV might
be expected to be susceptible to acyclovir since it is a herpesvirus, studies have shown that acyclovir produces
reduction of oral shedding of the virus but no other significant clinical benefit. Ampicillin should be avoided, not
just because it is ineffective against viruses but because it can precipitate a rash.

03_USMLE-STEP1_ch03_039-088.indd 74 9/17/18 3:31 PM


Microbiology CHAPTER 3 75
CASE 36
A 55-year-old woman presents to the ED with confusion
and lethargy. On physical examination she is found to
be tachypneic and tachycardic, and her breath smells
sweet and fruity. A review of her electronic medical record
reveals that she was diagnosed with type 1 diabetes
mellitus at 12 years of age. She has never been very
compliant with her diabetes and insulin regimen. She has
been admitted three times in the past year with diabetic
ketoacidosis. She is admitted to the hospital and treated
for diabetic ketoacidosis, and her symptoms begin to
improve. However, 4 days after admission she develops
fever, mucoid nasal discharge, and periorbital swelling
(see Figure 3-33). While cultures are pending, she is treated
with empiric antibiotics but fails to improve.

What is the most likely diagnosis?


Mucormycosis.
FIGURE 3-33. (Courtesy of the Centers for Disease Control and
What is the microscopic appearance of the fungus Prevention/Dr. Thomas F. Sellers/Emory University.)
involved in this condition?
The zygomycetes are nonseptate, branching fungi, with
wide (> 90-degree) branch angles and large hyphae.
By contrast, Aspergillus tends to have narrow, septated
hyphae and branches in acute angles. Figure 3-34 shows
the large irregular nonseptations.

What patient populations are at increased risk for this


condition?
The fungus is ubiquitous in nature and spores can be
transmitted in air. Most patients are exposed to these
spores several times per year, but an intact immune
system is usually sufficient protection. Therefore,
mucormycosis is typically seen in diabetic patients
with poor glucose control or those with ketoacidosis.
Neutropenic patients, burn victims, and patients treated FIGURE 3-34. Zygomycetes of Mucor. (Reproduced courtesy of
with iron-chelating drugs are also at risk. Dr. Yale Rosen.)

What is the pathogenesis of this condition?


The fungi grow along blood vessels and invade their walls. Protruding hyphae are highly thrombogenic, resulting
in ischemia and necrosis of distal tissues, which, in turn, provide nutrients for continued fungal growth. Necrosis
compromises the integrity of the bony walls of the sinuses and the cribriform plate, allowing fungal growth into
paranasal sinuses, the bony orbit around the eye (as seen in Figure 3-33), or the brain. This rhinocephalic form of
infection is rapidly progressive and carries a high mortality rate. Other forms include pulmonary, gastrointestinal,
cutaneous, and disseminated infection.

What is the management for this condition?


Surgical debridement of infected and necrotic tissue is paramount. Reversal of the permissive condition (in
this case ketoacidosis) should be initiated in conjunction with potent antifungal therapy with amphotericin B.
However, despite optimal management, this condition still carries a high mortality rate.

03_USMLE-STEP1_ch03_039-088.indd 75 9/17/18 3:31 PM


76 CHAPTER 3 Microbiology

CASE 37
A 21-year-old man from Honduras presents to his physician with a 2-day history of painful unilateral testicular
swelling. The patient complains of minimal fever and myalgia about a week earlier. Physical examination reveals
swollen and painful parotid glands. He immigrated to the United States when he was younger, and he does not
remember much of his medical history.

What is the most likely diagnosis?


Orchitis secondary to mumps. In rare instances, orchitis can affect both testes and lead to sterility. It can also cause
sterility if it affects the ovaries.

How is this condition transmitted?


The mumps virus is a member of the Paramyxoviridae family of single-stranded RNA viruses. It is not stable
enough to be aerosolized but can be transmitted by droplets (eg, sneeze). The virus has a 2- to 3-week incubation
period, after which the infection results in painful inflammation and edema of glandular tissues, including the
parotid gland, testes, and ovaries. This question would typically state the person had not been vaccinated or had
immigrated to the United States, which could point to a lack of immunization.

What other clinical syndrome can result from infection with this microorganism?
If the viral infection spreads to the meninges, aseptic meningitis may develop. Pancreatitis can also develop from
this as well. This can be differentiated from bacterial meningitis by analysis of the cerebrospinal fluid (CSF):
Viral meningitis:
CSF protein ∼ 150 mg/dL
Normal CSF glucose
CSF lymphocytes
Bacterial meningitis:
CSF protein > 300 mg/dL
Low CSF glucose
CSF neutrophils
Most viral meningitides are self limited and require only symptomatic treatment. Other common pathogens are
coxsackievirus and echovirus. Mumps meningitis is rare in the United States because of vaccination.

How is the underlying condition diagnosed?


For the most part, the diagnosis of mumps can be made clinically. Supportive laboratory values include elevated
amylase due to infection in the parotid glands. In uncertain cases, viral polymerase chain reaction assay may also
be used.

What is the treatment for this condition?


The treatment is supportive and directed at reducing pain. Analgesics, compression, and icing of the parotid gland
can be useful. In children, aspirin should be avoided as it has been linked to Reye syndrome. Vaccination with live
attenuated mumps virus as part of the measles-mumps-rubella vaccine is used to prevent disease.

03_USMLE-STEP1_ch03_039-088.indd 76 9/17/18 3:31 PM


Microbiology CHAPTER 3 77
CASE 38
A 23-year-old sexually active woman presents 2 weeks
after having sex with a new partner. She has been having
worsening pain in her left knee and pain with urination.
Physical exam reveals a swollen, tender, painful knee
with decreased range of motion. Examination of her
skin reveals small papules with an erythematous base
on her arms. Pelvic examination is notable for purulent
endocervical discharge (see Figure 3-35). Synovial fluid
was sampled from the joint space, and it was purulent FIGURE 3-35. (Reproduced courtesy of the Centers for Disease Con-
with 50,000 polymorphonuclear lymphocytes. trol and Prevention/Susan Lindsley.)

What is the likely causative organism of this condition?


Neisseria gonorrhoeae is contracted through sexual contact with an infected partner, and males are usually
asymptomatic. The vaginal infection can cause discharge (see Figure 3-35) and dysuria. If the bacteria disseminate,
skin lesions, tenosynovitis, or septic arthritis can develop. Septic arthritis is a serious condition that must be treated
aggressively to prevent permanent damage to the joint. Septic arthritis is treated with intravenous antibiotics,
analgesia, and joint washout.

How are septic arthritis, reactive arthritis, rheumatoid arthritis, and osteoarthritis differentiated and what
is the gold standard of diagnosis?
Synovial fluid WBC count is the gold standard for diagnosis and differentiation between the various types of arthritis.
• Osteoarthritis is the most benign and is considered a non-inflammatory arthritis. The synovial fluid WBC count
in osteoarthritis is < 2000 cells/mm3. Gram stain will be negative.
• Reactive arthritis (most commonly associated with Chlamydia infection leading to the classic triad of uveitis,
urethritis, and arthritis) and rheumatoid arthritis are both types of inflammatory arthritis. The synovial fluid
WBC count in these conditions is 2000–75,000 cells/mm3. Gram stain will be negative.
• Septic arthritis, which this patient has, presents with a synovial fluid WBC count of > 100,000 cells/mm3, and
Gram stain/culture of the fluid yields the causative organism.

In this patient, what is Gram stain of a cervical swab likely


to show?
Gram-negative kidney-shaped cocci in pairs. However, the
endocervical Gram stain is insensitive and nonspecific and is best
diagnosed by nucleic acid testing (see Figure 3-36).

What antibiotic is recommended for treatment of this condition?


Ceftriaxone is a first-line treatment for gonococcal infections,
particularly if disseminated. Patients with gonorrhea have a high
risk of co-infection with Chlamydia trachomatis. Therefore, patients
are also empirically treated for Chlamydia with doxycycline or
azithromycin because chlamydia is the most prevalent STD. If
just gonorrhea is positive, the CDC recommends dual coverage
ceftriaxone and azithromycin. If only the chlamydia is positive, then FIGURE 3-36. Neisseria gonorrhoeae in a cervical
azithromycin or doxycycline only is used. Azithromycin can be added smear using the Gram-stain technique. (Repro-
not only for the coverage of chlamydia, but it can be used because of duced courtesy of the Centers for Disease Control
the growing resistance of gonorrhea to cephalosporins. and Prevention/Joe Miller.)

If not treated early, what is a serious potential gynecologic complication of this condition?
If the infection persists, it can develop into pelvic inflammatory disease. The bacteria can ascend to the uterus,
fallopian tubes, and ovaries, which can cause endometritis, salpingitis, oophoritis, and tubo-ovarian abscesses. The
infection and subsequent scarring can reduce the patient’s fertility, as oocytes are unable to travel through the
scarred uterine tubes. In addition, untreated infection increases the risk of ectopic tubular pregnancy. In advanced
stages, fibrotic adhesions between the fallopian tubes, uterus, and liver can occur in a condition known as Fitz-
Hugh–Curtis syndrome. Empirical treatment for both gonorrhea and chlamydia is important because they both
can cause infertility, so both are covered in treatment modalities, even if testing is negative for one.

03_USMLE-STEP1_ch03_039-088.indd 77 9/17/18 3:31 PM


78 CHAPTER 3 Microbiology

CASE 39
An 18-year-old college freshman is brought to the university health center by his dormitory roommate because he
was delirious and becoming less responsive. The patient’s roommate says that he has had 2 days of fever, several
episodes of vomiting, and joint and muscle pain. The patient’s temperature is 38.9°C (102°F). Physical examination
reveals a petechial rash on the lower extremities and photophobia; both Kernig and Brudzinski signs are positive.

What is the most likely diagnosis, and what other organ can be affected?
This patient likely has Neisseria meningitidis meningitis, a gram-negative,
kidney-shaped diplococci infection. The adrenal glands can be affected,
which is called Waterhouse-Friderichsen. There is hemorrhage into the
adrenal glands. Figure 3-37 is a Gram stain showing the diplococcal bacteria
of the Neisseria meningitidis.

FIGURE 3-37. Gram stain shows Neisseria


meningitidis. (Reproduced courtesy of the
Centers for Disease Control and Prevention/
Dr. Brodsky.)

What test can confirm the diagnosis?


Culture of meningococcus from cerebrospinal fluid (CSF) or blood. Lumbar puncture shows an elevated WBC count
(with mainly polymorphonuclear cells), decreased glucose levels, and normal to high protein levels. Bacteria are
visible on Gram stain. This pathogen can also be cultured on Thayer-Martin media (chocolate agar with antibiotics
to kill competing bacteria). N meningitidis is oxidase positive and ferments both maltose and glucose.

What are Kernig and Brudzinski signs?


Kernig sign is considered positive when the patient resists straightening of the knee joint while the hip is flexed.
Brudzinski sign is considered positive when the patient, while supine, lifts the legs when the examiner flexes the
neck. Neither test is very sensitive, but a positive result is suggestive of meningeal irritation either by blood or
inflammation.

What are the most important virulence factors and toxins of this microorganism?
N meningitidis has thin protrusions called pili that help it attach to nasopharyngeal epithelial cells. Once attached
to the nasopharynx, the pathogen secretes IgA protease to neutralize the predominant antibody idiotype found
on mucous membranes. N meningitidis also features a capsule that protects it from other host defenses such as
complement and phagocytosis. As with all gram-negative bacteria, N meningitidis has lipopolysaccharide, a potent
endotoxin, in its cell wall.

What is the treatment for this condition?


Intravenous penicillin G or ceftriaxone must be administered immediately. Bacterial meningitis can be fatal within
hours, so empiric antimicrobial treatment must be started before culture results or other confirmatory tests
become available. Rifampin is recommended as prophylactic treatment for contacts of the patient, especially if
they are immunocompromised or have not been vaccinated. Contacts can include those sitting on a plane nearby
and those intubating the patient.

03_USMLE-STEP1_ch03_039-088.indd 78 9/17/18 3:31 PM


Microbiology CHAPTER 3 79

CASE 40
A 4-year-old boy is brought to the pediatrician because
of perianal itching, which is worse at night. He attends
preschool during the day, where he shares toys and play
areas with other children. The patient’s mother recalls her
son playing with another child who had been “scratching
his backside” and wonders if there is a connection. The
doctor performs a Scotch tape test which reveals the
findings below (see Figure 3-38).

FIGURE 3-38. (Reproduced with permission from Zahariou A,


et al. J Med Case Rep. 2007;1:137.)

What is the most likely diagnosis?


Pinworm infection caused by Enterobius vermicularis, a nematode (roundworm).

What is the life cycle of this organism?


After fecal-oral transmission, the eggs hatch in the small intestine. Adults mature in the ileum and large intestine
and mate in the colon. Females exit the rectum at night to lay eggs in the perianal area. This irritates the perianal
area, inducing the host to scratch. Scratching transfers eggs onto hands, greatly increasing the likelihood of
transmission to another host and reinfection of the original host.

What sequelae may this infection have?


Intense scratching may compromise the integrity of the perianal skin, predisposing the patient to dermatitis or
folliculitis due to dermal invasion of fecal bacteria.

What test can help confirm the diagnosis?


The adhesive or “Scotch” tape test (result shown in FIgure 3-38). The physician places adhesive tape over the
perianal area and then removes and examines the tape. The presence of eggs under light microscopy indicates
pinworm infection.

What are the treatments for this condition?


Mebendazole or albendazole is first-line therapy. Pyrantel pamoate can also be helpful. Individual cases are easily
treated, but institutional outbreaks (such as in preschool) are more difficult to treat because undertreated patients
quickly reinfect others who had already been cured.

03_USMLE-STEP1_ch03_039-088.indd 79 9/17/18 3:31 PM


80 CHAPTER 3 Microbiology

CASE 41
A 10-year-old boy is camping with his family in the Adirondack Mountains when he is bitten on the leg by a raccoon.
The animal was not provoked by the boy but attacked him unexpectedly. His family brings the boy to the nearest ED.

What condition is this boy at risk of contracting?


Rabies. If left untreated, rabies results in a nearly 100% mortality rate. Rabies causes only a few deaths per year
in the United States but is significantly more dangerous in countries with unvaccinated domestic animals. For
example, rabies-infected dog bites cause tens of thousands of deaths each year in India.

What is the morphology of the pathogen that causes this condition?


Rabies is caused by a rhabdovirus, a single-stranded RNA virus
enveloped by a bullet-shaped capsid, covered by glycoprotein
“spikes” (see Figure 3-39). The spikes bind to acetylcholine
receptors, a property that may contribute to its virulence.

FIGURE 3-39. Electron micrograph of rabies virus.


(Reproduced with permission from USMLE-Rx.com.)

What is the pathogenesis of this condition?


Animals transfer the virus to humans through bites that inoculate the host. The virus remains local for a period of
days to months, then binds to acetylcholine receptors on neurons and travels to the central nervous system (CNS),
utilizing axonal retrograde transport mechanisms. In the CNS, the virus infects neurons, including Ammon horn
cells of the hippocampus. Rabies carries a significant person-to-person transmission risk through bites or mucous
membrane exposure.

What signs and symptoms are associated with this condition?


Spasms of the pharyngeal muscles cause dysphagia, which leads to painful swallowing and accidental aspiration.
This often leads to hydrophobia, a fear of water. Excess autonomic stimulation can cause hypersalivation. The
buildup of saliva accounts for the apparent “foaming at the mouth.” As rabies multiplies in the CNS, nonfocal
neurological symptoms result, including confusion, agitation, hallucinations, and photophobia. Focal neurological
deficits include cranial nerve palsies. Encephalitis can give rise to seizures, and inflammatory edema eventually
leads to coma, then death.

How is the condition diagnosed and treated?


Identifying cytoplasmic inclusions called Negri bodies in tissues obtained from brain biopsy, polymerase chain
reaction for viral RNA, and serology are diagnostic. In the event of a bite by an infected animal, the human diploid
cell vaccine (a live attenuated virus) is administered. Additionally, human rabies immune globulin (IgG) is used
to confer passive immunity to the patient. Both treatments are effective only in the lag period between the bite
and onset of symptoms. Once symptoms appear, there is no effective treatment.
Microbiology CHAPTER 3 81

CASE 42
A 30-year-old man who recently joined a gym complains
of itching between his toes. Physical examination reveals
pustules on the fingers of both hands and white macerated
tissue between the toes (see Figure 3-40). The patient says the
pustules have been itchy and appeared about a week after the
itching between the toes began.

FIGURE 3-40. (Reproduced courtesy of the Centers for


Disease Control and Prevention/Dr. Lucille K. Georg.)

What is the most likely diagnosis?


Ringworm (tinea pedis) infection. Although caused by the same group of fungi, each dermatophytosis is named
after the region of the body that it infects: tinea cruris (perineum and creases of inner thigh; “jock itch”), tinea pedis
(“athlete’s foot,” see Figure 3-40), tinea capitis (scalp), tinea unguium (nails), and tinea corporis (body).

What three microorganisms are often responsible for this presentation?


Microsporum, Trichophyton, and Epidermophyton are three filamentous fungi that cause dermatophytosis.

How is the microorganism transmitted?


After contact with an infected host, the keratinized epithelium of warm, moist skin is colonized. The infection
expands radially and is characterized by curvy (wormlike) circular borders. Thus, it is termed “ringworm,” despite
the fact that the causative microorganism is actually a fungus.

What tests can help confirm the diagnosis?


Branched hyphae are observed on potassium hydroxide preparation (KOH mount). This fungus is not dimorphic. A
sample from the lesions on the patient’s feet will likely demonstrate the organism.

What is the treatment for this condition?


Topical azoles (eg, fluconazole), butenafine, terbinafine, or griseofulvin is used for treatment of tinea pedis.

03_USMLE-STEP1_ch03_039-088.indd 81 9/17/18 3:31 PM


82 CHAPTER 3 Microbiology

CASE 43
A 13-year-old boy, who had been camping in the Appalachian Mountains with his family, was brought to the ED
because of a headache, rash, and the abrupt onset of a high fever. The rash began on his palms and soles but spread
up his ankles and arms (see Figure 3-41). On physical examination he was found to have palpable purpura on his
wrists and lower legs.
FIGURE 3-41. (Reproduced cour-
tesy of the Centers for Disease
Control and Prevention.)

What infections frequently cause a rash on the palms and soles?


Infections with Rickettsia rickettsii, Treponema pallidum, and coxsackievirus A can all cause a rash in this distribution.

What is the most likely infectious agent in this case?


This patient is most likely infected with R rickettsii. The clinical course is typical for Rocky Mountain spotted fever
(RMSF), which is characterized by fever with abrupt onset and a rash that spreads from the extremities toward the
trunk (as shown in Figure 3-41). RMSF can occur throughout the United States, Canada, and Mexico and is the most
common rickettsial infection in the United States. Palpable purpura is a poor prognostic sign, as it indicates active
vasculitis and leakage of blood into the skin. The rash in syphilis and coxsackievirus is typically more gradual in
onset. However, syphilis should never be excluded because of the patient’s age, as it is occasionally seen in sexually
abused children.

How is this pathogen transmitted to humans?


R rickettsii is acquired from Dermacentor tick bites.

What other diseases are caused by these organisms?


Endemic typhus (also known as murine typhus) is caused by R typhi. This illness presents with vasculitis leading to
headache, fever, chills, and myalgia. Epidemic typhus is caused by R prowazekii and often presents with high fevers,
hypotension, and delirium. It often occurs in conditions of crowding and after disasters (eg, refugee camps). Both
diseases are caused by arthropod-borne agents: endemic typhus is transmitted by fleas, whereas epidemic typhus
is transmitted by the human body louse.

What is the pathogenesis of this disease?


R rickettsia is an obligate intracellular microbe that infects vascular endothelial cells, resulting in damage to the
endothelium and vascular injury. The prostaglandins produced by the endothelial cells play an important role
in the increased vascular permeability and inflammation. The infection also results in activation of the clotting
cascade. The host response to the infection also contributes to the disease but is secondary to the vascular damage.

What laboratory technique can be used to identify the cause of this patient’s illness?
Diagnosis is made clinically. Direct fluorescent antibody or polymerase chain reaction testing of a skin biopsy of
one of the petechial lesions will confirm the diagnosis but not early enough to guide therapy. Rocky Mountain
spotted fever antibody testing can also be used to confirm the diagnosis after recovery.

What is the treatment of choice for organisms of this class?


Tetracyclines (such as doxycycline) are most frequently used in rickettsial infections. The rickettsiae are obligate
intracellular organisms, as they require coenzyme A and nicotinamide adenine dinucleotide from the host cell;
they also lack the peptidoglycan cell wall targeted by many antibiotic classes. Tetracyclines can enter cells and act
on ribosomal targets. Tetracyclines are typically not used in children but are in the case of Rocky Mountain Spotted
Fever due to the harm of not treating the infection in comparison to the potential adverse effects, which are not as
risky or likely to occur (tooth staining).

03_USMLE-STEP1_ch03_039-088.indd 82 9/17/18 3:32 PM


Microbiology CHAPTER 3 83

CASE 44
A 59-year-old woman presents to her physician with prominent
scattered erythematous papules (see Figure 3-42) on the right side
of her forehead. She says she has had a “burning” pain and general
hypersensitivity in that area for the past 2 days. On review of systems,
she denies headaches, mental status changes, or recent infections.
Neurological examination indicates that her pain is localized to the
right supraorbital area.

Which nerve relays the painful sensation in this patient?


Sensory information from the face is relayed by the trigeminal nerve
to the ventral posterolateral nucleus of the thalamus. The trigeminal
nerve, as the name suggests, has three main branches: V1, V2, and FIGURE 3-42. (Reproduced with permission from
Kalogeropoulos CD, et al. Med Hypothesis Discov
V3. The V1 sensory branch covers most the forehead, supraorbital,
Innov Ophthalmol. 2015;4(4):142-156.)
and nasal bridge areas. V2 is termed the maxillary branch and covers
the area of the maxillary bone as well as the alar areas of the nose
(V2 comes out at the infraorbital foramen to the skull, high yield).
V3 covers the mandibular portion of the face and so it is named.
Specifically, the area described by the patient is the left V1 dermatome.

What infectious agent may be responsible for this woman’s pain?


This woman is suffering from herpes zoster, or “shingles,” a late complication of prior infection with varicella zoster
virus, one of the herpes viruses.

What are the characteristic distribution patterns of this condition?


The virus remains latent in the ganglia of sensory nerves after the primary infection. When reactivation occurs
(usually under conditions such as high stress or immunosuppression), virions are transported to the dermatome
innervated by this sensory nerve using axonal transport mechanisms. Therefore, the characteristic shingles rash is
always confined to a dermatome and does not cross the midline. In Figure 3-42, note that the rash occurs along the
distribution of V1 on the left side of the face.

What are the likely findings on histologic examination of these vesicles?


A Tzanck smear is a historical assay that may demonstrate multinucleate giant cells, which are typical for infection
with varicella and herpes simplex viruses. This test is rarely used for diagnosis, having been superseded by more
specific molecular tests such as polymerase chain reaction or a direct antibody immunoassay. Further, shingles is
more of a clinical diagnosis as the classic rash and symptoms of hypersensitivity and pain is enough to provide a
diagnosis and treatment.

What are the treatments for this condition?


Acyclovir is activated by viral thymidine kinase to inhibit viral DNA polymerase. Valacyclovir and famciclovir have a
similar mechanism and longer half-lives. These drugs target herpesviruses and may provide relief, speed recovery,
and prevent postherpetic neuralgia. The drug doses for varicella are substantially higher than for herpes simplex
virus. Primary infection with varicella and zoster can be prevented by vaccination. A zoster vaccine is available for
the elderly population.

What are the advantages of a live attenuated vaccine? Table 3-2. Live vs Killed Vaccines
Three general types of vaccinations are available: live attenuated,
Live attenuated vaccines Killed vaccines
killed, or passive. Passive vaccination is the transfer of antibodies
and is limited by antibody half-life in the bloodstream. A killed Smallpox Rabies
vaccine can stimulate only a humoral response as the virus Yellow Fever Influenza
cannot replicate within cells; therefore, no viral proteins can be Varicella Zoster Polio (Salk)
displayed on major histocompatibility complex receptors. Live Sabin’s Polio Hepatitis A
attenuated virus vaccine stimulates both arms of the immune
MMR
system (humoral and cell mediated) as it leads to limited viral
replication. Live attenuated vaccines should be avoided in Helpful mnemonic for killed vaccines:
pregnancy as only the humoral arm of the mother’s immune RIP Always: Rabies, Influenza, Polio (Salk), Hep A.
system can protect the fetus across the placental barrier.
Table 3-2 provides examples of types of vaccines.

03_USMLE-STEP1_ch03_039-088.indd 83 9/17/18 3:32 PM


84 CHAPTER 3 Microbiology

CASE 45
A 36-year-old woman from Alabama presents with diffuse abdominal pain and diarrhea of 3 days’ duration. She
denies nausea, vomiting, or fever. She has no sick contacts or significant travel history. A complete blood count
shows eosinophilia. A stool sample reveals larvae. On further questioning, she describes that she frequently gardens
in her backyard while barefoot.

What is the most likely diagnosis?


Strongyloidiasis, caused by Strongyloides stercoralis, a nematode (roundworm).

What is the life cycle of this parasite?


Larvae in the soil penetrate the skin, usually the sole of the foot (fecal-cutaneous transmission). Local itching at
the entry site promotes scratching, which aids larval entry into the bloodstream. Once in the blood, the larvae
settle in the respiratory tree and travel up the trachea into the pharynx to be swallowed. They enter the small
intestine, where the larvae mature into adults. Female adults invade the intestinal wall and lay eggs. During
passage through the gastrointestinal tract, the eggs hatch into larvae. Most of these larvae pass with the stool
and can continue the life cycle in the soil; some, however, directly penetrate the colonic wall or perianal skin and,
uniquely, continue the life cycle within the original host. These continually migrating parasites are responsible for
the eosinophilia commonly seen in chronic Strongyloides infections.

What other two organisms demonstrate the same route of transmission in humans?
Necator americanus (New World hookworm), and Ancylostoma duodenale (Old World hookworm).

What is the pathogenesis of hyperinfection syndrome?


Hyperinfection syndrome, caused by an uncontrolled autoinfection, can increase parasitic burden and widely
disseminated disease. This is more common in individuals with defective eosinophil function, such as patients
treated with steroids or cytotoxic chemotherapeutic agents that cause granulocytopenia.

What tests can help confirm the diagnosis?


Stool sample reveals larvae (not eggs as in hookworm infection). Blood samples reveal eosinophilia. Strongyloides
antibody testing can also be valuable in patients with unexplained eosinophilia, as larvae are not always
detectable in the stool.

What are the treatments for this condition?


Ivermectin or thiabendazole. Due to the intestinal nature of this organism, it can interfere with the normal
absorption role of the small intestine. Specifically, Strongyloides can cause a B12 deficiency, which will manifest as a
macrocytic anemia. Replenishment of B12 will be helpful in these circumstances.

03_USMLE-STEP1_ch03_039-088.indd 84 9/17/18 3:32 PM


Microbiology CHAPTER 3 85

CASE 46
A 52-year-old man from Michigan presents with worsening cough,
fever, chills, and pleuritic chest pain. He was diagnosed with community-
acquired pneumonia at a hospital but seeks a second opinion. He recently
developed multiple ulcerated sores on his skin, which began as pimple-
like lesions. X-ray of the chest reveals segmental consolidation. Biopsy of
a skin lesion reveals big, broad-based, budding yeasts (see Figure 3-43).

What is the most likely diagnosis? FIGURE 3-43. (Reproduced courtesy of


Blastomycosis, one of the systemic mycoses. Dr. Yale Rosen.)

Which diagnostic tests can differentiate between the systemic mycoses?


Systemic mycoses are caused by dimorphic fungi, which grow as molds in the cold (eg, in the soil) and as yeast at
higher temperatures (eg, in tissues at 37°C [98.6°F]), as shown in Figure 3-43. The exception is coccidioidomycosis,
which is a spherule in tissue. Therefore, growing cultures on Sabouraud agar at multiple temperatures aids in the
diagnosis. In addition, a tissue biopsy revealing broad-based budding yeast is diagnostic for blastomycosis. Tissue
biopsy demonstrating yeast cells within macrophages is diagnostic of histoplasmosis. A biopsy showing “captain’s
wheel” morphology of budding yeast is diagnostic of paracoccidioidomycosis. Serologic testing for antifungal
antibodies is also useful in some patients, and a histoplasma antigen test is the diagnostic test of choice for systemic
(but not localized) Histoplasma infections.

What are the typical x-ray findings?


These diseases can mimic tuberculosis, forming granulomas, which appear as small calcium deposits on x-ray.

What is the treatment for this condition?


Systemic infection is treated with itraconazole or amphotericin B.

What are the other types of systemic mycoses? Where do they infect? (See Table 3-3.)
Table 3-3. Types of Systemic Mycoses
Name of fungus Location Important characteristics
Coccidiomycosis Southwestern United States Can cause pneumonia and meningitis with dissemination to
bone (arthralgia) and skin (erythema nodosum)
Increased rate of infection after earthquakes
Spherule (much larger than RBC) is filled with endospores

A
Histoplasmosis Mississippi and Ohio River Causes mainly pneumonia in infected individuals
Valleys (United States) Infects macrophages (invasive) and is smaller than RBCs
Caused by bird or bat droppings

B
Blastomycosis East of the Mississippi River Causes inflammatory lung disease and can disseminate to
(United States) skin and bone
Typically forms granulomatous nodules
Has broad base budding (same size as RBCs)

C
Paracoccidioidomycosis Latin America Characteristically looks like a “captain’s wheel”
Look for travel to Latin America

D
(Images reproduced courtesy of the US Department of Health and Human Services and authors Dr. D.T. McClenan [histoplasmosis], Dr. Libero
Ajello [blastomycosis], and Dr. Lucille K. Georg [paracoccidioidomycosis].)

03_USMLE-STEP1_ch03_039-088.indd 85 9/17/18 3:32 PM


86 CHAPTER 3 Microbiology

CASE 47
A 54-year-old man with HIV infection presents to the ED after suffering a grand mal seizure. He has no known
personal or family history of seizures. He is afebrile, and his vital signs are stable. Funduscopic examination reveals
yellow cotton-like lesions on his retina. Findings on physical examination are otherwise unremarkable. CT scan of
the head demonstrates multiple ring-enhancing lesions in the cerebral cortex. Laboratory findings reveal a CD4+
cell count of 53 cells/mm3.

What is the most likely cause of this patient’s seizure?


Toxoplasma gondii infection.

How did this patient likely become infected with this microorganism?
It is likely that this man (like most individuals) has been latently infected with this protozoan for many years.
However, his immunocompromised status has resulted in disease reactivation. Humans are most often infected
by ingestion of cysts in undercooked meat or by fecal-oral transmission of cat feces. Cats may shed the protozoan,
and pregnant women are discouraged from cleaning the litter box as inhaled aerosolized particles are sufficient to
cause infection.

Why is this condition dangerous in pregnancy?


Primary infection with T gondii in a pregnant woman can allow parasites to cross the placenta. This leads to
congenital problems in the newborn, including intellectual disability, microcephaly, chorioretinitis, intracerebral
calcifications, and blindness. It is one of the TORCHeS infections (Toxoplasmosis, Other infections, Rubella,
Cytomegalovirus, Herpes simplex virus, Syphilis).

Given this patient’s ring-enhancing lesions on CT scan, what other conditions should be included in the
differential diagnosis?
This patient is also at an increased risk of lymphoma, cryptococcosis, and tuberculosis, all of which appear as
ring-enhancing lesions on CT scan (although less likely to cause multiple lesions) and can also cause seizures.

What is the treatment for this condition?


First-line treatment is a regimen of pyrimethamine and sulfadiazine.

What other opportunistic infections occur with HIV, and at what viral load are they likely to occur?
(See Table 3-4.)

Table 3-4. Common HIV Infections and Viral Loads


Viral load (cells/μL) Infections
< 200 Pneumocystitis jiroveci
TB
Coccidiodomycosis
Candidiasis
< 100 Toxoplasmosis
Cryptococcus
< 50 CMV
MAC
PML

03_USMLE-STEP1_ch03_039-088.indd 86 9/17/18 3:32 PM


Microbiology CHAPTER 3 87

CASE 48
A 17-year-old boy who recently immigrated to the United States from India
presents to the ED with complaints of spiking fevers, weight loss, lethargy, and a
large skin ulcer. He has not previously received any vaccinations. He reports that
he has been generally healthy until he immigrated and has been having these
symptoms. He is afraid that he has hepatitis or malaria because he had siblings
die from each. On examination he is cachectic with a gray skin tone, and he is
found to have pronounced splenomegaly and mild hepatomegaly. Laboratory
tests reveal pancytopenia. Microscopic examination of a bone marrow aspirate
reveals parasites in the histiocytes (see Figure 3-44).

FIGURE 3-44. (Reproduced with per-


mission from the Centers for Disease
Control and Prevention.)

What is the most likely diagnosis?


This patient is suffering from kala azar, or visceral leishmaniasis. Visceral leishmaniasis is caused by the protozoan
Leishmania donovani and is characterized by spiking fevers, hepatosplenomegaly, and pancytopenia.

Is the organism found in the amastigote or promastigote form in the infected human?
The form found in the human host is the amastigote, which is small and round and has a flagellum that is difficult
to visualize. The prominently flagellated form of the parasite is found in the insect vector and is known as the
promastigote.

What is the vector of this pathogen?


Humans are infected with Leishmania donovani through the bite of a sandfly. It can also be transmitted by
intravenous drug use or blood transfusion. There is especially high prevalence in India and Africa.

On the blood smear, some macrophages contain basophilic inclusions. What are these inclusions?
These inclusions are called Donovan bodies and consist of the amastigote form of the parasite (see Figure 3-44).

What is the treatment for this condition?


Treatment is sodium stibogluconate or pentamidine.

What diseases are caused by other blood-borne flagellates?


Trypanosomes are another flagellated parasite that can be found in the blood. Trypanosoma cruzi is transmitted
by the reduviid bug found in South America and causes Chagas disease. T gambiense and T rhodesiense are
transmitted by the tsetse fly and are the cause of African sleeping sickness.

03_USMLE-STEP1_ch03_039-088.indd 87 9/17/18 3:32 PM


88 CHAPTER 3 Microbiology

CASE 49
A 45-year-old man visiting rural Brazil develops fever, headache, pain in his knees and back, and nausea and vomiting.
After 3 days these symptoms resolve, and he decides not to seek medical help. However, 2 days later the symptoms
return, and he develops epigastric pain and yellowing of his skin. His vomitus is now dark in color.

What is the most likely diagnosis?


Yellow fever is endemic in South America and parts of Africa. It is characterized by an initial febrile illness, during
which time serum aspartate aminotransferase and alanine aminotransferase levels begin to rise, followed by a
remission of symptoms. Approximately 15% of infected patients experience a return of symptoms 2–3 days later,
developing further liver dysfunction (resulting in jaundice and coagulopathy), renal damage, and myocardial
damage.

What type of organism is responsible for this patient’s condition?


The yellow fever virus is a flavivirus. These viruses have positive, single-stranded RNA genomes and icosahedral,
enveloped capsids.

What are the most likely liver biopsy findings?


The characteristic finding on liver biopsy is midzone hepatocellular death, with sparing of cells bordering the
central vein and portal tracts. Councilman bodies are found in the affected hepatocytes. These are eosinophilic
inclusions that represent condensed chromatin. Typically, there is no inflammatory response. Liver biopsies are
usually not done because of their concomitant coagulopathy. Specifically, yellow fever can infect Zone 2 of the
liver.

Enzyme-linked immunosorbent assay (ELISA) may be useful in confirming the diagnosis by detecting
antibody to the virus. How does ELISA work?
ELISA is a technique often used for serologic testing. It involves coating the surface with the desired antigen (in
this case, yellow fever viral particles) and then placing the patient’s serum on the surface, followed by a secondary
antibody (antihuman antibody) that is linked to an enzyme. If the patient’s serum has antibody to the antigen,
the secondary antibody will bind. The linked enzyme can be detected by a reaction that produces an alteration in
color with a colorimetric agent (eg, horseradish peroxidise). The color change can be quantified by spectroscopy.
Detection of antibody to yellow fever virus in a patient with exposure can support a clinical diagnosis of the
disease.

What are some other infections commonly found in Africa?


Dengue fever is a flavivirus (RNA, enveloped, single stranded, with icosahedral capsid) that uses an arthropod
as a vector. It classically causes high fever, headache, vomiting, arthralgias, malaise, and a skin rash. The rash is
described as a morbilliform rash with macular lesions. At its worst, dengue fever can cause a thrombocytopenia
and anemia by infecting the bone marrow, which can then develop into shock.
Another infection that may show up is West Nile virus. It is also a flavivirus and uses the mosquito as its vector (like
Dengue). It is characterized by headache, nausea, vomiting, and may progress to encephalitis and viral meningitis.
The prominent finding of West Nile virus, if left untreated, can cause muscle weakness and flaccid paralysis. Its
reservoir is known to be birds of the region.

03_USMLE-STEP1_ch03_039-088.indd 88 9/17/18 3:32 PM


Chapter 4

Microbiology

71
High-Yield Principles
72 Section I: General Principles  •  Questions

Q u e st i o n s

1. A 30-year-old sexually active woman presents (A)


Blastomyces dermatitidis
with a painful vesicle on her external genita- (B)
Coccidioides immitis
lia and bilateral inguinal lymphadenopathy. A (C)
Cryptococcus neoformans
Tzanck smear from the vesicle is negative, and (D)
Histoplasma capsulatum
a Venereal Disease Research Laboratory assay (E)
Paracoccidioides brasiliensis
is also negative. Which of the following medi-
cations would be most appropriate for this pa- 3. A 17-year-old boy visits his physician with com-
tient? plaints of recurrent bouts of dizziness, palpita-
tions, and joint pain. He went on a summer
(A) Acyclovir
hiking trip in eastern Massachusetts about six
(B) Ceftriaxone
months ago but does not recall getting a tick
(C) Foscarnet
Microbiology

bite and notes no rashes. The ECG is shown


(D) Ribavirin
in the image. What is the most likely diagnosis
(E) Vancomycin
of this patient’s symptoms?
2. A 51-year-old man living near St. Louis, Mis- (A) Brugada syndrome
souri presents to his primary care physician (B) Chagas disease
with recent-onset productive cough, pleuritic (C) Hypertrophic cardiomyopathy
chest pain, and a fever of 39.1°C (102.4°F). (D) Lyme disease
He recently returned from a business trip to (E) Third-degree heart block
Phoenix, Arizona. A sample of the man’s puru-
lent sputum is sent for analysis, which reveals 4. A 30-year-old woman complains of a nonpro-
yeast cells up to 15 µm in diameter. The pa- ductive cough that has developed over the past
thologist is able to identify several dividing 10 days. She reports feeling achy, and having
yeast organisms. Direct fluorescent antibody a sore throat and headaches. X-ray of the chest
staining (see image) is notable for large, broad- demonstrates patchy bilateral interstitial infil-
based budding from mother cells. What fungal trates. After work-up, the patient is diagnosed
species is responsible for this man’s illness? with Mycoplasma pneumoniae pneumonia.
Which of the following is associated with the
causative organism?
(A) Growth on buffered charcoal yeast extract
media
(B) IgM cold agglutinins
(C) Phyocyanin production
(D) Polysaccharide capsule
(E) Reticulate bodies

Courtesy of Dr. William Kaplan, Centers for Disease Control


and Prevention.
High-Yield Principles
Chapter 4: Microbiology  •  Questions 73

5. A homeless 37-year-old woman with HIV in-


fection comes to the clinic with a four-week
history of worsening hemiparesis, visual field
deficits, and cognitive impairment. The pa-
tient’s CD4+ count is 22/mm3. MRI shows
several hyperintensities on T2-weighted im-
ages that do not enhance with contrast and are
not surrounded by edema. A lumbar puncture
shows a normal opening pressure, and cerebro-
spinal fluid analysis shows a mildly elevated
protein level and the presence of myelin basic
protein, with a mild mononuclear pleocytosis.
Which of the following entities is most likely Courtesy of the Centers for Disease Control and Prevention.
responsible for this patient’s clinical picture?

Microbiology
(A) Cortical tuberculoma
(B) Cytomegalovirus encephalitis (A) Inhibition of cell wall synthesis
(C) JC virus (B) Inhibition of DNA polymerase
(D) Primary central nervous system lymphoma (C) Inhibition of genome uncoating
(E) Toxoplasmosis (D) Inhibition of nucleoside reverse transcrip-
tase
6. A family who recently emigrated from Roma- (E) Inhibition of protein synthesis
nia brings their 7-year-old child to the pedia-
trician with complaints of conjunctivitis and 8. A 38-year-old man comes to the emergency
periorbital swelling. The child has had cough- department complaining of cyclic fevers and
ing with a runny nose and high fever for three headaches. The fevers began about one week
days. Small lesions with blue-white centers are ago; two weeks ago the patient returned from
seen in his oral cavity. Which of the following a trip to Africa. Physical examination reveals
is the most likely cause of this child’s symp- hepatosplenomegaly. Imaging of the brain
toms? shows signs of significant cerebral involve-
ment. Which of the following parasites most
(A) Diphtheria likely caused this patient’s symptoms?
(B) Pertussis
(C) Roseola (A)
Plasmodium falciparum
(D) Rubella (B)
Plasmodium malariae
(E) Rubeola (C)
Plasmodium ovale
(D)
Plasmodium vivax
7. A 5-year-old girl is brought to her pediatrician (E)
Plasmodium knowlesi
because of an eight-day history of a painful
rash confined to her flank. Physical examina-
tion reveals the crusted lesion shown in the
image. Which of the following describes the
mechanism of action of the treatment for this
lesion?
High-Yield Principles
74 Section I: General Principles  •  Questions

9. A 15-year-old boy presents to the pediatrician 11. Influenza virus type A usually produces a mild,
with a two-day history of fever and headache. self-limited febrile illness in the general popu-
The patient is unable to touch his chin to his lation. However, worldwide epidemics have
chest when asked to do so. He also asks that occurred at different times in history due to
the lights in the room be turned down. In ad- rapid changes in viral genetic makeup. Which
dition to performing a lumbar puncture to ob- of the following is the most important reason
tain cerebrospinal fluid (CSF), the physician why these sporadic worldwide epidemics oc-
begins empiric treatment. The CSF is sent for cur?
analysis and culture. The patient’s condition
(A) Antigenic drift
improves over the next week. In the meantime,
(B) Antigenic shift
bacterial and fungal culture results are nega-
(C) Hemagglutinin develops the ability to de-
tive. Which of the following is the most likely
stroy a component of mucin, becoming
result of the CSF analysis?
more infectious
(D) Neuraminidase develops the ability to at-
Microbiology

Choice Pressure Lymphocyte Protein Sugar


tach to sialic acid receptors, becoming
count more infectious
A normal normal normal normal
(E) RBCs agglutinate with certain strains
B normal normal normal
C normal or normal normal 12. A 66-year-old woman who recently emigrated
D from Mexico comes to the physician be-
E cause she has begun to have seizures. A test
for anticysticercal antibodies is positive. A T1
Reproduced, with permission, from USMLERx.com. weighted, non-contrast MRI of the head is
shown in the image below. Which of the fol-
lowing organs or tissues is most likely to have
(A) A
similar lesions?
(B) B
(C) C
(D) D
(E) E

10. Oncogenic viruses act through a variety of


mechanisms. Some introduce oncogenes di-
rectly into host cells, while others force cells to
repeatedly undergo cycles of proliferation that
eventually become unregulated. Still others in-
troduce oncogenic potential by manipulating
chromosomal structure through deletions or
translocations. Which of the following viruses
causes neoplasia by inactivating tumor sup-
pressor genes such as p53 and Rb?
(A) Epstein-Barr virus
(B) Hepatitis C virus Courtesy of Dr. Per-Lennart Westesson, University of Roches-
(C) Human immunodeficiency virus ter Medical Center.
(D) Human papillomavirus
(E) Human T-cell lymphotropic virus type 1
High-Yield Principles
Chapter 4: Microbiology  •  Questions 75

(A) Bladder 15. A 27-year-old woman presents to her physi-


(B) Bone cian complaining of fever, chills, and flu-like
(C) Kidney cortex symptoms. A sputum culture at 25°C (77°F) is
(D) Skeletal muscle shown in the image. Which of the following is
(E) Small bowel most likely to be elicited on further question-
ing?
13. A young girl living in rural New Mexico is
brought to her pediatrician with complaints
of fever, cough, and fatigue for the past two
weeks. The physician notices that the patient
is having intermittent bouts of many coughs in
a single breath, followed by a deep inspiration.
The parents report that this pattern of cough-
ing had started in the past two days. The phy-

Microbiology
sician informs them that their daughter will
most likely recover with only supportive care.
However, she wants to confirm his diagnosis. A
throat swab is sent for culture for a specific or-
ganism. Which of the following culture media
will be used?
(A) Bordet-Gengou medium
(B) Charcoal yeast extract with iron and cyste- Courtesy of Dr. Hardin, Centers for Disease Control and
Prevention.
ine
(C) Chocolate agar with factors V and X
(D) Loeffler medium (A) A recent trip to Namibia
(E) Thayer-Martin medium (B) A recent trip to New Mexico
(C) A recent trip to Ohio
14. A worried mother brings her infant to the
(D) Recent hiking in wooded areas
emergency department because he appears to
(E) Recent work in her large rose garden
be unable to swallow and continues to choke
on his formula. On physical examination, the 16. At birth, a newborn is noted to be unrespon-
physician notes generalized muscle weakness. sive to verbal stimulation from the doctors,
On further questioning, the patient’s mother nurses, and his parents. A routine physical ex-
says that she recently started sweetening the amination of the child reveals a split S2 heart
baby’s food with honey. Which of the follow- sound with an accentuated P2 component.
ing is a characteristic of the organism most The newborn has bounding pulses with a wide
likely responsible for this infant’s symptoms? pulse pressure. After a week the newborn’s par-
(A) Production of IgA protease ents notice that he has developed shortness of
(B) Production of exotoxin A breath and respiratory distress. What pathogen
(C) Production of lecithinase did the mother contract during her pregnancy
(D) Production of lipopolysaccharide that could explain the newborn’s current con-
(E) Production of spores that can only be dition?
killed by autoclaving (A) Cytomegalovirus
(B) Herpes simplex virus 2
(C) Rubivirus
(D) Toxoplasma gondii
(E) Treponema pallidum
High-Yield Principles
76 Section I: General Principles  •  Questions

17. A 45-year-old man presents to the clinic com- (C) Erythromycin


plaining of several weeks of vague abdominal (D) Prompt replacement of water and electro-
discomfort and early satiety. The physician or- lytes; tetracyclines shorten the disease’s
ders upper GI endoscopy as part of his workup. course
During the study, mucosal rigidity and hyper- (E) Supportive care only, without antibiotics
plasia are seen in the stomach, and a biopsy
is taken from the affected area. Microscopic 19. A 65-year-old man with a history of viral hepa-
analysis of the biopsy specimen shows sheets of titis presents to his primary care physician with
atypical lymphocytes. The organism believed complaints of early satiety, a 4.5-kg (10-lb)
to be associated with this condition is best de- weight loss over three months, upper abdomi-
scribed by which set of laboratory results? nal pain, and yellowing of his eyes. The pa-
tient says he has lived in Rochester, New York
(upstate) for his entire life, has not traveled
Choice Urease Catalase Oxidase outside of the country, and received two blood
Microbiology

A negative negative negative


transfusions in the early 1970s following an au-
tomobile accident. Work-up reveals extensive
B negative negative positive macronodular cirrhosis with a 2-×2-cm mass
C negative positive
in his liver. Which of the following viral in-
negative
fections is most likely to result in this patient’s
D negative positive positive current presentation?
E positive negative negative (A) Cytomegalovirus
F
(B) Hepatitis A
positive positive negative
(C) Hepatitis C
G positive positive positive (D) Hepatitis E
(E) HIV
Reproduced, with permission, from USMLERx.com.
20. A neonate with purulent umbilical discharge
for one day presents with fever, irritability, and
(A) A diffuse flushing. One day later she is covered
(B) B in large, fluid-filled blisters that rupture easily,
(C) C leaving raw red areas beneath. Blood cultures
(D) D are taken, which within 24 hours grow an or-
(E) E ganism that is subsequently Gram stained with
(F) F the results shown below. The skin symptoms
(G) G observed in this case are due to the involve-
ment of which of the following intercellular
18. A 36-year-old man comes to the physician structures?
because he is experiencing abdominal pain,
vomiting, and a non-bloody diarrhea. He last
ate chicken and rice about four hours ago at
a Chinese restaurant. He has no other symp-
toms. Which of the following treatments
should this man receive?
(A) Bismuth subsalicylate, metronidazole, and
amoxicillin
(B) Ciprofloxacin
High-Yield Principles
Chapter 4: Microbiology  •  Questions 77

Courtesy of Dr. Richard Facklam, Centers for Disease Control Courtesy of Drs. E. Arum and N. Jacobs, Centers for Disease

Microbiology
and Prevention. Control and Prevention.

(A) Desmosomes (A) Azithromycin


(B) Gap junctions (B) Ceftriaxone
(C) Hemidesmosomes (C) Fluconazole
(D) Intermediate junctions (D) Penicillin
(E) Tight junctions (E) Vancomycin

21. A 5-year-old boy develops diarrhea after eat- 23. A 47-year-old woman comes to the clinic com-
ing at a fast-food restaurant. The following plaining of fever and malaise. She reports hav-
day, his mother notices that he seems lethargic ing severe headaches associated with some
and brings him to the urgent care center. His nausea and vomiting, over the past few days.
blood pressure is 150/90 mm Hg. Laboratory Her urine has been exceptionally dark for the
tests show a hemoglobin level of 9 g/dL, plate- past few days. The patient is mildly jaundiced
let count of 40,000/mm3, and creatinine level with scleral icterus. Based on these symptoms
of 2.8 mg/dL. What is the most likely cause of the physician suspects hepatitis B and draws
this patient’s condition? blood for serologic testing for hepatitis B mark-
ers. If the patient had unprotected intercourse
(A) Campylobacter
during this infection, the presence of which of
(B) Escherichia coli
the following would be most concerning for
(C) Rotavirus
her partner?
(D) Shigella
(E) Vibrio cholerae (A) Hepatitis B e antibody
(B) Hepatitis B e antigen
22. A 32-year-old man presents to his doctor with (C) Hepatitis B surface antibody
painful urination and a purulent urethral dis- (D) IgG hepatitis B core antibody
charge. The discharge material is cultured, (E) IgM hepatitis B core antibody
and a sample from the culture is stained with
Giemsa and is shown in the image. Which of
the following is the treatment of choice for this
infection?
High-Yield Principles
78 Section I: General Principles  •  Questions

24. A 1-year-old girl presents to the emergency (C) Hageman factor


room because of a three-day history of cough- (D) C5a
ing attacks that are occasionally followed by (E) Interleukin-1
episodes of vomiting. Her parents are espe- (F) Nitric oxide
cially concerned because sometimes she be-
comes blue after an episode. She has markedly 27. A 30-year-old man from Mexico comes to the
injected sclera but no discharge. She is afe- physician because of a two-day history of a fe-
brile and has no other symptoms. Laboratory ver of 38.8º C (101.8º F), a sore throat, and
tests show a WBC count of 25,000/mm3 with shortness of breath. On physical examination,
marked lymphocytosis. A bacterial infection is the patient is found to have cervical adenopa-
diagnosed. Which of the following is a result of thy and a thick, gray, tissue-like material cover-
the exotoxin produced by the organism caus- ing his tonsils. Which of the following types of
ing this child’s symptoms? culture should be used to determine the organ-
ism infecting this patient?
(A) Blocked release of acetylcholine into the
Microbiology

synaptic cleft (A) Löwenstein-Jensen agar


(B) Formation of a pore in the plasma mem- (B) Bordet-Gengou agar
brane (C) Chocolate agar with factors V and X
(C) Inactivation of elongation factor 2, a pro- (D) Loeffler medium, blood agar, and tellurite
tein involved in translation plate
(D) Inhibition of G proteins to increase cAMP (E) Thayer-Martin media
(E) Release of lipopolysaccharide-lipid A
28. A 12-year-old boy presents to the emergency
25. A 50-year-old man develops profuse, non- room five days after returning from a camping
bloody, watery diarrhea while working as trip in Virginia. He complains of a fever, head-
an aid worker in Bangladesh. He arrived in ache, and muscle aches for several days, and
the area two days ago. A stool smear shows recently developed a rash on his wrists, palms,
no WBCs. His mucous membranes are dry, and legs that has since spread to his trunk, as
and his skin shows signs of tenting. He subse- seen in the image. Which of the following is
quently develops electrolyte abnormalities that the most appropriate treatment?
lead to cardiac and renal failure. What is the
mechanism of action of the exotoxin produced
by the most likely causative organism?
(A) Cleaves host cell rRNA
(B) Directly acts as an adenylate cyclase
(C) Inactivates elongation factor 2
(D) Permanently inactivates Gi
(E) Permanently activates Gs

26. A biotechnology company is developing a


small protein to block the cascade by which
allergens can cause shock. Patients often die
from vasodilation and massive edema; there-
fore, the new protein could lead to a life-saving
Courtesy of the Centers for Disease Control and Prevention.
drug. Which of the following molecules is the
most promising target to block the anaphylac-
tic pathway? (A) Ceftriaxone
(A) γ Interferon (B) Doxycycline
(B) IgE surface receptors
High-Yield Principles
Chapter 4: Microbiology  •  Questions 79

(C) Gentamicin
(D) Nystatin
(E) Penicillin

29. A 53-year-old obese man with poorly con-


trolled noninsulin-dependent diabetes mel-
litus presents with fever to 39.6°C (103.2°F),
jaundice, hypotension, and acute onset of right
upper quadrant pain. Right upper quadrant
imaging shows multiple gallstones and chole-
cystitis. Urgent cholecystectomy is performed,
and subsequent gall bladder fluid and blood
cultures grow aerobic, nonlactose-fermenting,
Courtesy of the Centers for Disease Control and Prevention.
oxidase-positive, gram-negative rods. Blood

Microbiology
tests show:
Hematocrit: 29% (A) Bloody and foul-smelling vaginal discharge
WBC count: 14,700/mm3 (B) Pelvic pain
Platelet count: 76,000/mm3 (C) Profuse, frothy vaginal discharge
International Normalized Ratio: 3.2 (D) Thick, white, cottage cheese-like vaginal
D-dimer: 8500 ng/mL discharge
Fibrinogen levels: low (E) Thin, gray-white, fishy-smelling vaginal
discharge
Microscopic inspection of peripheral blood
smear shows schistocytes and multiple helmet 31. A 40-year-old man goes on a camping vaca-
cells. Clinically, there is no evidence of active tion with his family. One day after swimming
bleeding. What is the most appropriate treat- in a freshwater lake near the campsite, he de-
ment for this patient’s coagulopathy? velops nausea and vomiting and starts to be-
(A) Amoxicillin have irrationally. His family takes him to the
(B) Aztreonam emergency department, where blood samples
(C) Fresh frozen plasma are taken and a spinal tap is performed. He is
(D) Vancomycin diagnosed with a rapidly progressing meningo-
(E) Vitamin K encephalitis and dies shortly thereafter. Which
of the following protozoa was most likely the
30. A 16-year-old girl complains of abnormal vagi- cause of the man’s illness?
nal discharge as well as itching, tenderness, (A)
Cryptosporidium species
and burning in the vulvovaginal area. On ex- (B)
Entamoeba histolytica
amination there is vulvar and vaginal erythema (C)
Leishmania donovani
and colpitis macularis. Results of a wet mount (D)
Naegleria fowleri
examination are shown in the image. Which (E)
Plasmodium falciparum
additional symptom would most likely be seen
in this patient?
High-Yield Principles
80 Section I: General Principles  •  Questions

32. A mother brings her 12-year-old daughter to an (C) Mosquito bites


outpatient clinic. The child complains of ach- (D) Saliva and respiratory secretions
ing pain localized to the joints of the extremi- (E) Sexual contact
ties. The mother recalls her daughter was sick
with a sore throat about a month ago, but re- 34. While hospitalized for treatment of an episode
covered completely without medical attention. of aspiration pneumonia, a 45-year-old man
The girl is admitted to the hospital for further begins to have episodes of severe non-bloody
examination and testing. A tissue biopsy is diarrhea and lower abdominal pain. Tempera-
taken, and the abnormal results are seen in the ture is 38°C (100.4°F). CT of the abdomen
image. Given the most likely diagnosis, what and pelvis with oral and intravenous contrast
finding would be expected in this patient on demonstrates marked diffuse colonic thicken-
cardiac physical examination as an adult? ing (see image). What is the pathophysiology
of the most likely cause of this patient’s condi-
tion?
Microbiology

Courtesy of Dr. Ed Uthman.

(A) A friction rub heard throughout the pre- Reproduced, with permission, from USMLERx.com.
cordium
(B) A harsh crescendo-decrescendo early sys-
tolic murmur heard at the right upper ster- (A) Directly damages the microvilli of the en-
nal border with radiation to the carotids terocytes but does not invade
(C) A late diastolic murmur heard best at the (B) Produces an exotoxin that can induce cyto-
apex kine release and cause hemolytic uremic
(D) A midsystolic click heard best at the apex syndrome
(E) An S4 gallop heard at the apex (C) Produces an exotoxin that increases the se-
cretory activity of enterocytes
33. A 23-year-old man from Kenya presents with (D) Produces an exotoxin that kills enterocytes
night sweats, fevers, oliguria, and a large sub- (E) Produces both heat-stable and labile toxins
mandibular mass. Biopsy of the mass shows an that promote secretions in the intestines
aggressive tumor with sheets of lymphocytes
staining positive for CD20, as well as very high 35. A 56-year-old man presents with sharp subster-
levels of nuclear c-myc with interspersed mac- nal pain radiating to his back and arms. The
rophages. What is the mode of transmission of patient is seated and leaning forward. He states
the virus associated with this malignancy? that the pain is less severe in this position, and
worsens when he lies down and takes a deep
(A) Blood products
breath. He recently recovered from a febrile
(B) Fecal-oral transmission
illness. On physical examination a scratchy,
High-Yield Principles
Chapter 4: Microbiology  •  Questions 81

leathery sound is heard at the left lower sternal cutaneous needle biopsy. A Gram stain of this
border. An ECG shows diffuse ST-segment el- sample is most likely to show which of the fol-
evation. Which of the following describes the lowing?
microorganism that is the most likely cause of
(A) Acid-fast bacilli
this condition?
(B) Catalase-negative, gram-positive cocci in
(A) Catalase- and coagulase-positive cocci chains
(B) Double-stranded, linear, enveloped, icosa- (C) Coagulase-negative, gram-positive cocci in
hedral DNA virus clusters
(C) Double-stranded, segmented RNA virus (D) Coagulase-positive, gram-positive cocci in
(D) Positive, single-stranded, helical RNA virus clusters
(E) Small, naked, single-stranded RNA virus (E) Oxidase-negative, gram-negative bacilli

36. A 28-year-old man complains of increasing 38. A 28-year-old woman presents to her primary
muscle weakness and numbness that began care physician complaining of a generalized

Microbiology
in his legs and feet three days ago and that body rash, especially on the inside of her wrists
now involves his arms and hands. The patient and ankles, and lesions on her genitals (see im-
reports recently experiencing a self-limited age). Physical examination reveals generalized
episode of gastroenteritis. Which organism is lymphadenopathy and a mild fever. Which of
commonly associated with this patient’s neuro- the following could be used to confirm the di-
logic symptoms? agnosis?
(A) a-Hemolytic, encapsulated, gram-positive
cocci that produce an IgA protease
(B) Comma-shaped, oxidase-positive, gram-
negative bacteria that can be grown at
42°C
(C) Non-lactose-fermenting, oxidase-positive,
gram-negative, aerobic bacilli
(D) Rod-shaped, gram-positive, spore-forming
anaerobe that produces a heat-labile toxin
(E) Spiral-shaped bacteria with axial filaments,
visualized using dark-field microscopy

37. An otherwise healthy 15-year-old boy sustains


a deep puncture wound in his left heel while Courtesy of Dr. J. Pledger, Centers for Disease Control and
playing in a junkyard. Four days later, his fa- Prevention.
ther brings him to the emergency department
because the boy has become lethargic and has
developed shaking chills. He refuses to bear (A) Culture on Thayer-Martin agar
weight on his left foot. The patient’s tempera- (B) Tzanck preparation
ture is 39.7°C (103.4°F). Physical examination (C) Venereal Disease Research Laboratory test
shows a warm, swollen, and extremely tender (D) Weil-Felix reaction
area around the puncture wound. A specimen (E) Ziehl-Neelsen stain
through uninfected tissue is obtained via per-
High-Yield Principles
82 Section I: General Principles  •  Questions

39. A 43-year-old HIV-positive man presents to his 40. An 8-day-old infant has developed fever, ir-
physician complaining of recent-onset abdomi- ritability, decreased level of consciousness,
nal pain and diarrhea, along with an increased apnea, and a full anterior fontanelle. Prenatal
level of general fatigue and occasional night laboratory tests results are not available, as the
sweats. Physical examination is significant for infant’s mother received no prenatal care and
a fever of 37.8°C (100.1°F), bilateral cervical delivered at home at 36 weeks’ gestation. What
adenopathy, and a weight loss of 4.5 kg (10 lb) routine prophylactic intrapartum antibiotic
compared to his last physician’s visit. A blood could the mother have been treated with in or-
sample is taken, which when cultured shows der to eradicate the likely cause of the infant’s
the presence of nonbranching bacilli that stain illness?
positively for Ziehl-Neelsen stain. His CD4+
(A) Fluconazole
cell count is 50/mm3. Which of the following
(B) Metronidazole
is the most worrisome for causing the patient’s
(C) Moxifloxacin
recent symptoms?
(D) Penicillin
Microbiology

(A)
Actinomyces israelli (E) Vancomycin
(B)
Mycobacterium avium-intercellulare
(C)
Mycobacterium marinum
(D)
Mycobacterium tuberculosis
(E)
Nocardia asteroides
High-Yield Principles
Chapter 4: Microbiology  •  Answers 83

An s w e r s

1. The correct answer is B. The differential diag- quired the illness and/or the morphology of
nosis of a genital ulcer in a sexually active pa- the organism. In this case, one cannot reach a
tient should include primary syphilis (though conclusion based on location. The man lives
these ulcers are usually painless), genital her- near the Mississippi River basin (where his-
pes, and chancroid. Because the Tzanck smear toplasmosis and blastomycosis are endemic),
(which looks for multinucleated giant cells and he has recently traveled to the Southwest
typical of herpes infection), is negative, as is (where coccidioidomycosis is endemic). Only
the VDRL test for syphilis, chancroid becomes paracoccidioidomycosis (endemic to Central
most likely. Chancroid is a bacterial infection and South America) can be eliminated as a
caused by Haemophilus ducreyi, which pre­ likely answer by location. Rather, the morphol-
sents typically as a painful genital ulcer with ogy of the yeast is the key to reaching the cor-

Microbiology
associated inguinal lymphadenopathy. It is typ- rect conclusion. In the lower-right portion of
ically treated with ceftriaxone given as a one- the image, an example of broad-based budding
time, 250-mg, intramuscular injection, or with can be seen, which is most consistent with
azithromycin as a single 1000-mg dose. blastomycosis. A common mnemonic regard-
ing the appearance of Blastomyces is that it is a
Answer A is incorrect. Acyclovir is an antiviral
Big, Broad-Based, Budding organism.
agent used to treat herpes infections. It is acti-
vated by viral thymidine kinase, whereupon it Answer B is incorrect. The distinguishing
inhibits the herpes viral polymerase. It can be morphologic feature of Coccidioides immitis,
used to treat herpes simplex virus types 1 and which is endemic to the southwestern United
2, varicella-zoster virus, and Epstein-Barr virus States, is its tendency to form large (up to 70
infections. µm in diameter) spherules filled with endo-
spores.
Answer C is incorrect. Foscarnet inhibits viral
DNA polymerase without the need of activa- Answer C is incorrect. Unlike the other an-
tion by thymidine kinase. It is used to treat cy- swer choices, Cryptococcus neoformans most
tomegalovirus (CMV) retinitis, but it can also commonly causes fungal meningitis (with
be used to treat acyclovir-resistant herpes sim- pneumonia as the second most common mani-
plex virus. festation) in the immunocompromised. Like
Blastomyces, it is a budding organism, but is
Answer D is incorrect. Ribavirin is used to
distinguishable morphologically by a thick
treat respiratory syncytial virus. It functions by
capsule that may be visualized with India ink
inhibiting inosine monophosphate dehydro-
stain.
genase, thus blocking the synthesis of guanine
nucleotides. Answer D is incorrect. Histoplasma capsula-
tum, which is endemic to the Mississippi and
Answer E is incorrect. Vancomycin is a bac-
Ohio River valleys, is unique among the an-
tericidal antibiotic used for multidrug-resistant
swer choices in that it can live as an intracel-
gram-positive organisms such as Staphylococ-
lular pathogen. Microscopic evaluation of a le-
cus aureus and Clostridium difficile. It func-
sion can show numerous small (1-5 µm) yeast
tions by binding to mucopeptide precursors,
forms within an individual macrophage.
preventing formation of the bacterial cell wall.
Answer E is incorrect. Paracoccidioides brasil-
2. The correct answer is A. This patient is suf- iensis, which is endemic to rural Latin Amer-
fering from a fungal pneumonia. In USMLE- ica, is often described as having a captain’s-
style questions, students are often asked to wheel or Mickey-Mouse-head appearance.
differentiate between candidate yeast species This is due to several smaller daughter cells
based on the location where the patient ac-
High-Yield Principles
84 Section I: General Principles  •  Answers

that are simultaneously budding from a single Answer C is incorrect. Hypertrophic cardio-
mother cell. myopathy is the most common cause of death
in young athletes in the United States. It is
3. The correct answer is D. Lyme disease is characterized by an asymmetric hypertrophic,
caused by infection with the spirochete Bor- nondilated left ventricle. Histopathologically,
relia burgdorferi, and is transmitted by the bite the myocardial architecture is disorganized
of the Ixodes tick. Initially, the disease presents and scarred. The typical ECG shows repolar-
with constitutional symptoms such as fever and ization changes or frank hypertrophy.
malaise, as well as a rash surrounding the bite
Answer E is incorrect. Classic third-degree
site. However, the bite site often goes unno-
heart block is “complete,” which means that
ticed, and erythema chronicum migrans is not
the atria and the ventricles beat independently
necessarily present in every case. Early dissem-
of each other, with the P waves and the QRS
inated disease presents four-six weeks after the
waves bearing no relation to one another. Se-
initial infection and is characterized by cardiac
vere bradycardia is usually present, and sudden
Microbiology

and neurologic abnormalities. Cardiac abnor-


death is a possibility. This condition is usually
malities include myocarditis, arrhythmias, and
treated with a pacemaker.
conduction disturbances. Lyme arthritis is a
late-stage finding, occurs in about 60% of pa-
4. The correct answer is B. The gradual onset
tients months to years later, and is associated
of her symptoms, together with the radiologic
with pain and swelling of large joints, most of-
findings of diffuse interstitial infiltrates, sug-
ten in one or both knees. Lyme disease is most
gests atypical pneumonia. Atypical pneumo-
prevalent in the northeast Atlantic Coast states,
nia is caused most commonly by Mycoplasma
but cases have been reported throughout the
pneumoniae, Legionella pneumophila, Chla-
United States.
mydia pneumoniae, and viruses; however, IgM
Answer A is incorrect. Brugada syndrome is cold agglutinin production is seen only with
a conductive heart disease that usually affects Mycoplasma infection.
young men and carries an increased risk of
Answer A is incorrect. Culture on buffered
sudden cardiac death. The disease has been as-
charcoal yeast extract medium is performed to
sociated with sodium ion channel abnormali-
diagnose L pneumophila pneumonia. L pneu-
ties. The typical ECG pattern is a right bun-
mophila causes atypical pneumonia that is
dle-branch block and ST-segment elevation on
seen most commonly in older individuals who
leads V1-V3.
smoke and abuse alcohol. Although Legionella
Answer B is incorrect. Chronic Chagas dis- is transmitted through environmental water
ease usually manifests during its earliest phase sources, infection does not imply aspiration.
with arrhythmias (eg, heart block and ven-
Answer C is incorrect. Phyocyanins, a product
tricular tachycardia). Dilated cardiomyopathy,
of Pseudomonas aeruginosa, lead to the blue-
megacolon, and megaesophagus occur later in
green color of the organisms. Pseudomonas can
the course of the disease. The disease presents
cause pneumonia but typically in patients who
acutely after the transfer of Trypanosoma cruzi
have cystic fibrosis or are severely immuno-
(found in the southern United States, Mexico,
compromised.
and Central and South America) by the redu-
viid bug (also called the kissing bug). Trans- Answer D is incorrect. Polysaccharide cap-
mission is associated with a hardened red area sules are a characteristic of Streptococcus pneu-
or chagoma. This is followed by fever, malaise, moniae and other organisms including certain
lymphadenopathy, tachycardia, and menin- strains of Haemophilus influenzae, Neisseria
goencephalitis that resolve within one month. meningitidis, and Escherichia coli. S pneu-
The patient’s ECG tracing is classic for Mobitz moniae is the cause of typical lobar pneumo-
type II heart block. Chagas disease would be a nia, which is characterized by sudden onset of
rare diagnosis in the northern United States. fever, chills, cough, and pleuritic pain. X-ray
High-Yield Principles
Chapter 4: Microbiology  •  Answers 85

of the chest usually shows focal lung consoli- Answer B is incorrect. CMV encephalitis can
dation rather than diffuse infiltrates, as seen in mimic the appearance of PML, but would
this case. be associated with enhancing periventricular
white matter lesions in cortical and subepen-
Answer E is incorrect. Reticulate bodies are
dymal regions. CMV encephalitis also is asso-
the intracellular form of Chlamydia species,
ciated typically with more systemic signs and
including C pneumoniae. C pneumoniae can
symptoms. Polymerase chain reaction analysis
cause atypical pneumonia that presents simi-
of CSF would be positive for CMV, and histo-
larly to Mycoplasma pneumonia. It is difficult
logic exam shows giant cells with eosinophilic
to distinguish between the two based on symp-
inclusions in both the cytoplasm and the nu-
toms and presentation, so treatment usually is
cleus.
designed to cover both organisms. Mycoplasma
infection, however, is much more common. Answer D is incorrect. Central nervous system
(CNS) lymphoma typically affects those with
5. The correct answer is C. The clinical picture CD4+ cell counts <50/mm3. MRI will dem-

Microbiology
and imaging are consistent with progressive onstrate one or more enhancing lesions (50%
multifocal leukoencephalopathy (PML) sec- are multiple and 50% are single) that typically
ondary to reactivation of latent JC virus infec- are surrounded by edema, and can produce a
tion, which can occur with CD4+ counts <50/ mass effect. CNS lymphoma can present with
mm3. It typically presents with rapidly progres- polymerase chain reaction findings positive for
sive focal neurologic deficits without signs of Epstein-Barr virus on CSF.
increased intracranial pressure. Ataxia, apha-
Answer E is incorrect. Space-occupying le-
sia, and cranial nerve deficits also may occur.
sions due to toxoplasmosis infection represent the
Lumbar puncture is nondiagnostic and fre-
most common cause of cerebral mass lesions in
quently demonstrates mild elevations in pro-
HIV-infected patients, and typically present with
tein and WBCs. Cerebrospinal fluid (CSF)
multiple enhancing lesions on MRI. The le-
analysis can reveal the presence of myelin
sions typically are located at the corticomedullary
basic protein, which is due to demyelination
junction, and are surrounded by edema that fre-
caused by the JC virus. PML typically presents
quently produces a mass effect and distinguishes
as multiple nonenhancing T2-hyperintense
its appearance from PML. Positive Toxoplasma
lesions. When it is suspected, stereotactic bi-
serologies can assist in diagnosis, and clinical
opsy is required for definitive diagnosis, but a
improvements will result from treatment with
positive CSF polymerase chain reaction for JC
sulfadiazine/pyrimethamine or trimethoprim/
virus is diagnostic in the appropriate clinical
sulfamethoxazole.
setting. Histology of the lesions shows nuclear
inclusions in oligodendrocytes. Although there
6. The correct answer is E. Rubeola, also called
is no definitive treatment, clearance of JC virus
measles, is a relatively rare illness in the
DNA can be observed with response to highly
United States because of the ubiquity of the
active antiretroviral therapy.
measles/mumps/rubella (MMR) vaccine. It
Answer A is incorrect. Uncommon in the de- presents with the prodrome described in this
veloped world, but presenting with increased patient. The rash that spreads from head to
risk in homeless and HIV patients, cortical toe over a three-day period develops one or
tuberculomas are caseating foci within the two days after the appearance of Koplik’s spots,
cortical parenchyma occurring from previous which are red oral lesions with blue-white cen-
hematogenous mycoplasma bacillemia. The ters.
clinical presentation may be similar to that
Answer A is incorrect. Diphtheria is an ill-
of the current patient; however, presentation
ness virtually unknown in the United States
would include enhancing nodular lesions on
because of the prevalence of the diphtheria/
imaging and elevated protein and low glucose
tetanus/pertussis (DTaP) vaccine. It is caused
on CSF examination.
High-Yield Principles
86 Section I: General Principles  •  Answers

by Corynebacterium diphtheriae and is charac- zolid, erythromycin, tetracycline & doxycy-


terized by a membranous pharyngitis. cline, and the aminoglycosides. Doxycycline is
the main treatment for both Rickettsia rickettsii
Answer B is incorrect. Pertussis, or whooping
(Rocky Mountain spotted fever) and Borrelia
cough, is also rare due to widespread vaccina-
burgdorferi (Lyme disease) infections. The rash
tions. It is a respiratory infection of children
of Rocky Mountain spotted fever is typically
that characteristically produces coughing
petechial and begins around the wrists and an-
spasms followed by a loud inspiratory whoop.
kles, although it may begin on the trunk or dif-
Answer C is incorrect. Roseola is a febrile dis- fusely. The rash of Lyme disease may be solid
ease of very young children that begins with a red or may form a ring or multiple rings with a
high fever and progresses to a rash similar to bulls-eye appearance.
measles. Infants and young children are most
at risk. It is believed to be caused by human 8. The correct answer is A. Four members of the
herpesvirus 6. Plasmodium genus of protozoa commonly in-
Microbiology

fect humans and cause malaria. All are spread


Answer D is incorrect. Rubella, also known
by the female Anopheles mosquito; diagnosis is
as German measles, is a less severe viral ex-
made through a blood smear. The species that
anthem. Many infections are subclinical, but
causes cerebral involvement is P falciparum,
rubella can cause severe birth defects when in-
which is almost entirely responsible for the se-
fection occurs during the prenatal period.
vere cases of disease that proceed to coma and
7. The correct answer is B. Based on the derma- death.
tomal and unilateral distribution of this rash, Answer B is incorrect. Plasmodium malariae
the patient most likely has shingles. This is a infection causes a 72-hour cyclic fever. P ma-
focal reactivation of a prior varicella-zoster vi- lariae, however, does not cause cerebral ma-
rus (VZV) infection. Most patients who de- laria.
velop shingles have a two- to three-day pro-
Answer C is incorrect. Plasmodium ovale in-
drome of pain, tingling, or burning in the
fection causes a 48-hour cyclic fever. A unique
involved dermatome, followed by the devel-
feature of P vivax and P ovale organisms is that
opment of a vesicular rash. The treatment of
they can form hypnozoites that can remain
choice for herpesvirus infections is acyclovir,
dormant in the liver for long periods, only to
ganciclovir, and (for VZV specifically) fam-
resurface later. However, P ovale does not
ciclovir, which work by inhibiting viral DNA
cause cerebral malaria.
polymerase.
Answer D is incorrect. Plasmodium vivax in-
Answer A is incorrect. Bacteria, not viruses,
fection causes a 48-hour cyclic fever. A unique
have cell walls. Inhibition of cell wall synthe-
feature of both P vivax and P ovale organisms
sis is accomplished by the penicillin family of
is that they can form hypnozoites that can re-
antibiotics.
main dormant in the liver for long periods,
Answer C is incorrect. The antiviral medica- only to resurface later. However, P vivax does
tion amantadine, used only in the treatment of not cause cerebral malaria.
influenza A virus infection, works by inhibiting
Answer E is incorrect. Plasmodium knowlesi is
viral genome uncoating in the host cell.
a simian malaria parasite that primarily infects
Answer D is incorrect. Nucleoside reverse macaques, although it has been reported to
transcriptase inhibitors are first-line medica- infect humans in southeast Asia. There are re-
tions for treating HIV infection. ports of cerebral involvement in monkeys, and
Answer E is incorrect. Inhibition of protein of isolated fatal human cases.
synthesis is achieved by five types of antibiot-
9. The correct answer is C. These lab results and
ics: chloramphenicol & clindamycin, line-
the clinical presentation (fever, headache, nu-
High-Yield Principles
Chapter 4: Microbiology  •  Answers 87

chal rigidity, and photophobia) are typical of are associated with an increased risk of devel-
viral meningitis. Because the cultures are neg- oping hepatocellular carcinoma. The liver has
ative, a viral cause should be considered. This a high regenerative potential, but if this pro-
patient would have recovered without compli- cess is overused, the chance of an oncogenic
cations with only symptomatic support. Viral mutation occurring during the regeneration of
aseptic meningitis usually is caused by entero- cells increases.
viruses and runs a milder course than bacterial
Answer C is incorrect. HIV as a direct onco-
meningitis.
genic agent is being intensely researched, but
Answer A is incorrect. This profile does not it is already known that immune suppression
suggest meningitis. However, because the clin- and dysregulation caused by HIV infection
ical presentation strongly suggests meningitis, give rise to lymphomas and Kaposi sarcoma.
it is not likely that the CSF analysis would be
Answer E is incorrect. Human T-cell lym-
completely normal.
photropic virus causes adult T-cell leukemia,

Microbiology
Answer B is incorrect. In this profile, only the and although the mechanism of oncogenesis
CSF sugar level is elevated. This does not sug- remains unclear, there is some evidence that
gest bacterial meningitis, in which sugar levels integration into the host genome at locations
would decrease. This profile may suggest sys- near cellular growth genes may play a role.
temic hyperglycemia, such as in uncontrolled
diabetes. 11. The correct answer is B. Influenza virus has
both hemagglutinin (HA) and neuramini-
Answer D is incorrect. This is a typical profile
dase (NA) molecules on its surface. These
of fungal or mycobacterial meningitis. Note
two molecules are responsible for the ability
that it is the same profile as that of bacterial
of the virus to be absorbed and penetrate the
meningitis, except the increase in WBCs is
host cells. After a human is infected with the
due to lymphocytes, not neutrophils. Fungal
influenza virus, that person will be immune
and mycobacterial meningitis also have a more
to infection by the same virus because of an-
subacute presentation than has bacterial men-
tibodies created against HA and NA. If either
ingitis.
HA or NA is changed, as can be the case if two
Answer E is incorrect. This is a typical profile different influenza viruses infect the same cell
of bacterial meningitis, in which neutrophils and exchange RNA, antigenic shift can occur.
predominate over lymhocytes. This creates a new virus that has never been
exposed to the human immune system before,
10. The correct answer is D. Human papillomavi- with potentially catastrophic consequences.
rus (HPV) causes carcinoma (usually cervical) This type of mixing is most commonly thought
by inactivating tumor suppressor genes such to be between a human and an avian strain
as p53 and Rb through the actions of viral pro- mixing in an intermediary porcine host, thus
teins E6 and E7, respectively. leading to the term “avian flu.”
Answer A is incorrect. Epstein-Barr virus Answer A is incorrect. Antigenic drift de-
(EBV) is associated with Burkitt lymphoma (a scribes mutations that can occur in hemag-
B-lymphocyte lymphoma) and nasopharyngeal glutinin and neuraminidase, making them
carcinoma. The t(8;14) translocation is consis- less antigenic to the preexisting antibod-
tently associated with Burkitt lymphoma, but ies in the human host. Since this results in
the translocation alone is not responsible for small changes in viral toxicity, it will lead to
the neoplasm and is not found in nasopharyn- a slightly different strain, but it is not likely to
geal carcinomas. The other factors that deter- lead to a global epidemic.
mine oncogenesis of EBV remain unclear.
Answer C is incorrect. Hemagglutinin has the
Answer B is incorrect. Both hepatitis C ability to attach to sialic acid receptors, which
(HCV) and hepatitis B virus (HBV) infections
High-Yield Principles
88 Section I: General Principles  •  Answers

activates fusion of the virus to the cell. All in- Answer B is incorrect. Bone is an extremely
fectious influenza viruses have this molecule. unlikely source for cysticerci due to its rela-
tively low blood flow.
Answer D is incorrect. Neuraminidase has the
ability to destroy neuraminic acid, a compo- Answer C is incorrect. The kidney can be a
nent of mucin. This helps break down the bar- location for cysticerci but is much less likely
rier to the upper airways and aids in infectivity. than cysts involving muscle tissue.
Answer E is incorrect. RBCs agglutinate in Answer E is incorrect. While the small bowel
the presence of hemagglutinin; hence the is the site of infection of primary hosts like the
name. This does not affect the infection rate of pig, secondary hosts (humans) do not develop
the influenza virus. an adult tapeworm infection.

12. The correct answer is D. The image shows 13. The correct answer is A. This patient is pre-
multiple lesions throughout the brain paren- senting with a classic case of whooping cough
Microbiology

chyma and subarachnoid space, which are caused by Bordetella pertussis. The initial
characterized by ring-shaped regions of low T1 phase is characterized by flu-like symptoms
intensity consistent with calcification. This ap- for the first one-two weeks. During this time,
pearance is most consistent with the nodular erythromycin is an effective treatment. The
calcified stage of neurocysticercosis and is seen second phase, the paroxysmal stage, is marked
only in individuals with long-standing, chronic by bouts of multiple coughs in a single breath
infection from endemic areas. Although this followed by a deep inspiration (the classic
patient’s presentation is highly suspicious for whooping cough). Treatment during this phase
malignancy, the image provided and labora- does not change the disease course, so only
tory data confirm a diagnosis of neurocysticer- supportive care is indicated and the infection
cosis, which is caused by infection with Taenia ought to pass in otherwise healthy individu-
solium, a pork tapeworm. It is the most com- als. In the United States, the diptheria/tetanus/
mon parasitic infection of the CNS worldwide, pertussis (DTaP) vaccine is supposed to be
and is particularly endemic to Central and given to all infants and protects them against
South America, Eastern Europe, and some diphtheria, tetanus, and pertussis. Infants who
parts of Asia. After humans ingest the tape- are not vaccinated are at risk for infection.
worm’s eggs, the eggs hatch and the larvae B pertussis can only be cultured on Bordet-
invade the wall of the small intestine and dis- Gengou medium.
seminate hematogenously. Cysticerci may be
Answer B is incorrect. Charcoal yeast extract
found in any organ, but are most commonly
when buffered with increased levels of iron
found in the brain, muscles, skin, and heart.
and cysteine is used to culture Legionella
Since we know that this patient is already suf-
pneumophila.
fering from cysts in her brain, the most likely
additional location would be her muscles. Answer C is incorrect. Chocolate agar with
Fortunately, the disease rarely results in death factor V and X is used to culture Haemophilus
and patients are often asymptomatic; however, influenzae.
when the disease does result in neurologic se- Answer D is incorrect. Loeffler medium is
quelae, specific symptoms depend on the loca- needed to culture Corynebacterium diphthe-
tion of the cysts. Cysticercosis is treated with riae.
administration of albendazole.
Answer E is incorrect. Thayer-Martin me-
Answer A is incorrect. Although the cysticerci dium is used to culture Neisseria gonorrhoeae.
may be found in virtually any organ, they al-
most never involve the urinary bladder. Schis- 14. The correct answer is E. This describes all

tosomiasis is a parasite that commonly invades spore-forming bacteria, which include Bacil-
the bladder. lus anthracis, Bacillus cereus, and Clostridium.
High-Yield Principles
Chapter 4: Microbiology  •  Answers 89

However, only Clostridium botulinum pro- common fungal infection in the US. About
duces the symptoms seen in this baby and 60% of these infections cause no symptoms,
also fits the mode of transmission. C botuli- and in the remaining 40% of cases, the symp-
num causes botulism via the production of a toms can range from mild to severe. Severe
heat-labile toxin that inhibits the release of forms of the infection can present with blood-
acetylcholine into the neuromuscular junc- tinged sputum, loss of appetite, weight loss,
tion. Infants may initially become constipated a painful red rash on the legs, and change in
and then develop generalized muscle weak- mental status. Cultures from sputum samples
ness (“floppy baby”). The organism is spread or biopsy show a dimorphic fungus seen as hy-
through the ingestion of contaminated canned phae at 25°C (77°F) and spherules filled with
or bottled food. Additionally, fresh honey has endospores at 37°C (98.6°F). Treatment with
been shown to harbor the organism. amphotericin B or fluconazole is usually re-
quired only in severe, disseminated disease.
Answer A is incorrect. IgA protease is pro-
duced by some bacteria so they can cleave Answer A is incorrect. In a patient with fever,

Microbiology
secretory IgA and colonize mucosal areas; chills, and flu-like symptoms who has recently
Neisseria gonorrhoeae, Neisseria meningitidis, returned from Namibia, there is concern for
Streptococcus pneumoniae, and Haemophilus infection by Plasmodium species, which cause
influenzae are the most well known. However, malaria. Malaria is transmitted by the female
none of these are typically transmitted via food, Anopheles mosquito. The time course and pat-
namely honey. tern of symptoms depend on the Plasmodium
species with which the patient is infected.
Answer B is incorrect. Exotoxin A is produced
Treatment is tailored to the geographic area
by Pseudomonas aeruginosa as well as some
of infection and the Plasmodium species in-
Streptococcus species. Exotoxin A has been as-
volved; agents include chloroquine, hydroxy-
sociated with toxic shock syndrome and scarlet
chloroquine, and atovaquone-proguanil. Ma-
fever. However, none of these organisms pro-
larial infection would be evident on a blood
duce the symptoms seen in this case or is trans-
smear.
mitted by honey ingestion.
Answer C is incorrect. Sickness after travel
Answer C is incorrect. Lecithinase is pro-
to the Mississippi and Ohio River valleys is
duced by Clostridium perfringens and is re-
suggestive of histoplasmosis. Although histo-
sponsible for the development of gas gangrene,
plasmosis typically does not present symptom-
cellulitis, and diarrhea. This organism is associ-
atically, some patients experience a flu-like
ated with contaminated wounds, which is not
illness with fever, cough, headaches, and myal-
a part of this baby’s history.
gias. Histoplasmosis can result in lung disease
Answer D is incorrect. Lipopolysaccharide, resembling tuberculosis (TB) and widespread
also called endotoxin, is produced by gram- disseminated infection affecting the liver,
negative bacteria and Listeria. It is highly anti- spleen, adrenal glands, mucosal surfaces, and
genic and can cause sepsis in severe infections. meninges. On microscopy, histoplasmosis ap-
However, generalized muscle weakness is not pears as spherules filled with endospores, as
characteristic of sepsis. opposed to the hyphae and spherules observed
in the sputum of those with coccidiomycosis.
15. The correct answer is B. This patient’s history
and sputum culture are suggestive of coccidi- Answer D is incorrect. Recent hiking in
oidomycosis, a fungal infection caused by the wooded areas carries the risk of contracting
inhalation of Coccidioides immitis or Coccidi- tick-borne illnesses, such as those carried by
oides posadasii. These organisms are found in the Ixodes tick: Babesia microti, a protozoon
soil in dry areas of the southwestern United that causes babesiosis; Borrelia burgdorferi, a
States, Mexico, and Central and South Amer- spirochete that causes Lyme disease; and Ehr­
ica. Coccidioidomycosis is the second most lichia chaffeensis, a rickettsial bacterium that
High-Yield Principles
90 Section I: General Principles  •  Answers

causes erlichiosis. None of these organisms ap- valve, such as “physiologic split” and, in this
pears as hyphae on microscopy. case, a large PDA, we hear the split S2 sound.
Answer E is incorrect. A wound while garden- Answer A is incorrect. Congenital CMV in-
ing, such as a thorn prick, can cause inocula- fection is marked most commonly by pete-
tion with Sporothrix schenckii. This fungus can chial rashes, jaundice, hepatosplenomegaly,
be found in various environments, including and sensorineural hearing loss. Cardiovascular
sphagnum moss, decaying vegetation, hay, abnormalities are not features of congenital
and soil. When S schenckii is introduced into CMV infection.
the skin, it causes a local pustule or ulcer with
Answer B is incorrect. Congenital herpes in-
nodules along the draining lymphatics (as-
fection most often either affects skin, eyes, and
cending lymphangitis). S schenckii is a dimor-
mouth or presents as localized CNS infection.
phic fungus, existing as hyphae at 25°C (77°F)
Symptoms usually develop within four weeks
and as a budding yeast form at 37°C (98.6°F).
of birth. CNS symptoms can include tempera-
Itraconazole or potassium iodide is used for
Microbiology

ture instability, respiratory distress, poor feed-


treatment. On microscopy, one would not ex-
ing, and lethargy. Herpes simplex virus type 2
pect to see spherules.
is also one of the most common causes of neo-
natal encephalitis.
16. The correct answer is C. All five answers

are part of ToRCHeS (Toxoplasmosis, Ru- Answer D is incorrect. Congenital toxoplas-
bella, Cytomegalovirus, Herpesvirus/HIV, mosis infection most often is asymptomatic ini-
and Syphilis) infections, the group of infec- tially. The class triad of symptoms that develop
tions for which every newborn is tested. If the include chorioretinitis, hydrocephalus, and
mother becomes infected during pregnancy, intracranial calcifications. Early symptoms can
the pathogens can cross the placenta and in- include a maculopapular rash, jaundice, and
fect the fetus. Rubivirus causes rubella in hepatomegaly. Most complications develop if
adults. The virus crosses the placenta in the the infection is not treated soon after birth.
first trimester and causes congenital abnor-
Answer E is incorrect. Newborns with con-
malities that range from deafness to cataracts
genital syphilis normally are asymptomatic
to cardiovascular abnormalities. The abnormal
at birth. When symptoms develop, the babies
heart exam findings in this newborn are clas-
often have hearing problems (based on cra-
sic for a patent ductus arteriosus (PDA) with
nial defect VIII) and cutaneous lesions that
delayed symptoms of heart failure. A PDA is
normally appear on the palms and soles first.
the most common cardiovascular abnormal-
More serious symptoms include anemia, jaun-
ity seen in congenital rubella syndrome. A
dice, and hepatomegaly. Patent ductus arterio-
PDA results from the failure of the ductus ar-
sus is not found with congenital syphilis.
teriosus to close in the first days of life. This
results in a left-to-right shunt from the aorta to 17. The correct answer is G. This patient is likely
the pulmonary artery. In a substantial shunt, presenting with a mucosa-associated lymphoid
deoxygenated blood returning from the body tissue (MALT) lymphoma. This type of indo-
to the heart bypasses the lung via the PDA to lent lymphoma is believed to be associated
the aorta. Because blood is shunted, there is a with infection by the organism Helicobacter
widening of pulse pressure (the difference be- pylori. H pylori is commonly identified by the
tween systolic and diastolic). A split S2 sound presence of urease, catalase, and oxidase. Erad-
occurs in PDA because of the increased flow ication of the infection with antibiotics and
through the pulmonary artery. The S2 sound is proton-pump inhibitors is often sufficient to
a composite of two distinct heart valves. Nor- cause regression of the lymphoma.
mally, the aortic valve closes just before the
pulmonary valve. When the pulmonary valve Answer A is incorrect. This pattern is seen in
is forced to stay open longer than the aortic benign flora such as the lactobacilli.
High-Yield Principles
Chapter 4: Microbiology  •  Answers 91

Answer B is incorrect. This pattern would be Moreover, an invasive process by the organism
commonly seen in streptococcal species such would likely take >4 hours to produce symp-
as Streptococcus pneumoniae. toms.
Answer C is incorrect. This pattern of results Answer D is incorrect. Vibrio cholerae causes
is commonly seen in facultative anaerobes large-volume, watery diarrhea. Treatment in-
such as Escherichia coli. volves prompt replacement of water and elec-
trolytes. Although antibiotics are not needed
Answer D is incorrect. This pattern represents
for treatment, tetracyclines have been shown
Pseudomonas aeruginosa, an opportunistic
to reduce the course of the disease. However,
lung pathogen.
there is nothing about this patient’s history to
Answer E is incorrect. This represents a pat- suggest that he has been exposed to cholera.
tern common of true anaerobes in the Bacter­ Rather, the history indicates a food-related
oides family. cause.

Microbiology
Answer F is incorrect. This pattern is seen in
19. The correct answer is C. This patient presents
Proteus mirabilis, a common cause of urinary
with classic symptoms of hepatocellular carci-
tract infection.
noma. Approximately 10%-30% of people in-
18. The correct answer is E. Food poisoning is fected with HCV will develop cirrhosis of the
the major cause of illness in this patient, and liver. Approximately 1%-5% of these patients
the most likely cause in this case is preformed develop hepatocellular carcinoma. HCV is
exotoxin from Bacillus cereus secreted into the transmitted via blood or blood transfusions
gastrointestinal (GI) tract. These exotoxins are and, rarely, by sexual contact. This patient is
fast acting, so the symptoms of food poisoning more likely to have HCV infection due to his
(nausea, vomiting, diarrhea) are usually rapid lack of travel, history of blood transfusion prior
in onset (within four-eight hours of ingestion). to the availability of sensitive screening meth-
Other major causes of food poisoning resulting ods, and extensive macronodular cirrhosis. Up
in nonbloody diarrhea include Staphylococcus to 90% of HCV-related hepatocellular carcino-
aureus, Clostridium perfringens, and entero- mas occur in patients with cirrhosis. Onset of
toxigenic Escherichia coli, which cause trav- hepatocellular carcinoma occurs on average
eler’s diarrhea. These organisms are typically 30 years after initial infection with HCV.
found in specific types of food, and B cereus is Answer A is incorrect. In the immunocom-
found in reheated rice. Because the food poi- petent host, CMV infection is often asymp-
soning in B cereus infection is caused by pre- tomatic, or it may produce a mononucleosis
formed enterotoxins, antibiotic treatment will syndrome. Disease manifestations are more
not help and supportive care is recommended. common in the immunocompromised host,
Answer A is incorrect. Bismuth subsalicylate, and include CMV colitis and CMV retinitis.
metronidazole, and amoxicillin are used to Patients with symptomatic CMV infection
treat Helicobacter pylori infection, which does may have subclinical transaminitis; however,
not result in the symptoms seen in this patient. there is no association with chronic hepatitis.

Answer B is incorrect. Fluoroquinolones Answer B is incorrect. Hepatitis A is transmit-


can be used to treat severe Shigella infection, ted via the fecal-oral route and causes an acute
which causes a bloody diarrhea. self-limited GI infection. It may rarely result
in fulminant hepatic failure requiring liver
Answer C is incorrect. Campylobacter jejuni transplantation, but it does not cause a chronic
enterocolitis can be treated with erythromy- hepatitis or cirrhosis.
cin or ciprofloxacin. Infection with this organ-
ism is not associated with eating reheated rice Answer D is incorrect. Hepatitis E is transmit-
and would typically result in bloody diarrhea. ted via the fecal-oral route and has been linked
to fatalities in pregnant women. It does not
High-Yield Principles
92 Section I: General Principles  •  Answers

cause chronic hepatitis except very rarely in 21. The correct answer is B. This boy is suffering
patients who have previously received solid or- from the classic hemolytic-uremic syndrome
gan transplants. (HUS) caused most often by the endotoxin
of Escherichia coli O157:H7 contracted from
Answer E is incorrect. HIV is transmitted via
undercooked beef. This disease is caused by
bodily fluids and causes the death of CD4+ T
endothelial injury and platelet aggregation
lymphocytes, resulting in an immunocompro-
that lead to the classic triad of microangio-
mised state and increased susceptibility to op-
pathic hemolytic anemia, thrombocytopenia,
portunistic infections. It does not cause hepa-
and acute renal injury. Up to 75% of cases of
titis.
classic HUS occur as a result of infection with
20. The correct answer is A. The image shows
E coli O157:H7.
gram-positive cocci in clusters. Staphylococcal Answer A is incorrect. Campylobacter in-
scalded skin syndrome (SSSS) is caused by the fections can cause diarrheal illnesses and
release of two exotoxins (epidermolytic toxins have been implicated in the development of
Microbiology

A and B) from Staphylococcus aureus. Des- Guillain-Barré syndrome, but would not be ex-
mosomes (also called “macula adherens”) are pected to cause HUS.
responsible for binding epithelial cells to one
Answer C is incorrect. Rotavirus is a common
another to form a coherent whole. The exotox-
cause of infantile gastroenteritis. It is caused
ins that are released bind to a molecule within
by a double-stranded virus (reovirus). It is typi-
the desmosome called desmoglein 1, thereby
cally spread throughout daycare centers but
disrupting cell adhesion. In SSSS, the epider-
does not cause HUS.
mis separates at the stratum granulosum due to
the binding of exotoxins to desmosomes in this Answer D is incorrect. Shigella is known to
layer. Clinically, this results in bullous lesions cause bloody diarrhea. It can cause HUS, but
and a positive Nikolsky sign. this is less common than E coli-induced HUS.
Answer B is incorrect. Gap junctions are cir- Answer E is incorrect. Vibrio cholerae causes
cular intercellular contact areas that permit massive watery diarrhea by secreting cholera
the passage of small molecules between adja- toxin. Vibrio infections are not implicated in
cent cells, allowing communication to facili- classic HUS. This bacterium is generally asso-
tate electrotonic and metabolic function. ciated with contaminated water sources rather
than ill-prepared food.
Answer C is incorrect. Hemidesmosomes are
present on the basal surface of epithelial cells
22. The correct answer is A. These symptoms are
adjacent to the basement membrane, and
typical of urethritis. The most common causes
serve to connect epithelial cells to the underly-
of urethritis in males are Chlamydia tracho-
ing extracellular matrix.
matis and Neisseria gonorrhoeae. The image
Answer D is incorrect. Intermediate junctions shows intracellular inclusions consistent with
lie deep to tight junctions, comprised of actin infection by C trachomatis. While they may be
filaments forming a continuous band around difficult to differentiate, C trachomatis infec-
the cell, providing structural support just be- tion induces a predominantly immunologic
low tight junctions. reaction, with only a few polymorphonuclear
leukocytes (PMNs), while N gonorrhoeae in-
Answer E is incorrect. Tight junctions are lo-
duces predominantly nonimmunologic in-
cated beneath the luminal surface of simple
flammation, with a PMN-rich infiltrate. The
columnar epithelium (eg, intestinal lining)
antibiotic of choice for chlamydia urethritis is
and seal the intercellular space to prevent dif-
azithromycin (macrolide) or doxycycline (tet-
fusion between cells.
racycline).
High-Yield Principles
Chapter 4: Microbiology  •  Answers 93

Answer B is incorrect. Ceftriaxone is an effec- tor of recent disease, given that IgM is the first
tive treatment for gonorrhea, but the cephalo- antibody produced in response to an antigen.
sporin class of antibiotics is relatively ineffec-
tive against Chlamydia trachomatis. 24. The correct answer is D. This patient likely
has pertussis, or whooping cough, which is
Answer C is incorrect. Fluconazole inhibits
caused by the gram-negative rod Bordetella
fungal steroid synthesis. It is used in the treat-
pertussis. This organism has four virulence fac-
ment of fungal infections, such as Candida al-
tors, including pertussis toxin. The A subunit
bicans.
of this exotoxin inhibits membrane-bound
Answer D is incorrect. Penicillin has been Gai proteins, which ultimately results in the
shown to suppress chlamydial multiplication. accumulation of cAMP. The effects of this ac-
However, it does not eradicate the organ- cumulation include histamine sensitization,
ism and thus is not the best treatment for this increased insulin synthesis, lymphocytosis, and
type of infection. Penicillin is the treatment of inhibition of phagocytosis.

Microbiology
choice for syphilis.
Answer A is incorrect. The botulinum toxin
Answer E is incorrect. Vancomycin has not released by Clostridium botulinum prevents
been shown to be effective in the treatment of the release of acetylcholine into the synaptic
chlamydial infection. It is used to treat drug- cleft, resulting in muscle weakness and paraly-
resistant Staphylococcus aureus and Clostrid- sis.
ium difficile.
Answer B is incorrect. The alpha toxin of
Staphylococcus aureus binds to the plasma
23. The correct answer is B. HBV is transmit-

membrane of host cells, forming a pore in
ted via parenteral, sexual, or maternal-fetal
the membrane that allows ions and small
routes. Of the markers listed, only hepatitis B
molecules to enter the cell. This leads to cell
e antigen (HBeAg) signifies active viral replica-
swelling and eventual lysis. Streptolysin O of
tion, and would therefore make transmission
Streptococcus pyogenes functions in a similar
of HBV to a partner more likely. HBeAg and
manner.
hepatitis B core antigen (HBcAg) are antigenic
markers of the virus core. They can be de- Answer C is incorrect. The exotoxin of Cory-
tected two-four months after exposure. nebacterium diphtheriae functions via the inac-
tivation of elongation factor 2, causing pharyn-
Answer A is incorrect. Hepatitis B e anti-
gitis and the formation of a pseudomembrane
body (HBeAb) is an antibody directed against
in the throat. Pseudomonas aeruginosa exo-
HBeAg. Its presence indicates low transmissi-
toxin A also works via this mechanism.
bility. HBeAb can be detected five months af-
ter exposure to HBV and one month after the Answer E is incorrect. Lipopolysaccharide-
detection of HBeAg. lipid A is an endotoxin released only by gram-
negative bacteria, with the exception of Liste-
Answer C is incorrect. Hepatitis B surface an-
ria monocytogenes, a gram-positive bacteria
tibody (HBsAb) provides immunity to HBV in-
responsible for meningitis in neonates and
fection. It can be detected in former carriers of
immunosuppressed patients. Endotoxins are
HBV or in patients immunized with the HBV
a normal part of the bacterial membrane re-
vaccine.
leased upon lysis of the cell.
Answer D is incorrect. IgG HBcAb is an indi-
cator of chronic disease. 25. The correct answer is E. Vibrio cholerae

causes watery stools, often called “rice-water”
Answer E is incorrect. Hepatitis B core anti-
stool. This illness is not accompanied by ab-
body (HBcAb) is produced in response to hep-
dominal pain, but the symptoms are due to
atitis core antigen (HBcAg). IgM is an indica-
dehydration, which leads to electrolyte imbal-
ances. Cholera toxin binds to the GM1 entero-
High-Yield Principles
94 Section I: General Principles  •  Answers

cyte receptor via the pentameric B subunit. mechanism of shock through anaphylaxis is
Once inside the cell, the toxin must undergo not related to DIC.
cleavage of the active, A1 component, which
Answer D is incorrect. The C5a component
goes on to constitutively activate the Gs protein
of the complement cascade, activated by en-
through ADP ribosylation. This results in high
dotoxin, functions in neutrophil chemotaxis.
cyclic AMP levels, which activate the CFTR
This is not the mechanism involved in anaphy-
(cystic fibrosis transmembrane conductance
laxis.
regulator) channel, leading to a large efflux
of chlorine and other ions into the GI lumen. Answer E is incorrect. The cytokine
This results in extremely watery diarrhea ac- interleukin-1, released by macrophages acti-
companied by electrolyte imbalances. vated by endotoxin, causes fever. Fever is ab-
sent in anaphylaxis.
Answer A is incorrect. This is a characteristic
of Shiga toxin, which typically leads to bloody Answer F is incorrect. Nitric oxide, released
diarrhea. by macrophages activated by endotoxin, causes
Microbiology

hypotension (shock). However, this is not the


Answer B is incorrect. Bacillus anthracis pro-
mechanism of anaphylactic shock.
duces a toxin that acts as an adenylate cyclase
(edema factor) but is not associated with severe 27. The correct answer is D. Loeffler medium,

watery diarrhea. blood agar, and tellurite plate are used to cul-
Answer C is incorrect. Diphtheria toxin and ture a throat swab from patients with suspected
exotoxin A from Pseudomonas inactivate elon- Corynebacterium diphtheriae infection. This
gation factor 2, but neither is a likely cause of organism causes symptoms of pseudomembra-
watery diarrhea. nous pharyngitis (thick gray membrane) and
lymphadenopathy. A possible complication of
Answer D is incorrect. Bordetella pertussis
this disease is the extension of the membrane
produces an exotoxin that increases cAMP lev-
into the larynx and trachea, resulting in airway
els by inactivating the inhibitory Gi. However,
obstruction. Rapid treatment with diphtheria
pertussis causes whooping cough, not severe
antitoxin, penicillin, or erythromycin and the
watery diarrhea.
DTP vaccine is indicated in this patient
26. The correct answer is B. Anaphylactic shock Answer A is incorrect. Löwenstein-Jensen agar
is a life-threatening disorder that occurs when is used to culture Mycobacterium tuberculosis,
an allergen overactivates mast cells and ba- the bacterium that causes TB.
sophils, leading to widespread release of his-
Answer B is incorrect. Bordet-Gengou agar is
tamines, serotonins, and other compounds
used to culture Bordetella pertussis, the etio-
stored in the granules of these immune cells.
logic agent responsible for whooping cough.
These compounds lead to vasodilation and
leaky capillaries. The crosslinking of IgE re- Answer C is incorrect. Chocolate agar with
ceptors present on these cells causes the activa- factors V and X is used to culture Haemophilus
tion of these cell types. By blocking this cross influenzae. This organism typically causes up-
reaction, the anaphylactic pathway can be sty- per respiratory infections but is not responsible
mied. for pseudomembranous pharyngitis.
Answer A is incorrect. γ Interferon is pro- Answer E is incorrect. Neisseria gonorrhoeae
duced by T lymphocytes and, among other is cultured on Thayer-Martin media. This bac-
functions, activates tumoricidal macrophages. terium can cause gonorrhea, septic arthritis,
neonatal conjunctivitis, and pelvic inflamma-
Answer C is incorrect. Endotoxin can directly
tory disease.
activate Hageman factor, activating the coagu-
lation cascade and leading to disseminated in-
travascular coagulation (DIC). However, the
High-Yield Principles
Chapter 4: Microbiology  •  Answers 95

28. The correct answer is B. This patient presents treonam is a b-lactamase-resistant monobac-
with Rocky Mountain spotted fever (RMSF) tam that interferes with cell wall biosynthesis
caused by Rickettsia rickettsii, a small, gram- by binding to penicillin-binding protein 3. Az-
negative bacterium carried by the American treonam is a potent antipseudomonal agent in-
dog tick (Dermacentor variabilis). Despite dicated for pseudomonal sepsis.
its name, RMSF is more common in the
Answer A is incorrect. Amoxicillin is an ami-
southeastern United States than in the Rocky
nopenicillin antibiotic that interferes with cell
Mountains. Patients often present first with
wall synthesis. Although amoxicillin has an
severe headache, fever (>38.9°C or >102°F),
extended spectrum compared with penicillin
and myalgias followed by a petechial rash on
(covering Haemophilus influenzae, Escherichia
the palms and soles (or wrists and ankles) that
coli, Listeria, Proteus, Salmonella, and Entero-
spreads to the trunk. The Weil-Felix assay re-
cocci), it does not provide antipseudomonal
action is the classic test for rickettsial disease.
coverage.
The treatment of choice for adults with RMSF

Microbiology
is doxycycline; chloramphenicol is also used Answer C is incorrect. Use of fresh frozen
but has more significant adverse effects. plasma (FFP) is reserved for patients with co-
agulopathy and signs of active, life-threatening
Answer A is incorrect. Cephalosporins (cef-
bleeding. Although provision of FFP will tem-
triaxone) are not effective against Rickettsia
porarily reverse some of this patient’s labo-
rickettsii. They are generally used to treat neo-
ratory signs of DIC (elevated International
natal/infant sepsis or gonorrhea, as well as bac-
Normalized Ratio, decreased fibrinogen),
terial meningitis.
treatment of the underlying cause (ie, Pseudo-
Answer C is incorrect. Aminoglycosides (gen- monas sepsis) is most important.
tamicin) are not effective against Rickettsia
Answer D is incorrect. Vancomycin is an an-
rickettsii. Aminoglycosides are effective against
tibiotic used for serious multidrug-resistant,
many gram-negative bacteria and some strains
gram-positive infections. Major uses are for
of Staphylococcus aureus. It is also used as
methicillin-resistant Staphylococcus aureus
broad-spectrum therapy when combined with
and moderate to severe Clostridium difficile in-
a penicillin or metronidazole.
fections. Its mechanism of action is to inhibit
Answer D is incorrect. Nystatin is used in the cell wall mucopeptide formation by binding
treatment of fungal infections such as oral can- the D-ala-D-ala portion of cell wall precursors.
didiasis.
Answer E is incorrect. Coagulopathy caused
Answer E is incorrect. Penicillin is not effec- by warfarin overdose is reversed by pharma-
tive against Rickettsia rickettsii. Penicillin is cologic administration of vitamin K. This pa-
still the treatment of choice for syphilis. Other tient’s coagulopathy is caused by Pseudomonas
derivatives that are used more frequently in- sepsis, so vitamin K therapy plays no role here.
clude oxacillin, cloxacillin, dicloxacillin, and
amoxicillin when Staphylococcus and Strepto- 30. The correct answer is C. Trichomoniasis is

coccus species prove sensitive. caused by Trichomonas vaginalis. Symptoms
include a profuse, frothy discharge that is as-
29. The correct answer is B. This patient has leu- sociated with vulvovaginal pruritus, tender-
kocytosis and Charcot’s triad (fever, jaundice, ness, and burning. The vulva and vagina are
right upper quadrant pain), along with the frequently inflamed. The cervix and vagina
ominous sign of hypotension, a clear clini- may develop small, red, punctate lesions, caus-
cal picture of cholecystitis. In addition, he ing the classic “strawberry” appearance. Diag-
has Pseudomonas aeruginosa sepsis and DIC. nosis is made by wet mount, on which small,
Gram-negative rod sepsis is the clear cause of pear-shaped, flagellated organisms can be seen
this patient’s DIC, and antipseudomonal cov- moving around.
erage with aztreonam is most appropriate. Az-
High-Yield Principles
96 Section I: General Principles  •  Answers

Answer A is incorrect. Bloody and foul- Answer B is incorrect. Entamoeba histolytica


smelling vaginal discharge suggest a vaginal infection presents with bloody diarrhea (dysen-
foreign body. tery), abdominal cramps with tenesmus, and
pus in the stool. It can also cause right upper
Answer B is incorrect. Pelvic or lower ab-
quadrant pain and liver abscesses. E histolyt-
dominal pain with an abnormal, foul-smelling
ica is transmitted via cysts in water (fecal-oral
vaginal discharge is suggestive of pelvic inflam-
transmission). On microscopy, one observes
matory disease, which can involve the endo-
amebas with ingested RBCs. Treatment for
metrium, fallopian tubes, ovaries, and peri-
E histolytica infection includes metronidazole
toneum. Fever, nausea, and vomiting may be
and iodoquinol.
present. The typical organisms involved are
Neisseria gonorrhoeae and Chlamydia tracho- Answer C is incorrect. Leishmania donovani
matis. infection presents with hepatomegaly and sple-
nomegaly, malaise, anemia, and weight loss.
Answer D is incorrect. Candida species typi-
L donovani is transmitted via the sandfly. Mi-
Microbiology

cally cause intense pruritus and a thick, odor-


croscopically, macrophages containing amas-
less, white, cottage cheese-like vaginal dis-
tigotes are observed. Sodium stibogluconate is
charge. Erythema, edema, dysuria, and urinary
used to treat L donovani infection.
frequency may be present. Budding yeasts and
hyphae are seen on a wet mount. Answer E is incorrect. The Plasmodium falci-
parum parasite is responsible for causing ma-
Answer E is incorrect. Bacterial vaginosis
laria. It is spread by the Anopheles mosquito.
typically presents with an unpleasant fishy-
Diagnosis of Plasmodium falciparum infection
smelling discharge that is thin, gray-white, and
is made through a blood smear.
homogeneous. Pruritus and inflammation are
unusual in bacterial vaginosis, thus the term
32. The correct answer is C. This image of the
“vaginosis” rather than “vaginitis.” On a wet
mitral valve reveals Aschoff’s nodule, which
mount, one will see clue cells that are formed
is pathognomonic for rheumatic fever. Rheu-
from bacteria-coated epithelial cells.
matic fever is caused by group A streptococci.
Years after a bout of rheumatic fever, rheu-
31. The correct answer is D. Naegleria fowleri

matic heart disease can develop due to calcifi-
presents with a rapidly progressing meningoen-
cation of warty vegetations on fibrotic healing,
cephalitis that can progress to coma or death
most commonly on the mitral valve. As a re-
within six days. Other symptoms include nau-
sult, it most commonly causes mitral stenosis,
sea, vomiting, and irrational behavior. Trans-
which in mild disease causes a late diastolic
mission occurs through swimming in fresh-
murmur heard best at the apex. In the US, the
water lakes. Microscopic analysis will reveal
incidence of this mitral stenosis secondary to
amebas in the spinal fluid. Unfortunately,
rheumatic fever is low given the widespread
there is no treatment for N fowleri.
use of antibiotics. However, rheumatic mitral
Answer A is incorrect. Cryptosporidium spe- stenosis is prevalent in immigrant populations.
cies infection presents with severe diarrhea in
Answer A is incorrect. A friction rub heard
HIV-positive patients and mild watery diarrhea
throughout the precordium would correspond
in HIV-negative patients. Cryptosporidium spe-
to pericarditis, an inflammation of the pericar-
cies are transmitted via cysts in water (fecal-
dial sac that leads to friction with an expanding
oral transmission). Microscopically, acid-fast
and contracting myocardium. It is not a long-
staining cysts are found. Unfortunately, there
term complication of rheumatic fever. Rather,
is no treatment available for Cryptosporidium
a friction rub may be associated with malig-
species infection; however, in healthy patients,
nancy, uremia, active infections such as viral
cryptosporidiosis is self-resolving.
or tuberculous, and other rare causes.
High-Yield Principles
Chapter 4: Microbiology  •  Answers 97

Answer B is incorrect. Aortic stenosis is repre- munoglobulin heavy chain enhancer. There
sented by a harsh crescendo-decrescendo early is a strong association between Burkitt lym-
systolic murmur heard best at the right upper phoma and Epstein-Barr virus (EBV). In en-
sternal border with radiation to the carotids. demic regions, children are typically infected
Rheumatic heart disease can lead to calcifica- with EBV by the age of three years, compared
tion of the aortic valve, which leads to aortic with infection during adolescence in other re-
stenosis, but for unclear reasons, this occurs gions. EBV, the cause of infectious mononu-
much less frequently than on the mitral valve. cleosis, is spread by saliva and respiratory secre-
An aortic stenosis murmur in a middle-aged tions.
woman should raise the suspicion for a calci-
Answer A is incorrect. HBV and HCV are
fied bicuspid aortic valve (remember the “fish
spread by blood-borne contacts. Although
mouth”).
these viruses are associated with hepatocellu-
Answer D is incorrect. A systolic click heard lar carcinoma, there is no association between
best at the apex corresponds to mitral valve HBV or HCV and Burkitt lymphoma.

Microbiology
prolapse, which is a relatively common and
Answer B is incorrect. Hepatitis A virus (HAV)
benign finding in middle-aged women. It oc-
is transmitted by the fecal-oral route. There is
curs when an abnormally thickened mitral
no association between HAV and Burkitt lym-
valve leaflet displaces into the left atrium dur-
phoma.
ing ventricular systole. It is not related to rheu-
matic heart disease. It may lead to mitral regur- Answer C is incorrect. Arboviruses and flavi-
gitation and ultimately valve surgery may be ruses can be transmitted through mosquito
required, but typically it is followed by a car- bites. These viruses cause tropical diseases
diologist with serial exams and echocardiogra- such as dengue fever and yellow fever, but not
phy. Burkitt lymphoma.
Answer E is incorrect. An S4, which is heard Answer E is incorrect. Viruses such as HIV,
best at the apex and is associated with concen- herpes simplex virus type 2, and human her-
tric left ventricular hypertrophy, is indicative of pesvirus type 8 are spread by sexual contact.
either chronic extensive afterload (ie, from un- None of these pathogens is associated with
controlled hypertension) or long-standing aor- Burkitt lymphoma.
tic stenosis. It also can be caused by diastolic
heart failure because the left ventricle is not as 34. The correct answer is D. This patient’s con-
compliant as it should be. The exam finding is stellation of symptoms and prior hospitaliza-
due to turbulent blood flow caused by blood tion points toward pseudomembranous coli-
filling a stiff ventricle. An S4 is not associated tis. Clostridium difficile proliferation causes
with rheumatic heart disease. the severe non-bloody diarrhea associated
with pseudomembranous colitis by produc-
33. The correct answer is D. This patient has
ing an exotoxin that kills enterocytes. Antibi-
Burkitt lymphoma, a lymphoma endemic otic treatments suppress the normal flora of
to Africa. Clinically, Burkitt lymphoma of- the GI tract, allowing C difficile to multiply.
ten presents with “B symptoms” (fever, night Clindamycin was the first antibiotic associated
sweats, weight loss), signs of tumor lysis syn- with C difficile gastroenteritis and is used of-
drome such as oliguria, and solitary jaw ten to treat anaerobic infections above the dia-
masses. Histopathologically, Burkitt lymphoma phragm, such as aspiration pneumonia. How-
typically assumes a “starry sky” appearance ever, many antibiotics have been implicated
with sheets of lymphocytes interspersed with since then, especially cephalosporins and am-
occasional macrophages. The cytogenetic ab- picillin. Always consider C difficile in patients
normality associated with Burkitt lymphoma is with gastroenteritis and recent antibiotic use.
a t(8;14) translocation in which the oncogene
c-myc is placed under the expression of the im-
High-Yield Principles
98 Section I: General Principles  •  Answers

Answer A is incorrect. This describes how the drome in young adults similarly characterized
Norwalk virus can cause gastroenteritis charac- by fever and pharyngitis. Both cause pericardi-
terized by nausea, vomiting, and diarrhea that tis only rarely.
resolves spontaneously within 12-24 hours.
Answer C is incorrect. Reoviridae is the only
Answer B is incorrect. This describes the exo- double-stranded RNA virus. Members of this
toxin produced by Shigella species, which can family include rotavirus, which is responsible
cause a bloody and mucus-rich diarrhea. for diarrhea in children, and reovirus, which
causes Colorado tick fever.
Answer C is incorrect. This describes the
exotoxin produced by Vibrio cholerae, which Answer D is incorrect. This answer choice de-
causes a large volume of watery diarrhea de- scribes the structure of Coronaviridae. Coro-
void of RBCs or WBCs (sometimes called rice- navirus (CoV) is a common virus that causes
water stool). It is not associated with prior anti- a self-limited cold-like syndrome. SARS-CoV,
biotic use. however, has been identified as the cause of se-
Microbiology

vere acute respiratory syndrome.


Answer E is incorrect. This describes Esch-
erichia coli, which causes the abrupt onset of
36. The correct answer is B. Guillain-Barré syn-
profuse watery diarrhea.
drome (GBS) is a common cause of acute
peripheral neuropathy that results in progres-
35. The correct answer is E. The patient presents
sive weakness over a period of days. Labora-
with classic signs and symptoms of pericardi-
tory abnormalities associated with GBS in-
tis, including precordial chest pain. This pain
clude elevated gamma-globulin, decreased
is relieved when leaning forward and worsens
nerve conduction velocity indicative of demy-
with inspiration. On physical exam the patient
elination, and albuminocytologic dissociation
has a pericardial friction rub, which accounts
(CSF shows increased protein concentration
for the scratchy, leathery sound heard during
with normal cell count in the setting of nor-
both systole and diastole. Classic ECG find-
mal glucose). Although one third of patients
ings include diffuse ST-segment elevation and
report no history of an infection, the other
depression of the PR segment. Pericarditis is
two thirds will have recently experienced an
frequently preceded by a viral upper respira-
acute GI or influenza-like illness prior to de-
tory infection. Although many viruses may
veloping the neuropathy. The most common
cause pericarditis, coxsackie B is the most
epidemiologic associations involve infections
common cause of inflammation of the pericar-
with Campylobacter jejuni, a comma-shaped,
dial membrane. Coxsackievirus is a picornavi-
oxidase-positive, gram-negative bacterium that
rus, the smallest of the RNA viruses. They are
can be grown at 42°C. C jejuni causes a gas-
positive, single-stranded, naked, icosahedral
troenteritis that often presents with bloody di-
RNA viruses.
arrhea. Although a recent history of C jejuni
Answer A is incorrect. Staphylococcus au- enteritis is epidemiologically linked with GBS,
reus is a gram-positive, catalase-positive, and the infection does not directly cause GBS. The
coagulase-positive bacterium. Infection with proposed mechanism of GBS is that a preced-
S aureus may lead to acute bacterial endocar- ing infection incites an immune response that
ditis from seeding secondary to bacteremia. cross-reacts with peripheral nerve components,
The bacterium rarely causes pericarditis. leaving them susceptible to damage also.
Answer B is incorrect. This describes the Answer A is incorrect. Streptococcus pneu-
structure of Herpesviridae. Herpesvirus is char- moniae is an a-hemolytic, encapsulated, gram-
acterized by multinucleated giant syncytial positive coccus that produces an IgA protease.
cells with intranuclear inclusion bodies. Mem- Pneumococcal pneumonia can result in bac-
bers of this family include CMV and Epstein- teremia, meningitis, osteomyelitis, or septic
Barr virus. Both cause a mononucleosis syn-
High-Yield Principles
Chapter 4: Microbiology  •  Answers 99

arthritis, but is not associated with the develop- Group A streptococci can cause cellulitis, nec-
ment of GBS syndrome. rotizing fasciitis, and streptococcal toxic shock
syndrome. Group B streptococci are a com-
Answer C is incorrect. Pseudomonas aeru-
mon cause of neonatal bacterial meningitis.
ginosa is a non-lactose-fermenting, oxidase-
Group D streptococci (Enterococcus) are a fre-
positive, gram-negative, aerobic bacillus that
quent cause of urinary tract infections and sub-
can cause otitis externa, urinary tract infection,
acute bacterial endocarditis.
pneumonia, and sepsis in immunocompro-
mised hosts. It is not associated with GBS syn- Answer C is incorrect. Staphylococcus epider-
drome or other neurologic conditions. midis is a coagulase-negative, gram-positive
coccus that grows in clusters. It is common
Answer D is incorrect. Clostridium botulinum
skin flora. It can be a cause of osteomyelitis,
is a rod-shaped, gram-positive, spore-forming,
but this is more common after implantation
anaerobe that produces a heat-labile toxin that
with orthopedic appliances.
inhibits acetylcholine release at the neuromus-

Microbiology
cular junction, causing flaccid paralysis. Botu- Answer E is incorrect. Salmonella species are
lism, therefore, does not lead to sensory find- oxidase-negative, gram-negative bacilli. Salmo-
ings, and is not associated with increased CSF nella osteomyelitis is often associated with pa-
protein on lumbar puncture. tients who have sickle cell disease.
Answer E is incorrect. Treponema pallidum,
38. The correct answer is C. The patient is now
the causative agent of syphilis, is a spiral-
in the secondary stage of a syphilis infection,
shaped bacterium with axial filaments, visu-
characterized by wart-like lesions known as
alized using dark-field microscopy. Tertiary
condylomata lata, generalized rash, and sys-
syphilis causes sensory deficits rather than
temic symptoms such as lymphadenopathy,
muscle weakness and is preceded by a painless
weight loss, and fever. The fluorescent trepo-
chancre and maculopapular rash. Neurologic
nemal antibody absorption (FTA-ABS) test,
findings develop after years, not weeks, of un-
the Venereal Disease Research Laboratory
treated infection.
(VDRL) test, and the rapid plasma reagin test
approach 100% sensitivity for detecting syphi-
37. The correct answer is D. This teenager most
lis.
likely has osteomyelitis secondary to a contigu-
ous focus of infection, such as bites, puncture Answer A is incorrect. Thayer-Martin agar is
wounds, and open fractures. Most of these used to culture Neisseria gonorrhoeae.
cases are caused by Staphylococcus aureus, a
Answer B is incorrect. Tzanck preparation
gram-positive, coagulase-positive coccus that
using Giemsa stain reveals multinucleated gi-
occurs in clusters. This organism expresses re-
ant cells indicative of herpes simplex virus
ceptors for bone matrix components, such as
(HSV) or vesicular stomatitis virus infection.
collagen, which help it to attach to and infect
Although HSV can cause genital lesions, they
bone. When compared to hematogenous os-
are usually painful and come in clusters.
teomyelitis, continguous-focus infections are
more likely to also include gram-negative and Answer D is incorrect. The Weil-Felix re-
anaerobic bacteria. action assay tests for antirickettsial antibod-
ies. This test would be positive if the patient
Answer A is incorrect. Acid-fast bacilli such
were infected with Rickettsia rickettsii (Rocky
as Mycobacterium tuberculosis are found in
Mountain spotted fever). Rocky Mountain
patients with TB. Infection of the bone by TB
spotted fever can also present with a rash on
is known as Pott disease, but its usual presenta-
the palms and soles but would not present with
tion is vertebral osteomyelitis.
genital lesions.
Answer B is incorrect. Streptococcus species
are catalase-negative, gram-positive cocci.
High-Yield Principles
100 Section I: General Principles  •  Answers

Answer E is incorrect. Ziehl-Neelsen stain is 40. The correct answer is D. This infant is show-
used to stain acid-fast mycobacteria such as ing signs of neonatal sepsis and meningitis.
Mycobacterium tuberculosis. The most common cause of neonatal sep-
sis and meningitis is group B Streptococcus
39. The correct answer is B. The first step in
(GBS), which frequently colonizes the genito-
answering this question is to recognize that urinary tract of women and can be passed to
Ziehl-Neelsen stain is what is used for the the infant during delivery. Penicillin G would
acid-fast test. The presence of acid-fast rods have provided adequate coverage for the moth-
in the blood indicates that this man has a dis- er’s likely colonization with GBS.
seminated mycobacterial infection. In order of
Answer A is incorrect. Fluconazole, an an-
prevalence, the three main candidate organ-
tifungal agent that interferes with fungal er-
isms in patients with HIV are: (1) M avium-
gosterol synthesis, would not have been an
intracellulare (MAC), (2) Mycobacterium tu-
appropriate antibiotic for GBS prophylaxis.
berculosis, and (3) Mycobacterium kansasii.
Fluconazole has good coverage against fungal
Microbiology

Disseminated MAC infection can present


infections.
with generalized symptoms (such as fatigue,
night sweats, fever, and weight loss) as well as Answer B is incorrect. Metronidazole, a dis-
organ-specific symptoms, and occurs when the ruptor of nucleic acid structure, would not
patient’s CD4+ cell count has dropped to <50/ have been an appropriate antibiotic for GBS
mm3. It can include gut pain (from mycobac- prophylaxis. Metronidazole has good activity
terial enteritis), pulmonary symptoms, or ad- against protozoans and anaerobic organisms.
enopathy. Although M tuberculosis infection
Answer C is incorrect. Moxifloxacin, a DNA
is frequently seen in HIV-infected patients, it is
gyrase inhibitor, would not have been an ap-
rare to recover it from blood cultures.
propriate antibiotic for GBS prophylaxis.
Answer A is incorrect. Actinomyces israelii is a Fluoroquinolones have good coverage against
bacterial pathogen with a fungal-like morphol- gram-negative and select gram-positive organ-
ogy. Unlike Nocardia, it is not acid-fast. isms, and are often used to treat urinary tract
infections and community-acquired pneumo-
Answer C is incorrect. Mycobacterium mari-
nia. Additionally, fluoroquinolones are a class
num is a species native to fresh-water and salt-
C substance, as they have the potential to
water environments. It is a rare cause of cuta-
cause teratogenic or embryocidal effects. Giv-
neous wound infection in anglers, swimmers,
ing fluoroquinolones during pregnancy is not
and aquarium owners. It is not commonly as-
recommended unless the benefits justify the
sociated with disseminated infection in immu-
potential risks to the fetus.
nocompromised patients.
Answer E is incorrect. Vancomycin, a cell
Answer D is incorrect. M tuberculosis could
wall synthesis inhibitor, would not have been
be a causative agent for the symptoms found
an appropriate antibiotic for GBS prophylaxis
in this patient, except that his CD4+ count
in this case. While vancomycin has good cov-
is <50/mm3, which makes infection with
erage against GBS, its use is reserved for cases
M avium-intracellulare most likely.
of known resistance. Vancomycin could have
Answer E is incorrect. Nocardia is eliminated been used if the mother were known to harbor
as the correct answer by the given morphology GBS that was resistant to other first-line antibi-
of the observed bacteria. Although Nocardia otics, or had an allergy that prevented the use
can stain weakly acid-fast, they demonstrate of other agents.
a branching filamentous morphology that re-
sembles fungal hyphae.
CHAPTER

8 Infectious Diseases

PBD9 Chapter 8 and PBD8 Chapter 8: Infectious Diseases

BP9 Chapter 8 and BP8 Chapter 9: General Pathology of Infectious Diseases


  

1 A 45-year-old Bangladeshi woman with atrophic gastritis but no RBCs or WBCs. Which of the following mechanisms
has sudden onset of severe, profuse, watery diarrhea. Over the best accounts for this diarrhea?
next 3 days, she becomes severely dehydrated. On physical ex- A Decreased absorption of sodium and water
amination, she is afebrile, but has poor skin turgor. Laboratory Decreased breakdown of lactose to glucose and
B 
studies of the diarrheal fluid show microscopic flecks of mucus, galactose
but no blood and few WBCs. A blood culture is negative. The C Increased secretion of potassium and water
woman is hospitalized and receives intravenous fluid therapy D Mutation in CFTR gene
for 1 week. Which of the following is the most likely diagnosis? E Presence of Yop virulence plasmid
Amebiasis
A   
Aspergillosis
B
Cholera
C
Filariasis
D
Hydatid disease
E
Typhoid fever
F

2 A 5-year-old boy has had diarrhea for a week, averag-


ing six low volume stools per day, which appear mucoid and
sometimes blood-tinged. On physical examination, his temper-
ature is 37.4° C. He has mild lower abdominal tenderness, but
no masses. A stool culture is positive for Shigella sonnei. Which
of the following microscopic findings would most likely be
seen in this child’s colon?
Epithelial disruption with overlying neutrophilic
A 
exudate
Extensive scarring of lamina propria with stricture
B 
formation
Intranuclear inclusions within small intestinal
C  4 A 60-year-old man has had persistent bloody diarrhea,
enterocytes abdominal cramps, and fever for the past week. On physical
D Multiple granulomas throughout the colon wall examination, his temperature is 38.1° C. He has mild diffuse
Slight increase in lymphocytes and plasma cells in
E  abdominal pain. A stool sample is positive for occult blood.
lamina propria Colonoscopy shows marked mucosal erythema with focal ul-
ceration from the rectum to the ascending colon. The ulcers do
3 A 6-month-old infant has abrupt onset of vomiting fol- not penetrate the muscularis propria. A biopsy is performed,
lowed by profuse, watery diarrhea. On physical examination, and the microscopic appearance of the specimen is shown in
the infant has a temperature of 38.3° C. Development is nor- the figure. Which of the following infectious organisms is most
mal for age, and the only abnormal finding is poor skin turgor. likely to produce these findings?
Laboratory studies show serum Na+ of 153 mmol/L, K+ of 4.4 A
Bacillus cereus
mmol/L, Cl– of 113 mmol/L, CO2 of 28 mmol/L, and glucose B
Entamoeba histolytica
of 70 mg/dL. Examination of a stool specimen shows mucus,

94
C H A P T E R 8   Infectious Diseases 95

C
Giardia lamblia 9 A study of nosocomial infections involving urinary
D
Salmonella enterica catheters is performed. The study shows that the longer an
E
Shigella flexneri indwelling urinary catheter remains, the higher the rate of
F
Vibrio cholerae symptomatic urinary tract infections (UTIs). Most of these
infections are bacterial. Which of the following properties of
5 In an epidemiologic study of individuals who died in these bacteria increase the risk for nosocomial UTIs?
a worldwide pandemic after World War I, many individuals Biofilm formation
A
were shown to have contracted a virulent form of influenza Enzyme elaboration
B
pneumonia. At the beginning of the 21st century, a similar Exotoxin release
C
epidemic is still possible from such a strain of influenza Quorum sensing
D
for which no vaccine may be readily available. Molecular Superantigen stimulation
E
analysis of samples of tissues showed changes in the virus
responsible for these virulent forms of influenza. Which of 10 A 91-year-old woman is hospitalized with sepsis. On
the following changes most likely occurs in this virus to examination she has fever and hypotension. Laboratory
increase its virulence? studies show positive blood cultures. She has disseminated
A Ability to elaborate exotoxins intravascular coagulopathy and pulmonary diffuse alveolar
B Acquisition of antibiotic resistance genes damage with respiratory distress. Analysis of the microbiol-
Increased binding to intercellular adhesion
C  ogy laboratory findings shows that the organisms cultured
molecule-1 (ICAM-1) receptor are gram-negative bacilli. Which of the following substances
D Mutations in DNA encoding envelope proteins elaborated by these organisms is most likely to cause this
Recombination with RNA segments from animal
E  complex of clinical findings?
viruses Endotoxin
A
Exotoxin
B
6 An epidemiologic study is conducted with children Mycolic acid
C
ages 1 to 5 years who are infected with HIV-1. Most of these RNA polymerase
D
children have CD4+ lymphocyte counts above 500/mm3 and Superantigen
E
undetectable plasma HIV-1 RNA levels. On physical examina- Tumor necrosis factor
F
tion they have no abnormal findings. What is the most likely
mode of transmission by which these children get infected? 11 A 6-year-old girl has a blotchy, reddish-brown rash
Breast-feeding
A on her face, trunk, and proximal extremities that developed
Inhalation of droplet nuclei
B over the course of 3 days. On physical examination, she has
Fecal-oral contact
C 0.2-cm to 0.5-cm ulcerated lesions on the oral cavity mucosa
Transfer across placenta
D and generalized tender lymphadenopathy. A cough with
Sexual abuse
E minimal sputum production becomes progressively worse
over the next 3 days. Which of the following viruses is most
7 In an experiment, phagocytosis of bacteria by neutro- likely to produce these findings?
phils is studied. Bacteria are introduced into plasma containing Epstein-Barr
A
neutrophils. It is observed that phagocytosis of Streptococcus Mumps
B
pneumoniae, Haemophilus influenzae, and Neisseria meningitidis is Rubella
C
reduced, compared with Enterobacteriaceae. Which of the fol- Rubeola
D
lowing immune evasion mechanisms in these three bacterial Varicella zoster
E
species best explains this finding?
Antimicrobial peptide binding
A 12 An 8-year-old girl has developed a mild febrile illness
Carbohydrate capsule formation
B with a sore throat over the past 2 days. On physical examina-
Interferon homologue production
C tion, her temperature is 38.4° C, and she has a mild pharyngi-
MHC protein down-regulation
D tis. The girl’s symptoms subside in 1 week without therapy.
Surface antigen switching
E Over the next 2 months, she has increasing right-sided facial
drooping with inability to close the right eye. Which of the
8 A 26-year-old man is an injection drug user and has following infectious organisms is most likely to produce these
developed fever over the past day. On examination, he has findings?
a heart murmur. A blood culture is positive for Staphylococ- A
Cryptococcus neoformans
cus aureus and antibiotic sensitivity testing shows resistance Cytomegalovirus
B
to methicillin due to presence of the mecA gene in these organ- C
Listeria monocytogenes
isms. Through which of the following adaptations are these Poliovirus
D
bacteria most likely to acquire their methicillin resistance? E
Toxoplasma gondii
Biofilm formation
A
Exotoxin release
B
Pathogenicity island transfer
C
Superantigen stimulation
D
Surface adhesin expression
E
96 U N I T I   General Pathology

13 A 6-year-old girl who lives in the Yucatán peninsula has 15 A 6-year-old boy developed a rash over his chest that
developed a high fever over the past 3 days. On physical ex- began as 0.5-cm reddish macules. Within 2 days, the mac-
amination, she has a temperature of 39.6° C and marked ten- ules became vesicles. Three days later, the vesicles ruptured
derness in all muscles. Laboratory studies show WBC count of and crusted over. Over the next 2 weeks, crops of the lesions
2950/mm3 with 12% segmented neutrophils, 4% bands, 66% spread to the face and extremities. Which of the following
lymphocytes, and 18% monocytes. Over 1 week, she becomes clinical manifestations of this infection is most likely to appear
more lethargic, with a decreased level of consciousness, and decades later?
petechiae and purpura develop over the skin. Further labo- A Chronic arthritis
ratory studies show thrombocytopenia with markedly pro- B Congestive heart failure
longed prothrombin time and partial thromboplastin time. C Infertility
CT scan of the brain shows a hemorrhage in the right parietal D Paralysis
lobe. Which of the following is most likely the vector of the E Shingles
agent causing infection in this patient?   
A Louse
B Mosquito
C Pig
D Snail
E Tick
F Tsetse fly
  

16 A 31-year-old HIV-positive man has had increasing


respiratory difficulty for the past 2 days. On physical exami-
nation, crackles are auscultated over all lung fields. A chest
radiograph shows bilateral interstitial infiltrates. Laboratory
studies show 26,800 copies of HIV-1 RNA/mL. A transbron-
chial biopsy is performed; the microscopic appearance of the
specimen is shown in the figure. Which of the following is the
14 A 22-year-old woman has had recurrent vesicular le- most likely causative organism of his pulmonary disease?
sions on her labia majora and perineum for 5 years. On physi- Adenovirus
A
cal examination, she is afebrile. Clusters of clear, 0.2-cm to 0.5- Cytomegalovirus
B
cm vesicles are present on the labia, with some surrounding Epstein-Barr virus
C
erythema. The figure shows the representative microscopic Herpes zoster virus
D
appearance at low magnification of one of the lesions. Which Respiratory syncytial virus
E
of the following cellular changes is most likely to be seen un-
der higher magnification of this lesion? 17 A 14-year-old boy presents with fever, sore throat, and
Dysplastic epithelial cells containing human
A  cervical lymphadenopathy. He then develops hepatomegaly
papillomavirus sequences and splenomegaly lasting for 2 months. His peripheral blood
Mononuclear infiltrates containing protozoal
B  smear shows leukocytosis with “atypical” lymphocytes.
organisms Which of the following cell types is most likely to eliminate
Multinucleated (syncytial) cells containing
C  the virally infected cells?
pink-to-purple intranuclear inclusions Cytotoxic CD8 cells
A
Neutrophils containing ingested gram-negative
D  Epithelioid macrophages
B
diplococci Helper CD4+ cells
C
Perivascular lymphoplasmacytic infiltrate sur-
E  IgG-secreting plasma cells
D
rounding arterioles, with endothelial proliferation Polymorphonuclear neutrophils
E
C H A P T E R 8   Infectious Diseases 97

18 A 75-year-old woman has a postoperative wound infec- 21 A 20-year-old woman has had increasing delirium for 2
tion responding poorly to antibiotic therapy. Over the next days and is admitted to the hospital. On physical examination,
3 days she develops confusion, nausea, vomiting, diarrhea, she has acute pharyngitis with an overlying dirty-white, tough
chills, and myalgias. On examination, she is febrile; the wound mucosal membrane. Paresthesias with decreased vibratory
site is erythematous with necrotic, purulent exudate. Labora- sensation are present in the extremities. On auscultation, there
tory studies show neutrophilia with left shift. Gram stain of is an irregular cardiac rhythm. A chest radiograph shows car-
the wound exudate shows gram-positive cocci in clusters. diomegaly. A Gram stain of the pharyngeal membrane shows
Which of the following substances is most likely being elabo- numerous small, gram-positive rods within a fibrinopurulent
rated by the infectious organisms? exudate. Which of the following is the most likely mechanism
A Lactoferrin for development of cardiac disease in this patient?
B Lipopolysaccharide Exotoxin-induced cell injury
A
C Phage-encoded A-B toxin Granulomatous inflammation
B
D Pili proteins Lipopolysaccharide-mediated hypotension
C
E Superantigen Microabscess formation
D
   Vasculitis with thrombosis
E

22 A 33-year-old primigravida at 18 weeks’ gestation de-


velops nausea with vomiting for 3 days and then a severe
headache and neck stiffness. On physical examination, her
temperature is 38.2° C. She has no papilledema. A lumbar
puncture is performed, and a Gram stain of the CSF obtained
shows many short, gram-positive rods, and with a wet mount,
the organisms demonstrate tumbling motility. In culture, this
organism grows at 25° C. She most likely acquired this illness
through which of the following mechanisms?
Ingestion of contaminated food
A
Inhalation of droplet nuclei
B
Inoculation through a cut on the skin
C
Sharing infected needles
D
Using a friend’s toothbrush
E

23 When Pharaoh did not heed Moses and let the Hebrews
go, a series of plagues fell on Egypt. In the fifth plague, large
domesticated mammals, including cattle, horses, and sheep,
died. This was followed by a plague in which the Egyptians
19 A 46-year-old woman has had a high fever and swelling, developed cutaneous boils that probably appeared as 1-cm
warmth, and tenderness of the right leg for the past 3 days. On to 5-cm areas of erythema with central necrosis forming an
physical examination, she has a temperature of 39.4° C and the eschar. Some Egyptians also may have developed a mild, non-
facial appearance shown in the figure. She receives macrolide an- productive cough associated with fatigue, myalgia, and low-
tibiotic therapy with which she recovers. Infection with which of grade fever over 72 hours, followed by a rapid onset of severe
the following organisms has most likely produced these findings? dyspnea with diaphoresis and cyanosis. Vital signs might
A
Clostridium botulinum have included temperature of 39.5° C, pulse of 105/min, res-
B
Escherichia coli pirations of 25/min, and blood pressure of 85/45 mm Hg. On
C
Neisseria gonorrhoeae auscultation of the chest, crackles would be heard at the lung
D
Staphylococcus epidermidis bases. A chest radiograph would show a widened mediasti-
E
Streptococcus pyogenes num and small pleural effusions. “Legacy” laboratory find-
ings would include a CBC with WBC count of 13,130/mm3,
20 A 52-year-old man has a fever and cough productive of hemoglobin of 13.7 g/dL, hematocrit of 41.2%, MCV of 91 μm3,
thick, gelatinous sputum that worsens over 4 days. On physi- and platelet count of 244,000/mm3. Despite antibiotic therapy
cal examination, his temperature is 38.2° C. On auscultation with anachronistic ciprofloxacin and doxycycline, many of
of the chest, diffuse crackles are heard at the right lung base. those affected would die. Which of the following organisms is
Laboratory studies show WBC count, 13,240/mm3 with 71% most likely to produce these findings?
segmented neutrophils, 8% bands, 15% lymphocytes, and 6% A
Bacillus anthracis
monocytes. A sputum gram stain shows gram-negative bacilli Herpes simplex virus
B
with mucoid capsules. His condition improves after a course C
Mycobacterium leprae
of gentamicin therapy. Which of the following complications D
Staphylococcus aureus
of this infection is he most likely to develop? Variola major
E
Abscess formation
A F
Yersinia pestis
Adenocarcinoma
B
Bullous emphysema
C
Cavitary granulomas
D
Gas gangrene
E
98 U N I T I   General Pathology

24 A 42-year-old HIV-positive man has had a fever and 28 A 4-year-old child develops a runny nose and cough.
cough for the past month. On physical examination, his tem- After the cough persists for 2 weeks she exhibits paroxysms of
perature is 37.5° C. On auscultation of the chest, decreased coughing so severe she becomes cyanotic. On physical exami-
breath sounds are heard over the right posterior lung. A chest nation, her temperature is 37.4° C. Her mouth and pharynx re-
radiograph shows a large area of consolidation with a central veal no erythema or swelling. On auscultation of the chest, her
air-fluid level involving the right middle lobe. A transbron- lungs show crackles bilaterally. She has spasmodic coughing,
chial biopsy specimen contains gram-positive filamentous or- with a series of coughs on a single breath, bringing up mucus
ganisms that are weakly acid-fast. His course is complicated plugs, followed by labored inspiration. The pathogenesis of
further by empyema and acute onset of a headache. A head CT her disease most likely results from disabling of which of the
scan shows a 4-cm discrete lesion of the right hemisphere with following?
ring enhancement. Which of the following infectious agents is Ciliary movement
A
most likely causing his disease? Complement lysis
B
A
Aspergillus fumigatus Immunoglobulin secretion
C
B
Mucor circinelloides NK cell activation
D
C
Mycobacterium avium complex Phagolysosome formation
E
D
Nocardia asteroides
E
Staphylococcus aureus 29 A 66-year-old man incurs extensive thermal burns to his
skin and undergoes skin grafting procedures in the surgical
25 A 50-year-old man with a neurodegenerative disease has intensive care unit. Two weeks later, he has increasing respi-
had a fever and cough productive of yellow sputum for the ratory distress. Laboratory studies show hemoglobin, 13.1 g/
past 3 days. On physical examination, there is dullness to per- dL; hematocrit, 39.2%; platelet count, 222,200/mm3; and WBC
cussion at the left lung base. A chest radiograph shows areas of count, 4520/mm3 with 15% segmented neutrophils, 3% bands,
consolidation in the left lower lobe. Despite antibiotic therapy, 67% lymphocytes, and 15% monocytes. A chest radiograph
the course of the disease is complicated by abscess formation, shows extensive bilateral infiltrates with patchy areas of con-
and he dies. At autopsy, there is a bronchopleural fistula sur- solidation. Bronchoscopy is performed, and microscopic exam-
rounded by a pronounced fibroblastic reaction. Small, yellow, ination of a transbronchial biopsy specimen shows pulmonary
1-cm to 2-mm “sulfur granules” are grossly visible within the vasculitis and surrounding areas of necrosis with sparse in-
area of abscess formation. Which of the following organisms is flammatory exudate. Which of the following infectious agents
most likely to produce these findings? is most likely to produce these findings?
A
Actinomyces israelii Adenovirus
A
B
Blastomyces dermatitidis B
Histoplasma capsulatum
C
Chlamydophila pneumoniae C
Mycobacterium tuberculosis
D
Klebsiella pneumoniae D
Pseudomonas aeruginosa
E
Mycobacterium kansasii E
Pneumocystis jiroveci
F
Streptococcus pneumoniae
26 A 22-year-old man has had more than 6 episodes of ure-
thritis in the past 4 years since becoming sexually active. Each 30 In October 1347, a Genoese trading ship returning from
time, gram-negative cocci are identified in the neutrophilic ex- the Black Sea docked at Messina, Sicily. The ship’s crew had been
udate. Which of the following components in these organisms decimated by an illness marked by a short course of days from
undergo change that prevents development of lasting protec- onset of inguinal lymph node enlargement with overlying skin
tive immunity? ulceration to prostration and death. A small, ulcerated pustule
Chitin
A ringed by a rosy rash was seen on the lower extremities of some
Envelope
B of the crew. Within days, more than half of the population of
Lipopolysaccharide
C the port city had died. Which of the following insect vectors was
Peptidoglycan
D most likely responsible for the rapid spread of this disease?
Pili
E Fleas
A
Teichoic acid
F Mosquitoes
B
Reduviid bugs
C
27 A 25-year-old woman has had pelvic pain, fever, and Sand flies
D
vaginal discharge for 3 weeks. On physical examination, she Ticks
E
has lower abdominal adnexal tenderness and a painful, swol-
len left knee. Laboratory studies show WBC count of 11,875/ 31 A study of sexually transmitted infections identifies an
mm3 with 68% segmented neutrophils, 8% bands, 18% lym- organism most commonly found in tropical and subtropical
phocytes, and 6% monocytes. She receives ceftriaxone ther- regions. This organism is associated with painful ulcerating
apy, but is not adherent with this therapy. She undergoes a genital papules in HIV-infected persons. Microscopic exami-
work-up for infertility 5 years later. Which of the following nation of lesional exudate with silver stain shows coccobacilli.
infectious agents is most likely to produce these findings? Which of the following organisms is most likely to produce
A
Candida albicans these findings?
B
Gardnerella vaginalis A
Chlamydia trachomatis
Herpes simplex virus-2
C B
Haemophilus ducreyi
D
Neisseria gonorrhoeae C
Klebsiella granulomatis
E
Treponema pallidum D
Neisseria gonorrhoeae
F
Trichomonas vaginalis E
Treponema pallidum
C H A P T E R 8   Infectious Diseases 99

32 A 23-year-old man from Irian Jaya, Indonesia, has a le- 35 A 41-year-old man has had worsening fever, cough,
sion on his penis that has enlarged over the past 4 months. On and dyspnea for 2 weeks. On examination, he has rales and
physical examination there is a painless 2-cm papular lesion of diminished breath sounds on auscultation of his chest. A chest
the dorsum of his penis that evolves into a beefy red expansile radiograph shows scattered infiltrates in both lungs. A tuber-
ulceration that bleeds easily. No inguinal lymphadenopathy culin skin test shows 6 mm of induration. A sputum sample is
is present. A biopsy of the lesion is taken and examined mi- negative, but bronchoalveolar lavage is positive, for acid-fast
croscopically, showing pseudoepitheliomatous hyperplasia bacilli. His WBC count is 4600/mm3 with differential count
and mixed inflammatory infiltrate. Giemsa stain shows cocco- of 80% neutrophils, 10% lymphocytes, and 10% monocytes.
bacilli within vacuoles in macrophages. Social history reveals Which of the following is the most likely risk factor for his
multiple sexual partners. Which of the following is the most pulmonary disease?
likely diagnosis? A Alcohol abuse
A Balanoposthitis B Diabetes mellitus
B Chancroid C HIV infection
C Granuloma inguinale D Scurvy
D Lymphogranuloma venereum E Smoking
E Secondary syphilis   
  

33 A 31-year-old man has had cough with a low-grade 36 A 32-year-old man has maculopapular and nodular
fever and a 4-kg weight loss over the course of 3 months. On skin lesions, mainly involving his face, elbows, wrists, and
physical examination, his temperature is 37.5° C. Laboratory knees. The nodular lesions have slowly enlarged over the
studies show anemia of chronic disease. A bone marrow biop- past 10 years and are now beginning to cause deformity. The
sy is performed, and the microscopic appearance is shown lesions are not painful, but he has hypoesthesia to anesthesia
in the figure. An acid-fast stain of this tissue is positive. The in these areas. The figure shows a microscopic acid-fast stain
causative infectious agent is most likely being destroyed by of a biopsy specimen of a nodular skin lesion. Which of the
which of the following mechanisms? following is the most likely diagnosis?
A Complement-mediated lysis Anthrax
A
B Elaboration of nitric oxide by macrophages Chagas disease
B
Generation of NADPH-dependent oxygen free
C  Hansen disease
C
radicals Leishmaniasis
D
D Phagocytosis by eosinophils Lyme disease
E
E Superoxide formation within phagolysosomes Onchocerciasis
F

34 A 5-year-old child is exposed to Mycobacterium tubercu- 37 A 20-year-old man who has multiple sexual partners
losis. A month later the child’s tuberculin skin test is positive. and does not use barrier precautions has had a nontender ul-
The child then develops fever, inspiratory stridor, and non- cer on his penis for the past week. On physical examination,
productive cough. Which of the following findings is most the 0.6-cm lesion has a firm, erythematous base and sharply
likely to be present on the chest radiograph of this child? demarcated borders. The lesion is scraped, and microscopic
Hilar lymphadenopathy
A darkfield examination is positive for motile spirochetes.
Miliary pulmonary nodules
B Which of the following inflammatory processes is most likely
Pneumonic consolidation
C to accompany this infection?
Upper lobe cavitation
D Acute inflammation with abscess formation
A
Vertebral lytic lesions
E Granulomatous inflammation with caseation
B
Gummatous inflammation with necrosis
C
Perivascular inflammation with plasma cells
D
100 U N I T I   General Pathology

38 A longitudinal study of men and women who have de- Abscess formation
A
veloped aortic root dilation and aortic insufficiency in adult- Cytopathic effects with apoptosis
B
hood is performed. They have a history of unprotected sexual Fibrous scarring
C
intercourse with multiple partners. Which of the following Gangrenous necrosis
D
laboratory tests is most likely to yield a positive result in these Granulomatous inflammation
E
persons? Lymphocytic infiltrates
F
Blood culture
A
Darkfield microscopy of lymph node
B 43 A 43-year-old man cuts the skin over his shin while
Fluorescent treponemal antibody-absorption (FTA)
C repairing a fence on his farm. The wound heals without any
Rapid plasma reagin (RPR)
D complications. Four days later, he develops muscle spasms of
Venereal disease research laboratory (VDRL)
E the face and extremities. These spasms worsen to the point of
severe contractions. Which of the following actions of the mi-
39 An infant born at term to a 33-year-old woman is se- crobial toxins is most likely responsible for the clinical features
verely hydropic. On physical examination, there is a diffuse in this case?
rash with sloughing skin on the palms and soles. Within 2 Cleavage of synaptobrevin in synaptic vesicles of
A 
days, the infant dies of respiratory distress. At autopsy, there neurons
is marked hepatosplenomegaly. Microscopic examination of Degradation of muscle cell membranes by
B 
the femur and vertebrae shows periosteitis and osteochon- phospholipase C
dritis. The lungs have nodular masses with central necrosis Inhibition of acetylcholine release at neuromuscular
C 
surrounded by mononuclear leukocytes, palisading macro- junctions
phages, and fibroblasts. A serologic test result for which of the D Release of cytokine by T lymphocytes
following agents is most likely to be positive in the infant’s Stimulation of adenylate cyclase production in
E 
mother? myofibers
Cytomegalovirus
A   
Herpes simplex type 2
B
HIV
C
Syphilis
D
E
Toxoplasma gondii

40 A 44-year-old woman notices an erythematous papule


on her left lower leg that develops into a ring-like rash and
then subsides over 3 weeks. Over the next 5 months, she has
migratory joint and muscle pain, substernal chest pain, and an
irregular heart rhythm. These problems subside, but 2 years
after the initial rash appeared, she develops a chronic arthritis
involving the hips, knees, and shoulders. Which of the follow-
ing is the most likely diagnosis?
Chagas disease
A
Dengue fever
B
Leishmaniasis
C
Leprosy
D
Lyme disease
E 44 A 27-year-old man is involved in a rollover accident in
Syphilis
F which he is ejected from the vehicle. He sustains a compound
fracture of the right humerus and undergoes open reduction
41 A radical group commissions scientists to develop with internal fixation of the humeral fracture. Three days later,
a Category A bioterrorism agent. They want an agent that he has marked swelling of the right arm and palpable crepitus.
will paralyze victims within hours and be disguised within A Gram stain of necrotic exudate from the wound site has the
­innocuous-appearing cans of split pea soup. Which of the fol- appearance shown in the figure. Through which of the follow-
lowing organisms best meets the requirements stated? ing mechanisms is this organism most likely causing extensive
A
Chlamydia psittaci tissue necrosis?
B
Clostridium botulinum Elaboration of lipopolysaccharide
A
Ebola virus
C Inhibition of phagocytic cell function
B
Hantavirus
D Resistance to multiple antibiotics
C
E
Yersinia pestis Superinfection with Candida albicans
D
Toxin-mediated lecithin degradation
E
42 A 25-year-old soldier incurs multiple skin wounds that
get infected and produce extensive tissue damage within a
day. Culture of necrotic tissue from deep inside one of the
wounds reveals anaerobic spore-forming gram-positive rods.
Which of the following microscopic pathologic reactions are
the toxins produced by these organisms most likely to cause?
C H A P T E R 8   Infectious Diseases 101

45 A 24-year-old man who is sexually active with multiple


partners has had pain during urination for the past 4 days. On
physical examination, there are no lesions on the penis. He is
afebrile. Urinalysis shows no blood, ketones, protein, or glu-
cose. Microscopic examination of the urine shows few WBCs
and no casts or crystals. What infectious agent is most likely to
produce these findings?
A
Candida albicans
B
Chlamydia trachomatis
Herpes simplex virus
C
D
Mycobacterium tuberculosis
E
Treponema pallidum

46 A 50-year-old woman residing in Port-au-Prince has


observed a small vesicle on her right labium majus for the
past 4 days. She is sexually active. On physical examination,
the 0.5-cm vesicle is filled with purulent exudate. Tender in-
guinal lymph nodes are palpable. She was diagnosed with 48 A 10-year-old girl with leukemia undergoes hematopoi-
non-Hodgkin lymphoma 10 years ago. A biopsy of one of the etic stem cell transplantation. She has poor engraftment, and
lymph nodes is performed and microscopically shows mul- 1 month later she develops fever and dyspnea. On physical
tiple abscesses in which central necrosis is surrounded by pali- examination, her temperature is 39° C. On auscultation of the
sading histiocytes. These clinical and pathologic findings are chest, wheezes and crackles are heard in both lungs. A chest
most likely caused by which of the following conditions? CT scan shows nodular lesions accompanied by cavitation,
A Candida albicans vaginitis hemorrhage, and infarction. Laboratory studies show hemo-
B Chlamydia trachomatis cervicitis globin, 8.8 g/dL; hematocrit, 26.5%; platelet count, 91,540/
C Gardnerella vaginalis vaginosis mm3; and WBC count, 1910/mm3 with 10% segmented neu-
D Herpes simplex virus infection of the perineum trophils, 2% band neutrophils, 74% lymphocytes, and 14%
Treponema pallidum infection of the external
E  monocytes. A bronchoalveolar lavage is performed; the fluid
genitalia was stained with Gomori methenamine silver stain and ana-
lyzed microscopically, as shown in the figure. Which of the
47 A 15-year-old boy has developed a small eschar on his following infectious agents is most likely to produce these
left forearm around the site of a tick bite he received 6 days findings?
ago. A hemorrhagic rash involving the trunk, extremities, A Aspergillus fumigatus
palms, and soles then develops over the next 3 days. Over B Coccidioides immitis
the past day, small, 0.2-cm to 0.4-cm foci of skin necrosis have C Corynebacterium diphtheriae
­developed on his fingers and toes. His temperature is 39° C. D Histoplasma capsulatum
He is treated with doxycycline and improves over the next 2 E Mycobacterium tuberculosis
weeks. Which of the following organisms is most likely to pro- F Pneumocystis jiroveci
duce these findings?   
A Borrelia burgdorferi
B Leishmania braziliensis
C Mycobacterium leprae
D Rickettsia rickettsii
E Yersinia pestis
  
102 U N I T I   General Pathology

49 A 24-year-old woman has noted worsening pain on the 51 A 19-year-old woman goes on a camping trip to a wooded
right side of her face for the past 24 hours. On examination, area in New England (USA) with lots of insects, but has forgot-
there is marked tenderness and swelling inferior to the zygo- ten to bring insect repellant. A month later, she has increas-
matic arch and lateral to the nasolabial fold on the right. Labo- ing malaise, low-grade fever, headaches, and myalgias. On
ratory studies show WBC count, 9900/mm3; serum creatinine, physical examination, she has hepatosplenomegaly. Labora-
2 mg/dL; sodium, 151 mmol/L; potassium, 5.4 mmol/L; tory studies show hemoglobin, 10.4 g/dL; WBC count, 5820/
chloride, 119 mmol/L; bicarbonate, 8 mmol/L; and glucose, mm3; and platelet count, 205,000/mm3. Her peripheral blood
483 mg/dL. A head CT scan shows soft tissue swelling and smear is shown in the figure. Which of the following infectious
bony destruction around the right maxillary sinus. A biopsy agents is most likely to produce these findings?
is performed; the figure shows the findings on microscopic A
Babesia microti
examination. Which of the following organisms is the most B
Borrelia burgdorferi
likely causative agent for this patient’s infection? C
Giardia lamblia
A Aspergillus niger D
Rickettsia rickettsii
B Actinomyces israelii E
Wuchereria bancrofti
C Candida albicans
D Clostridium perfringens 52 A 45-year-old man experiences malaise and fatigue,
E Cryptococcus neoformans which slowly become more noticeable over a 2-month peri-
F Mucor circinelloides od. He returned from a vacation along the Costa del Sol near
   Barcelona 10 months ago. He now has occasional diarrhea
and a low-grade fever. His abdominal discomfort worsens
over the next month. On physical examination, his vital
signs include temperature of 38.3° C. He has pronounced
splenomegaly, an increased liver span, and generalized
lymphadenopathy. Laboratory studies show hemoglobin,
11.8 g/dL; hematocrit, 34.9%; platelet count, 89,000/mm3;
and WBC count, 3350/mm3 with 29% segmented neutro-
phils, 5% bands, 48% lymphocytes, and 18% monocytes. His
total serum protein is 7.6 g/dL, albumin is 3.2 g/dL, AST
is 67 U/L, ALT is 51 U/L, alkaline phosphatase is 190 U/L,
and total bilirubin is 1.3 mg/dL. A stool sample is negative
for occult blood. Which of the following is the most likely
diagnosis?
Borreliosis
A
Echinococcosis
B
Leishmaniasis
C
50 An 11-year-old boy from Liberia has had episodic fevers Lyme disease
D
for 2 weeks. He developed a severe headache a week ago and Schistosomiasis
E
has become progressively more somnolent. On funduscopic Typhus
F
examination, he has papilledema. The representative micro-
scopic appearance of a cerebral vein is shown. Which of the
following organs is most likely to serve as the reservoir for
proliferation of the infectious agent producing this disease?
A Brain
B Heart
C Liver
D Lymph nodes
E Spleen
  
C H A P T E R 8   Infectious Diseases 103

53 A 24-year-old soldier stationed in the Middle East has Dilated cardiomyopathy


C
noted the appearance of a 0.5-cm papule on his left forearm. Meningitis
D
It becomes a 1-cm nodule with a central depression, and then Mucocutaneous ulcers
E
ulcerates over the next month. On physical examination, the Paranasal bony destruction
F
2-cm ulcerated lesion has an indurated border, and there are
three smaller satellite lesions. There is no hepatosplenomega- 56 A 28-year-old woman from rural Guyana with a his-
ly, but he has left axillary lymphadenopathy. Laboratory stud- tory of rheumatoid arthritis develops painful swelling of her
ies show hemoglobin, 14.1 g/dL; hematocrit, 42.5%; platelet hands and feet. She is treated with corticosteroid therapy.
count, 233,200/mm3; and WBC count, 6270/mm3. Which of A month later, she develops profuse, watery diarrhea along
the following infectious organisms is most likely to produce with fever and cough. On examination, she has a tempera-
these findings? ture of 37.3° C. Laboratory studies show WBC count, 12,900/
A Borrelia recurrentis mm3; and the WBC differential count shows 57% segmented
B Brugia malayi neutrophils, 5% bands, 16% lymphocytes, 8% monocytes,
C Leishmania major and 14% eosinophils. Microscopic examination of a stool
D Listeria monocytogenes specimen shows ova and small rhabditoid larvae. Similar
E Mycobacterium leprae larvae are present in a sputum specimen. Which of the fol-
F Trypanosoma gambiense lowing infectious diseases is most likely to produce these
   findings?
Cysticercosis
A
Onchocerciasis
B
Schistosomiasis
C
Strongyloidiasis
D
Trichinosis
E

57 A 17-year-old boy has had generalized muscle pain with


fever for 1 week. Over the past 2 days, he has developed in-
creasing muscular weakness and diarrhea. On physical exami-
nation, his temperature is 38° C. All of his muscles are tender
to palpation, but he has a normal range of motion, and no
significant decrease in muscle strength. Laboratory findings
include hemoglobin, 14.6 g/dL; hematocrit, 44.3%; MCV, 90
μm3; platelet count, 275,000/mm3; and WBC count, 16,700/
mm3 with differential of 68% segmented neutrophils, 6%
bands, 10% lymphocytes, 4% monocytes, and 12% eosinophils.
What is the most likely diagnosis?
Hemorrhagic fever
A
54 A 22-year-old man with extensive travel history is Influenza
B
bitten by an insect and has developed a rubbery, red, 1-cm Poliomyelitis
C
chancre on his right forearm over the past week. Three Scrub typhus
D
months later, he develops splenomegaly and lymphade- Trichinosis
E
nopathy. Two months later, he exhibits progressive wast-
ing with cachexia and decreased mentation. His peripheral 58 A 29-year-old man has had hematuria for the past
blood smear has the appearance shown in the figure. Where month. On physical examination, he is afebrile. There is dif-
is his disease most likely to have been acquired? fuse lower abdominal tenderness, but no palpable masses. An
Central America
A abdominal radiograph shows a small bladder outlined by a
Polynesia
B rim of calcification. Cystoscopy is performed, and the entire
Southeast Asia
C bladder mucosa is erythematous and granular. Biopsy sam-
Southern Europe
D ples are taken. Which of the following histologic findings is
West Africa
E most likely to be seen in these samples?
Acid-fast bacilli of Mycobacterium avium complex
A
55 A 9-year-old child who is living in a mud hut in Paraguay Eggs of Schistosoma haematobium
B
has a sore persisting on her face for 4 days. Physical examina- Larvae of Trichinella spiralis
C
tion shows an indurated area of erythema and swelling just Migrating Ascaris lumbricoides
D
lateral to the left eye, accompanied by posterior cervical lymph- Taenia solium cysts
E
adenopathy. She has unilateral painless edema of the palpebrae
and periocular tissues. Two days later, she has malaise, fever,
anorexia, and edema of the face and lower extremities. On
physical examination 1 week later, there is hepatosplenomeg-
aly and generalized lymphadenopathy. Which of the following
pathologic findings is most likely to develop in this patient?
Cerebral abscesses
A
Chronic arthritis
B
104 U N I T I   General Pathology

59 In a study of individuals living in a subtropical region in 61 Persons living in southern Africa where black flies are
which an irrigation project has been completed, it is noted that common and who have developed blindness are studied to
rice farmers have experienced an increased rate of an infec- identify a potential infectious cause. These persons are found
tious illness since the project began. Investigators determine to have a chronic dermatitis that preceded their blindness.
that the infection is acquired through cercariae that penetrate Skin lesions are pruritic, scaling, and hypopigmented. Ocu-
the skin. The cercariae are released from snails living in the lar lesions include punctate keratitis and focal corneal opaci-
irrigation canals. Infected individuals develop progressive as- ties, sclerosing keratitis, iridocyclitis with glaucoma, and
cites. Which of the following pathologic findings is most likely retinitis. Which of the following morphologic forms of the
to be present in these infected individuals as a consequence of infectious agent is most likely to be found in skin biopsies of
the infection? these persons?
A Dilated cardiomyopathy Acid-fast bacilli
A
B Scrotal elephantiasis Elementary bodies
B
C Hepatic fibrosis Intracellular diplococci
C
D Mucocutaneous ulcers Intranuclear inclusions
D
E Urinary bladder carcinoma Microfilariae
E
  
62 Within the same day, an emergency department is vis-
ited by 20 individuals, all of whom work in the same building.
Over the past day, they all experienced the sudden onset of
high fever, headache, backache, and malaise. On examination,
they are febrile. They do not have lymphadenopathy or hepa-
tosplenomegaly. Over the next 2 days, they develop a maculo-
papular rash on the face, forearms, and mucous membranes of
the oropharynx. Despite supportive care, a third of these pa-
tients die. Which of the following organisms is the most likely
causative agent?
A
Chlamydia psittaci
B
Francisella tularensis
Hantavirus
C
D
Mycobacterium kansasii
E
Rickettsia typhi
Variola major
F

60 A 40-year-old man has had progressive enlargement of


the right leg for the past 6 years, leading to the appearance
shown in the figure. On physical examination, he is afebrile.
He has inguinal lymphadenopathy and scrotal edema. Infec-
tion with which of the following organisms is most likely to be
present in this man?
A
Echinococcus granulosus
B
Leishmania tropica
C
Schistosoma mansoni
D
Trichinella spiralis
E
Wuchereria bancrofti
C H A P T E R 8   Infectious Diseases 105

ANSWERS
1  C  The lack of stomach acid in this woman predisposes to 4  B  Amebiasis is a common cause of dysentery in de-
enteric infections. Vibrio cholerae organisms are noninvasive. veloping nations. The figure shows two single-cell pro-
Instead, they produce severe diarrhea by elaboration of an en- tozoa invading tissue with inflammatory cells. Entamoeba
terotoxin, called cholera toxin, that acts on bowel mucosal cells histolytica organisms are resistant to gastric acid and can
to cause persistent activation of adenylate cyclase and high lev- invade the colonic submucosa via contact-dependent cy-
els of intracellular cyclic AMP that drives massive secretion of tolysis. The amebae not only produce local necrosis with
sodium, chloride, and water. The fluid loss is life-threatening ulceration and hemorrhage, but also gain access to the ve-
because of resultant dehydration. Amebiasis tends to produce nules of the portal system, which drains to the liver. Ame-
dysentery, with a bloody diarrhea, because the organisms can bic liver abscess is an uncommon complication of amebia-
invade the mucosa. Aspergillosis is seen in immunocompro- sis. The colonic lesions typically have disappeared by the
mised patients, particularly patients with neutropenia, and is a time the liver lesions appear. In some cases, there can be
rare cause of a diarrheal illness. Filariasis involves the lymphat- extensive mucosal involvement with characteristic flask-
ics and produces elephantiasis. Hydatid disease caused by Echi- shaped (similar to an Erlenmeyer flask) ulcerations similar
nococcus produces space-occupying cystic lesions in viscera. Ty- to those seen in other severe inflammatory bowel diseases.
phoid fever produces diarrhea, and the organisms can invade Bacillus cereus is a cause of food poisoning (most often as a
mucosa and disseminate to produce many systemic symptoms. contaminant in reheated fried rice) and has a short incuba-
PBD9 342, 363  BP9 321, 581–582  PBD8 338–339  tion time. Giardiasis tends to involve the small intestine
BP8 324, 327, 606–607 and produces variable inflammation, but no ulceration.
Salmonellosis more typically involves the small intestine
2  A  Shigellosis results in bloody dysentery because Shigella and in most cases produces self-limited enteritis, although
is highly virulent, resistant to gastric acid, and can invade and more severe disease with dissemination to other organs
destroy the colonic mucosa. There is typically a mononuclear can occur with Salmonella typhi infection. Shigellosis can
infiltrate extending to the lamina propria, with a neutrophilic produce bloody dysentery with irregular superficial co-
exudate overlying the ulcerated areas. Stricture formation lonic mucosal ulceration, but the organisms typically do
may follow intestinal tuberculosis. Intranuclear inclusions in not invade beyond the lamina propria. Cholera is charac-
enterocytes point to infection with DNA viruses, such as her- terized by massive, secretory diarrhea without intestinal
pesviruses like cytomegalovirus. Granulomatous inflamma- mucosal invasion or necrosis.
tion may be seen with granulomatous colitis (Crohn disease) PBD9 343, 390  BP9 324  PBD8 335, 806  BP8 608
and intestinal tuberculosis (rare). An increase in mononuclear
inflammatory cells may be seen with milder forms of entero-
colitis caused by viruses, Giardia, and Salmonella spp. 5  E  The influenza pandemic in 1918 resulted from an an-
PBD9 342–343, 345–346  BP9 316, 583  PBD8 338, 344  tigenic shift in the influenza A type. This antigenic shift
BP8 327, 332, 608 occurs when there is recombination with RNA sequences
of influenza viruses found in animals such as pigs (“swine
3  A  Rotavirus, an encapsulated RNA virus, is a major cause flu”) or birds (“avian flu”). A swine flu virus has been iden-
of diarrhea in infancy. The small intestinal villous destruction tified as a cause of the 1918 pandemic. The H5N1 strain
with atrophy leads to decreased absorption of sodium and wa- of influenza has been found in bird populations in mod-
ter. The development of IgA antibodies from secretory immu- ern times. Mutations in the viral hemagglutinin (H) and
nity in the bowel to rotavirus surface antigens provides older neuraminidase (N) envelope genes are responsible for epi-
children and adults a relative resistance to rotavirus infection. demics. These mutations allow evasion from host antibod-
Such antibodies are present in maternal milk and confer some ies. Influenza viruses do not bind to intercellular adhesion
degree of resistance to infants who breast-feed. Rotavirus in- molecule-1 (ICAM-1) receptors; rhinoviruses do. Influenza
fection occurs worldwide. By the age of 3 years, virtually ev- viruses have a genome with RNA, not DNA. Viruses do
ery individual has been infected by rotaviruses at least once. not make exotoxins and do not acquire antibiotic resistance
Most rotavirus infections are subclinical or cause mild gastro- like bacteria.
intestinal illnesses that do not require hospitalization. The first PBD9 343, 345–347  BP9 491  PBD8 339, 342  BP8 330–331
infection is the most likely to be symptomatic; subsequent in-
fections are often mild or asymptomatic. Many enteroviruses
also produce diarrhea by inhibiting the intestinal absorption 6  D  Congenital HIV infection is an example of verti-
of intraluminal sodium and water, but not as severe. Muta- cal transmission, and antiretroviral therapy for pregnant
tions in the CFTR gene lead to formation of thick mucus plugs, women has proven effective in reducing maternal-fetal HIV
giving rise to meconium ileus in infants. Decreased break- transmission. Breast-feeding is a potential mode of HIV
down of lactose occurs in disaccharidase deficiency and gives transmission, but less frequent than placental transmission.
rise to an osmotic diarrhea. Cholera is the result of secretion of Respiratory tract infections are spread by droplet nuclei, but
an exotoxin by the Vibrio cholerae organism, which potentiates HIV is not spread by this mode. The fecal-oral route of trans-
the epithelial cell production of adenylate cyclase and causes mission is typical for enteroviruses and hepatitis A virus, but
secretory diarrhea with sodium chloride and water loss. The not HIV. Though sexual abuse of children unfortunately oc-
Yop plasmid confers infectivity to Yersinia ­organisms. curs, it is an uncommon mode of transmission of infections.
PBD9 345–347  BP9 310, 585  PBD8 346, 349  BP8 322, 606 PBD9 244, 344  BP9 144, 318  PBD8 237, 340  BP8 156, 329
106 U N I T I   General Pathology

7  B  Bacteria that produce capsules are more resistant duces a ­positive-sense messenger RNA (mRNA) that directs
to phagocytosis, and help them to avoid an initial innate the host cell to produce ­viral components. Superantigens
immune response with neutrophils. This enables them to may produce findings similar to lipopolysaccharide-induced
establish infection (respiratory tract, and possible spread septic shock; the best known is toxic shock syndrome toxin,
to meninges) and become more virulent. Antimicrobial which is elaborated by some staphylococcal organisms. Tu-
peptides produced by epithelial cells can bind to bacterial mor necrosis factor (TNF) is elaborated by human inflamma-
organisms and form pores in the cell walls to kill them by tory cells, not by microorganisms, but by the release of TNF
osmotic lysis. Interferons are produced against viral or- by the action of endotoxins on macrophages that can mimic
ganisms, not bacteria. Some viruses, such as herpesvirus- gram-negative sepsis.
es, can impair expression of MHC class I molecules so that PBD9 350  BP9 321  PBD8 334, 344  BP8 333
viral antigens are not effectively displayed to CD4+ and
CD8+ cells. Surface antigen switching also helps organ-
isms such as trypanosomes to evade an adaptive immune 11  D  The rash and the Koplik spots on the buccal mucosa
response. are characteristic findings in measles (rubeola), a childhood
PBD9 346  BP9 322–323  PBD8 345–347  BP8 330 infection. It occurs only sporadically when immunizations
have been administered to a large part of the population. The
severity of the illness varies, and measles pneumonia may
8  C  Bacteria have multiple mechanisms for exchanging complicate the course of the disease, which in some cases
genetic material that affords selective growth advantages. can be life-threatening. Mononucleosis, which results from
Pathogenicity islands are bacterial chromosomal elements car- Epstein-Barr (EBV) virus infection, is more likely to occur in
rying virulence genes, such as those involving antibiotic resis- adolescence. Mumps produces parotitis and orchitis. Vari-
tance. Presence of the mecA gene imparts resistance to methi- cella-zoster virus infections in children manifest as chicken-
cillin (methicillin-resistant Staphylococcus aureus, or MRSA) pox. Rubella, also called German measles, is a much milder
and other β-lactam antibiotics. Additional bacterial genetic infection than rubeola.
transfer mechanisms include plasmids, transposons, and in- PBD9 355  BP9 310  PBD8 349–350  BP8 322
tegrons. Microbes on tissue surfaces form biofilms of sticky
polysaccharide goo to isolate themselves from immune attack.
Exotoxins impart virulence through tissue damage. 12  D  Poliomyelitis is an enterovirus spread through fecal-
Organisms such as S. aureus can express superantigens that oral contamination. The virus often infects the oropharynx
nonselectively stimulate many T cell clones, leading to un- first. It then spreads to bulbar nuclei and/or lower motor neu-
regulated cytokine release and toxic shock. Adhesins aid in rons in the anterior horn of the spinal cord to produce the mus-
microbial binding to host cells. cular paralysis typical of polio. In places where vaccination is
PBD9 349  BP9 320  PBD8 343–344  BP8 323 routinely available, this disease is rare. Cryptococcosis is a fun-
gal disease that most often involves the lungs and meninges.
Cytomegalovirus infection can be congenital; in immunocom-
9  A  Microbes form biofilms of sticky polysaccharide goo promised adults, it can involve many organs, principally the
that adheres particularly well to artificial surfaces such as gastrointestinal tract, brain, and lungs. Listeriosis is most often
catheters. The biofilm helps isolate the organisms from in- acquired via contaminated food or water; in most adults, it pro-
flammatory cells and limit penetration of antibiotics. Bacte- duces mild diarrheal illness, but in some adults and children,
rial enzymes, such as the hyaluronidases and streptokinases and in fetuses, it can produce meningitis or dissemination with
of streptococcal organisms, promote spread through tissues. microabscess (microgranuloma) formation. Toxoplasmosis can
Exotoxins of gram-positive organisms impart virulence be a congenital infection. In immunocompromised adults, it
through tissue damage. Bacteria have multiple mechanisms can produce inflammation in multiple tissues, but most often,
for exchanging genetic material that afford selective growth it causes chronic abscessing inflammation in brain.
advantages. When the number of bacterial organisms increas- PBD9 356  BP9 310, 828  PBD8 350–351   BP8 322, 329
es, they sense this (quorum sensing) and turn on virulence
genes. Organisms such as Staphylococcus aureus can express su-
perantigens that nonselectively stimulate many T cell clones, 13  B  Dengue fever, one form of hemorrhagic fever, is
leading to unregulated cytokine release and toxic shock. caused by an arbovirus of the Flavivirus group. This organ-
PBD9 349  BP9 320  PBD8 343–344  BP8 334 ism can be devastating because it produces bone marrow
suppression, and because any antibodies to the virus enhance
cellular viral uptake. It is transmitted by the mosquito vector
10  A  Gram-negative sepsis is classically mediated by en- Aedes aegypti. Louse-borne infections include rickettsial dis-
dotoxins, particularly the lipopolysaccharide component of eases. The pig can be involved in the life cycle of Taenia so-
the outer cell wall. With sepsis from gram-positive organ- lium and of Trichinella spiralis. T. spiralis can produce marked
isms there is release of exotoxins, such as tetanospasmin re- muscle pain, but typically not disseminated intravascular
leased by Clostridium tetani organisms. Mycolic acids found coagulopathy. Some snails can serve as an intermediate host
in the lipid wall of mycobacteria aids in the resistance of for Schistosoma organisms. Ticks can transmit ­typhus and
these organisms to degradation by acute inflammatory re- Lyme disease. The tsetse fly can transmit sleeping sickness,
sponses, leading to granulomatous inflammation. RNA which is endemic to Africa.
polymerase is found in negative-sense RNA viruses and pro- PBD9 354, 357  PBD8 349
C H A P T E R 8   Infectious Diseases 107

14  C  The figure shows a vesicle that has resulted from secreted by phagocytes activates T cells (the “atypical”
herpes simplex virus (HSV) infection. Most genital infec- lymphocytes), but the virally induced homologue does not.
tions are caused by HSV-2, whereas HSV-1 is responsible In general, viral infections are intracellular, and a cytotoxic
for most cases of herpetic gingivostomatitis. The viral cyto- CD8 T cell response is required to clear virus by eliminat-
pathic effect results in formation of intranuclear inclusions, ing infected cells. Epithelioid macrophages are most impor-
multinucleated cells, and cell lysis with vesicle formation in tant in granulomatous inflammatory responses that control
the epithelium. Cervical dysplasias do not produce vesicu- mycobacterial and fungal infections. Helper T cells may be
lar lesions and are the result of another sexually transmitted infected by EBV, but do not clear the virus. Immunoglobu-
disease—human papillomavirus infection. Protozoal infec- lin responses are most important to eliminate extracellular
tion with trichomoniasis, typically involving the vagina, may pathogens, such as bacteria. Neutrophils are most impor-
produce small blisters or papules, but these are often self- tant as an innate immune response directed against extra-
limited and not typically recurrent. Gram-negative diplococ- cellular organisms such as bacteria.
ci are characteristic of Neisseria gonorrhoeae infection, also a PBD9 360–362  BP9 426–427  PBD8 355–357  BP8 442– 443
sexually transmitted disease. Lymphoplasmacytic infiltrates
may be seen in chancres caused by Treponema pallidum, the
causative agent of syphilis. 18  E  Staphylococcal toxic shock syndrome (TSS) results
PBD9 357–358  BP9 310, 552, 681  PBD8 352  BP8 322, 708–709 from elaboration of superantigens that stimulate up to 20%
of T lymphocytes and generate a marked release of cyto-
kines and an extensive inflammatory response. Increasingly,
15  E  The skin lesions are typical of chickenpox, a common staphylococci have acquired the mecA gene that imparts resis-
childhood infection caused by varicella-zoster virus infec- tance to many penicillin (methicillin) and cephalosporin an-
tion. The infection can remain dormant for years in dorsal tibiotics has come to be associated with methicillin-resistant
root ganglia, only to reactivate when immune status is dimin- Staphylococcus aureus (MRSA). Most TSS cases occur in women,
ished. The virus, now designated herpes zoster (or varicella- because of the relationship to vaginitis. Lactoferrin is a sub-
zoster), spreads from the ganglion to the skin in the derma- stance secreted by human cells that binds iron needed by
tomal distribution of the corresponding sensory nerve, and bacteria, and is thus part of innate immunity. Lipopolysac-
it causes vesicular lesions with chronic, burning pain that is charides are elaborated by gram-negative organisms and pro-
difficult to stop. A chronic arthritis can be seen with Lyme duce endotoxic shock. Phage-encoded A-B toxin is elaborated
disease after Borrelia burgdorferi infection. Rheumatic heart by Corynebacterium diptheriae. Pili proteins are characteristic
disease can appear after group A β-hemolytic streptococcal for Neisseria gonorrhoeae to provide attachment to target cells
infection. Infertility is a complication of mumps orchitis. Pa- in the genital tract.
ralysis can complicate poliovirus infection. PBD9 362–363  BP9 321  PBD8 344, 357  BP8 334
PBD9 358  BP9 310  PBD8 353  BP8 322–323, 877

19  E  The rash and edema are manifestations of strepto-


6  B  This patient has a high HIV-1 RNA level consistent
1 coccal erysipelas, which is usually caused by group A or
with the diagnosis of AIDS. Although patients with AIDS are group C streptococci. Streptolysins elaborated by these or-
susceptible to many microbes, infections with cytomegalovi- ganisms aid in the spread of the infection through subcuta-
rus are particularly common. The biopsy specimen shows an neous tissues. Over 900 years ago the Order of St. Anthony
enlarged cell containing a large, distinct intranuclear inclu- was founded to treat persons with this illness, then known as
sion and ill-defined dark cytoplasmic inclusions, which are St. Anthony’s fire. Clostridium botulinum elaborates an exo-
typical of cytomegalovirus infection. Adenovirus is a viral toxin that, when ingested, results in paralysis. Escherichia coli
pathogen in both immunocompromised and immunocom- produces various infections, but skin infections are uncom-
petent adults that may produce a clinically significant pneu- mon. Neisseria gonorrhoeae is best known as a sexually trans-
monia, and intranuclear inclusions may be present, but the mitted disease, and a rash is possible, although usually there
cells are not large, and cytoplasmic inclusions are absent. Ep- is no pronounced swelling. Staphylococcus epidermidis is usu-
stein-Barr virus infection is seen frequently in patients with ally considered a contaminant in cultures.
HIV infection, but there are no distinct pulmonary lesions PBD9 364–365  PBD8 359–360
associated with it. Herpes zoster infections are most likely
to affect the peripheral nervous system, rarely can become
disseminated to affect the lungs in immunosuppressed pa- 20  A  Bacterial infections with predominantly neutrophilic
tients, and produce a different appearance than that shown. response are marked by suppurative inflammation, and a
Respiratory syncytial virus infections are seen in children, virulent organism such as Klebsiella can lead to tissue de-
but rarely in adults. struction with abscess formation. Carcinomas are not sequel-
PBD9 359–360  BP9 310, 500–501  PBD8 353–354  ae of bacterial infections. Infections of the lung do not result
BP8 159–161, 331 in emphysema, but may complicate emphysema. Granulo-
matous inflammation is characteristic of mycobacterial or
17  A  The features described fit with infectious mono- fungal infections. Gas-forming bacteria, such as anaerobic
nucleosis. EBV infection involves B cells that are activated Clostridium organisms, are unusual as a cause of respiratory
to elaborate a variety of cytokines that promote viral pro- infections.
liferation and reduced immune response. IL-10 normally PBD9 352  BP9 323–324  PBD8 347  BP8 334
108 U N I T I   General Pathology

21  A  This woman has diphtheria. The Corynebacterium 25  A  Actinomycetes that can produce chronic abscessing
diphtheriae organisms proliferate in the inflammatory mem- pneumonia, particularly in immunocompromised patients, in-
brane that covers the pharynx and tonsils. These gram-­ clude Actinomyces israelii and Nocardia asteroides. Persons with
positive organisms elaborate an exotoxin that circulates and neurodegenerative diseases are at risk for aspiration of oropha-
produces myocarditis and neuropathy. The organisms do ryngeal secretions that may contain these organisms. Sulfur
not disseminate to cause inflammation, abscesses or vascu- granules, formed from masses of the branching, filamentous
litis elsewhere in the body. Granulomatous inflammation is organisms, are more likely to be seen in Actinomyces. Blastomy-
more typical of mycobacterial and fungal infections. Endo- ces dermatitidis infections tend to produce a granulomatous in-
toxins such as lipopolysaccharide tend to be elaborated by flammatory process. Chlamydial infections produce an intersti-
gram-negative bacterial organisms. tial pattern similar to that of most viruses. Klebsiella infections,
PBD9 365  BP9 321  PBD8 360–361  BP8 333 similar to other bacterial infections, can result in abscess forma-
tion, although without distinct sulfur granules. Mycobacterium
kansasii infections are similar to Mycobacterium tuberculosis in-
22  A  The results of the Gram stain and culture are diag- fections in that granulomatous inflammation is prominent.
nostic for Listeria monocytogenes, an organism that is more PBD9 367  BP9 312  PBD8 362–363  BP8 324, 511
likely to produce disseminated disease in individuals who
are immunocompromised or pregnant, and it can produce
a congenital infection. Since the organism grows readily at 26  E  Pili are cell wall structures in gram-negative bacteria,
room temperature, it easily contaminates food and water. such as the Neisseria gonorrhoeae in this case, that facilitate at-
Unpasteurized dairy products are most often implicated. tachment to host cells. Pili proteins are altered by genetic re-
Listeriosis is not known to be acquired parenterally, or by combination, forming a “moving target” for host immunity,
the other listed routes. so reinfection can occur. Chitin is a prominent cell wall com-
PBD9 366  BP9 320  PBD8 361  BP8 256, 333, 874 ponent of fungi. Envelopes aid attachment of viruses to their
target host cells. Lipopolysaccharide in gram-negative bacte-
rial cell walls acts as an endotoxin. Peptidoglycan forms part
23  A  The features are those of cutaneous and respiratory of the bacterial cell wall, and a greater amount of it imparts
anthrax. Bacillus anthracis forms spores that resist environ- gram-positive staining. Teichoic acid is a prominent feature
mental degradation. The spores can be transmitted by aero- of gram-positive bacterial cell walls.
sols, making this organism an ideal terror weapon. Similar PBD9 368  BP9 317, 320  PBD8 343, 364  BP8 325, 332
to many gram-positive organisms, B. anthracis produces
disease via elaboration of exotoxins that have an active A
subunit and a binding B subunit. None of the other choices 27  D  This patient has pelvic inflammatory disease (PID),
involve outbreaks in domestic animals. Herpetic infections which may occur as a result of infection with Neisseria gonor-
form clear vesicles that can rupture to shallow ulcers. Myco- rhoeae or Chlamydia trachomatis. Both organisms cause sexu-
bacterium leprae can produce a faint rash early in its course, ally transmitted diseases, and chronic inflammation may
but involvement of peripheral nerves with loss of sensation lead to PID. Complications of PID include peritonitis, adhe-
predisposes to repeated trauma with deformity. Staphylococ- sions with bowel obstruction, and sepsis with endocarditis,
cus aureus can produce impetigo, typically on the face and meningitis, arthritis, and infertility. Of the remaining organ-
hands. Variola major is the agent for smallpox, which is char- isms listed, Candida can produce vaginitis with a curd-like
acterized by skin pustules, and pneumonia is the most likely discharge, but it does not typically produce PID. Gardnerella
cause of death. Yersinia pestis produces plague, which can produces a whitish discharge that has a “fishy” odor with
have bubonic and pneumonic forms, characterized by ulcer- bacterial vaginosis, which tends to remain localized. Herpes
ating lymph nodes surrounded by a rosy rash. simplex virus-2 (HSV-2), the most common agent of genital
PBD9 366–367  BP9 321  PBD8 361–362  BP8 321 herpes, can produce painful vesicles, usually on the exter-
nal genitalia, and is often recurrent. Treponema pallidum, the
causative agent of syphilis, produces a hard chancre on skin
24  D  Although nocardiosis typically begins in the lungs, it and mucosal surfaces. Trichomoniasis may also lead to infer-
often becomes disseminated, particularly to the central nervous tility, but this protozoan is not treated with cephalosporins,
system. These infections are most often seen in immunocom- and it generally does not produce disseminated disease.
promised patients. Aspergillosis also can affect immunocom- PBD9 368, 383  BP9 317, 695  PBD8 363–364  BP8 324–326, 727–728
promised individuals, particularly those with neutropenia, but
the fungal hyphae are easily distinguishable on hematoxylin 28  A  Bordetella pertussis is the causative agent for whoop-
and eosin stains. Mucor organisms have broad, nonseptate hy- ing cough. These infections occur infrequently when there is
phae and are seen most often in patients with diabetic ketoaci- widespread childhood vaccination against this organism. This
dosis or burn injuries. Mycobacterium avium complex infections coccobacillary organism is difficult to culture, and direct fluo-
are seen in individuals with AIDS, but these are short, acid-fast rescent antibody (DFA) testing is the fastest and most reliable
rods that produce poorly formed granulomas. Bacterial pneu- way to diagnose the infection. Nasopharyngeal aspirates and
monias also should be considered in immunocompromised swabs are the best specimens because the organisms attach
patients, and septicemia can complicate them, but Staphylococ- to ciliated respiratory epithelium. The toxin paralyzes cilia.
cus aureus organisms form clusters of gram-positive cocci. Complement lysis is most useful against circulating infectious
PBD9 367  BP9 312  PBD8 362–363  BP8 324, 511 agents. Immunoglobulins that circulate can bind organisms,
but secretion is an adaptive immune response taking days to
C H A P T E R 8   Infectious Diseases 109

weeks. NK cells attack host cells with MHC signaling turned stage, with a prominent lymphoplasmacytic infiltrate; the caus-
off by intracellular infectious agents such as viruses. Mycobac- ative agent is Treponema pallidum, and these spirochetes cannot
terial organisms inhibit phagolysosome formation to reduce be identified by Gram stain.
their intracellular destruction in macrophages. PBD9 370  BP9 677
PBD9 368–369  BP9 321  PBD8 364–365  BP8 324, 512–513

32  C  The causative organism is Klebsiella granulomatis (for-


9  D  Pseudomonas aeruginosa can infect the skin following
2 merly Calymmatobacterium granulomatis), the disease is most
burn injuries and spread to the lungs. These organisms se- common in tropical and subtropical regions, and the infec-
crete several virulence factors, as follows: exotoxin A, which tion may progress to scarring with urethral and lymphatic
inhibits protein synthesis; exoenzyme S, which interferes obstruction. Balanoposthitis is localized inflammation of
with host cell growth; phospholipase C, which degrades pul- the glans penis and prepuce, typically caused by Candida,
monary surfactant; and iron-containing compounds, which Gardnerella, or Staphylococcus spp. Chancroid caused by Hae-
are toxic to endothelial cells. These virulence factors result in mophilus ducreyi has features that overlap those of granuloma
extensive vasculitis with necrosis. Neutropenic patients are inguinale, but there is often lymph node involvement in the
particularly at risk. Histoplasma capsulatum yeasts can pro- former, and lack of the Donovan bodies in macrophages.
duce pulmonary disease resembling that of Mycobacterium Lymphogranuloma venereum is caused by Chlamydia tra-
tuberculosis, with granulomatous inflammation. Pneumocys- chomatis, which cannot be seen with Gram stain. Secondary
tis pneumonia is more likely to occur in patients with weak syphilis is marked by a maculopapular rash on the palms
cell-mediated immunity. Pneumococcal infections produce and soles, with a prominent lymphoplasmacytic infiltrate;
alveolar exudates without significant vascular involvement. the causative agent is Treponema pallidum, and these spiro-
PBD9 369  BP9 321  PBD8 364–365  BP8 324, 512–513 chetes cannot be identified with Gram stain.
PBD9 370–371  BP9 677

30  A  This incident marks the first appearance of the Black


Death in Europe, a disease that persisted during the 14th and 33  B  The figure shows a granuloma. Activated macro-
15th centuries. The plague spread through Italy and across the phages are the key cellular component within granulomas
European continent. By the following spring, it had reached as that form to control persistent organisms such as Mycobac-
far north as England, and within 5 years, it had killed 25 mil- terium tuberculosis. As part of delayed type hypersensitivity
lion people, one third of the European population. Rodents with a TH1 immune response, CD4+ cells secrete interferon-γ,
form the reservoir of infection (and cats weren’t as popular as which activates macrophages to kill organisms with reactive
they should have been). Flea bites and aerosols transmit the nitrogen intermediates. Complement-mediated lysis is not
infection very efficiently. The causative organism, Yersinia pes- involved in the destruction of intracellular bacteria such as
tis, secretes a plasminogen activator that promotes its spread. M. tuberculosis. Complement activation on the surface of M.
Plague was endemic in East Asia at the beginning of the 20th tuberculosis can opsonize the bacteria, however, for uptake by
century and was carried to San Francisco. Seeking to avoid a macrophages. Eosinophils are not a major component of most
panic that could be bad for business and tourism, California’s granulomas, and they cannot destroy mycobacteria. NADPH-
governor at the time did not enforce a quarantine. As a con- dependent reactive oxygen species are important in the lysis
sequence, plague is endemic in wild rodents in the western of bacteria by neutrophils. M. tuberculosis organisms reside in
United States, but it accounts for only occasional sporadic hu- phagosomes, which are not acidified into phagolysosomes.
man infections. Mosquitoes are best known as vectors of ma- PBD9 371–373  BP9 324  PBD8 347–348  BP8 334–336
laria; sand flies, of leishmaniasis; reduviid (triatomid) bugs, of
Chagas disease; and ticks, of Lyme disease.
PBD9 370  BP9 315  PBD8 365  BP8 321, 324 34  A  The child has primary tuberculosis. Most healthy per-
sons have subclinical disease, and a minority develop clini-
cal manifestations; of those, most have limited pulmonary
31  B  The causative organism of chancroid is Haemophilus du- involvement without dissemination. Primary tuberculosis is
creyi, which is difficult to grow in culture and often obscured by marked by the Ghon complex, which is a small subpleural
superinfecting organisms in the ulcerated lesions. Chancroid is granuloma at mid-lung along with prominent enlarged hilar
most common in Africa and Southeast Asia, is a co-factor in lymph nodes. These nodes may impinge upon central airways.
transmission of HIV, and its features overlap those of granu- When the cell-mediated immune response is poor, then there
loma inguinale, but there is often lymph node involvement in can be numerous small granulomas scattered throughout the
the former, and lack of the Donovan bodies in macrophages. lungs, or disseminated to other organs, as a miliary pattern
Lymphogranuloma venereum is caused by Chlamydia trachoma- (granulomas that are the size of millet seeds). Progressive pri-
tis, which cannot be seen with Gram stain. Klebsiella granuloma- mary tuberculosis can lead to more extensive lung involve-
tis (formerly Calymmatobacterium granulomatis) is also common ment with pneumonic infiltrates. Upper lobe cavitary disease
in tropical and subtropical regions. It causes granuloma ingui- is characteristic for secondary tuberculosis (reactivation or
nale, and the infection may progress to scarring with urethral reinfection) in persons who have previously mounted an im-
and lymphatic obstruction. Gonorrhea tends to produce a ure- mune response. One pattern of disseminated tuberculosis is
thritis in men and a cervicitis in women acutely, without genital Pott disease of the spine, sometimes as an isolated finding.
ulceration. Syphilis is marked by a chancre in the primary state PBD9 373–375  BP9 324  PBD8 347–348  BP8 334–336
and a maculopapular rash of palms and soles in the secondary
110 U N I T I   General Pathology

35  C  Anergy (less than the 10 mm of induration expected 38  C  Untreated infection with Treponema pallidum can lead
for a positive tuberculin test), sputum negativity despite ex- to tertiary syphilis years later. The most common manifes-
tensive pulmonary disease, and radiographic evidence of tations of tertiary syphilis include aortitis (typically in the
infiltrates resembling bacterial pneumonic consolidation all thoracic portion), neurosyphilis, and gummatous necrosis of
point to a poor cell-mediated immune response. HIV infection skin, soft tissue, bone, and joint (Charcot joint). This organ-
depletes the body of the CD4+ lymphocytes (explaining his ism cannot be cultured. The spirochetes are best identified
lymphopenia) essential for a TH1 immune response required by darkfield microscopy in exudates from primary chancres,
to contain mycobacterial infection. The debilitation accompa- but the organisms are hard to find in the tertiary stage of
nying alcohol abuse is more likely to lead to typical secondary the disease. Serologic testing is useful for screening and con-
tuberculosis, but more florid. Diabetes mellitus predisposes to firmation of syphilis. The nontreponemal tests (RPR, VDRL)
bacterial infections, but pulmonary disease is not characteristic are sensitive to a cardiolipin found in the more numerous
for diabetic complications. Scurvy may affect connective tis- spirochetes earlier in the disease; but these tests are not spe-
sues but not lung specifically. Smoking diminishes pulmonary cific because the presence of cardiolipin in human tissues is
innate immune defenses, mainly against bacterial pathogens. associated with other diseases, causing false-positive results.
PBD9 373–376  BP9 324  PBD8 347–348  BP8 334–336 The FTA test has specificity for T. pallidum.
PBD9 378–381  BP9 672–674  PBD8 374–375  BP8 701–703

36  C  Hansen disease (leprosy) is caused by the small, acid-


fast organism Mycobacterium leprae, which chronically infects 39  D  These are findings of congenital syphilis with nodules
peripheral nerves and skin. This organism cannot be cultured of gummatous necrosis. Because the spirochetes can cross the
in artificial media. Diagnosis is made by biopsy of a skin le- placenta in the third trimester, early stillbirths do not occur.
sion. There are two polar forms of leprosy. In the tuberculoid Infants who survive have features similar to adult secondary
form, a delayed type of hypersensitivity reaction, with a TH1 syphilis, with rash. With survival, late complications of the
immune response driven by interferon-γ and interleukin-2 periosteitis and perichondritis include bone and teeth defor-
(IL-2) cytokines, gives rise to granulomatous lesions that re- mities (e.g., saber shin). Herpes infections in the neonate usu-
semble tuberculosis; acid-fast bacilli are rare in such lesions. In ally are not initially obvious because most of these infections
contrast, in the lepromatous form, shown in the figure, T cell are acquired by passage through the birth canal. Most infants
immunity is markedly impaired, a TH2 immune response is born with HIV infection have no initial gross or microscopic
driven by IL-4 and IL-10, and granulomas are poorly formed. pathologic findings. Congenital toxoplasmosis and cytomeg-
Instead, there are large aggregates of lipid-filled macrophages alovirus produce severe cerebral disease.
that are stuffed with acid-fast bacilli. Leprosy is poorly trans- PBD9 378–381  BP9 312, 671–674  PBD8 375  BP8 592
missible through aerosols (not from direct contact); it probably
requires some genetic susceptibility, such as genetic variations
in IL-10 and Toll-like receptors; and, similar to most diseases 40  E  The acute stage of Lyme disease is marked by the ap-
throughout human history, is linked to poverty. Cutaneous pearance of erythema chronicum migrans of the skin. As the
anthrax, caused by Bacillus anthracis, produces a necrotic skin Borrelia burgdorferi organisms proliferate and disseminate,
lesion with eschar at the site of inoculation. The reduviid (tri- systemic manifestations of carditis, meningitis, and migratory
atomid) bug carries Trypanosoma cruzi, which causes Chagas arthralgias and myalgias appear. Arthritis involving the large
disease. Its bite may cause a localized area of skin erythema joints occurs 2 to 3 years after initial infection. Rheumatoid
and swelling. Mucocutaneous ulcers may be seen with Leish- arthritis can mimic Lyme disease but is not preceded by the
mania braziliensis infection, which is transmitted via sand flies. skin lesions described. Chagas disease may be associated with
The area of the tick bite that introduces Borrelia burgdorferi spi- acute and chronic myocarditis leading to heart failure; some
rochetes, the cause of Lyme disease, may manifest erythema patients have esophageal involvement, but arthritis and rash
chronicum migrans. Onchocerciasis occurs as a result of infec- are not features of the disease. Hemorrhagic fever, or dengue
tion with the filarial nematode Onchocerca volvulus and leads fever, caused by an arbovirus, can produce myositis and bone
to formation of a subcutaneous nodule. marrow suppression. Mucocutaneous ulcers may be seen with
PBD9 377–378  BP9 312  PBD8 372–373  BP8 324, 337 Leishmania braziliensis. Leprosy, or Hansen disease, is associat-
ed with skin anesthesia and granuloma formation with nodu-
lar deformities of the skin. In primary syphilis, a hard chancre
37  D  Infection with Treponema pallidum can lead to syphi- may be present at the site of inoculation (usually the external
litic chancres in the primary stage of syphilis. The chancres genitalia; less often in the oral cavity or anorectal region). In
are characterized by lymphoplasmacytic infiltrates and by secondary syphilis, a maculopapular rash may be present.
an obliterative endarteritis. Similar lesions also may appear PBD9 381–382  BP9 789–790  PBD8 377–378  BP8 324, 824
with secondary syphilitic mucocutaneous lesions. Acute in-
flammation with abscess formation is characteristic of bacte-
rial infections such as gonorrhea. Caseating granulomatous 41  B  The spores of Clostridium botulinum will survive the
inflammation is more characteristic of tuberculosis or fungal canning process in such non-acidic foods as peas when they
infections. Gummatous inflammation can be seen in adults are not heated sufficiently, so that organisms grow and elab-
with tertiary syphilis or in congenital syphilis. orate a neurotoxin. However, the plot fails when everyone
PBD9 378–381  BP9 672–674  PBD8 374–375  BP8 701–703 ­prefers to eat junk food and not vegetables. Perhaps the terror-
ists should promote trans fats, which, when combined with
C H A P T E R 8   Infectious Diseases 111

lack of exercise, will increase morbidity and mortality from sexually transmitted diseases. Candida infections typically
atherogenesis to a greater extent than any infectious agent. occur in immunocompromised patients or in patients receiv-
Chlamydia psittaci is a Category B agent that is airborne and ing long-term antibiotic therapy. Herpes simplex can pro-
causes pneumonia. Ebola virus produces a hemorrhagic fe- duce painful vesicles on the skin. Tuberculosis of the urinary
ver. Hantavirus spread through aerosolization of deer mouse tract is uncommon. A syphilitic chancre on the penis, not
droppings causes pneumonia and sepsis. Yersinia pestis is the present here, is an indicator of Treponema pallidum infection.
Black Death, which produces lymphadenitis, pneumonia, PBD9 383–384  BP9 676  PBD8 981  BP8 705–706
and sepsis; the vector is the rat flea.
PBD9 382–383  BP9 321  PBD 337–338  BP8 321, 334
46  B  Infection with Chlamydia trachomatis is a common
sexually transmitted disease. Most cases produce only ure-
42  D  Clostridia such as Clostridium perfringens represent one thritis and cervicitis; however, some strains of C. trachomatis
type of gram-positive rod like bacteria that produce powerful can produce lymphogranuloma venereum, a chronic ulcer-
exotoxins, causing extensive tissue necrosis so quickly that ative disease that is more endemic in Asia, Africa, and the Ca-
the acute inflammatory response lags. Abscesses are formed ribbean. In this disease, there is a mixed granulomatous and
of neutrophils responding to the inflammatory agent, often neutrophilic inflammatory reaction, as seen in this patient. In
a bacterial organism, but the liquefactive necrosis is mainly contrast, herpes simplex virus produces clear mucocutaneous
produced by enzymes released from the neutrophils. Fibrous vesicles with no exudates and is unlikely to involve lymph
scarring can certainly be part of the healing phase of inflam- nodes. Candidiasis can produce superficial inflammation
matory responses, but is less prominent with bacterial infec- with an exudate, but it is rarely invasive or disseminated in
tions than with agents producing more chronic inflammation. non-immunosuppressed individuals. Bacterial vaginosis due
Granulomatous inflammation typically develops in weeks to to Gardnerella produces a whitish discharge that has a “fishy”
months from persistent infection from agents such as myco- odor. Treponema pallidum, the causative agent of syphilis, pro-
bacteria. Lymphocytic infiltrates are most typical for chronic duces a hard chancre on skin and mucosal surfaces.
and viral infections, and there tends to be minimal necrosis. PBD9 383–384  BP9 311–313  PBD8 341, 380  BP8 322, 727–728
PBD9 382–383  BP9 323–325

47  D  This patient has Rocky Mountain spotted fever,


3  A  This man has tetanus. The contamination of a wound
4 which occurs sporadically in the United States, mostly in
with Clostridium tetani can result in the elaboration of a potent areas other than the Rocky Mountains. Rickettsial diseases
neurotoxin. This toxin is a protease that cleaves synaptobrevin, produce signs and symptoms from damage to vascular en-
a major transmembrane protein of the synaptic vesicles in in- dothelium and smooth muscle similar to a vasculitis. The
hibitory neurons. Clostridium perfringens elaborates a variety of most common vector is the wood tick Dermacentor andersoni.
toxins, one of which (alpha) is a phospholipase causing myo- Thrombosis of the affected blood vessels is responsible for
necrosis. Inhibition of acetylcholine release is not a feature of foci of skin necrosis. Headache and abdominal pain are often
infection. The toxin of Staphylococcus aureus is an enterotoxin prominent. Lyme disease, caused by Borrelia burgdorferi, can
that acts as a superantigen and stimulates T cell cytokine re- produce an erythema chronicum migrans of skin at the site
lease. Cholera is produced when the toxin elaborated by Vibrio of a tick bite. Mucocutaneous leishmaniasis mainly involves
cholerae stimulates epithelial cell adenylate cyclase. the nasal and oral regions. Hansen disease (leprosy), pro-
PBD9 382–383  BP9 312  PBD8 379  BP8 324, 334 duced by Mycobacterium leprae, results in skin anesthesia that
predisposes to recurrent injury. Plague, caused by Yersinia
pestis, can produce focal skin necrosis at the site of a flea bite,
44  E  The large, gram-positive rods seen in the figure are and ulceration over infected lymph nodes (bubos).
characteristic of Clostridium perfringens, which can contami- PBD9 384–385  BP9 311   PBD8 380–382  BP8 323
nate open wounds and produce gas gangrene. Clostridial
organisms can elaborate multiple toxins. C. perfringens al-
pha-toxin acts as a phospholipase C that degrades lecithin in 48  A  Aspergillus, Candida, and Mucor infections may be-
cellular membranes. Lipopolysaccharides are found in gram- come disseminated in the setting of neutropenia. Vascular
negative organisms. Inhibition of phagocytes is a feature of invasion can occur with fungal infections, particularly with
organisms such as Mycobacterium tuberculosis. Antibiotic re- Aspergillus and Mucor. The branching septate hyphae are
sistance is increasing in frequency, but is not the main mech- shown in the figure projecting from a fruiting body of Asper-
anism for clostridial virulence. Though devitalized tissues gillus. After these organisms gain a foothold (hyphae-hold)
can have polymicrobial infection, Candida is typically not the in tissues, they are very difficult to eradicate. Coccidioides im-
most virulent among superinfecting agents. mitis and Histoplasma capsulatum are fungi that can produce
PBD9 382–383  BP9 324   PBD8 379  BP8 333, 335 pulmonary disease resembling that of Mycobacterium tubercu-
losis, with granulomatous inflammation. They do not have a
propensity for vascular invasion. Corynebacterium diphtheriae
45  B  The most common cause of nongonococcal urethritis produces upper respiratory tract disease, mainly in children
in men is Chlamydia trachomatis. The condition is a nuisance who are not vaccinated against it. Pneumocystis pneumonia is
in men without significant sequelae; however, the behavior not typically accompanied by vascular changes.
that led to the infection can place the patient at risk for other PBD9 385–387  BP9 313, 502–504  PBD8 382, 384  BP8 324, 327
112 U N I T I   General Pathology

49  F  This patient is in diabetic ketoacidosis, which is a is a louse-borne rickettsial disease with skin rash that may
significant risk factor for mucormycosis. Note the broad, proceed to skin necrosis.
nonseptated hyphae more easily visible with H&E stain PBD9 392–393  BP9 310, 313   PBD8 388–390  BP8 324, 328
than special stains, unlike other fungi. In contrast, Asper-
gillus organisms have thinner hyphae with acute angle
branching and septations. Actinomyces organisms are long, 53  C  This patient has cutaneous leishmaniasis, and the
filamentous gram-positive bacilli. Candida infections are original papule was at the site of the sand fly vector bite. Leish-
typically superficial and have gram-positive budding cells maniasis is endemic in the Middle East, South Asia, Africa, and
with pseudohyphae. Large, gram-positive rods are char- Latin America. The organisms proliferate within macrophages
acteristic of Clostridium perfringens, which can contaminate in the mononuclear phagocyte system and can cause regional
open wounds and produce gas gangrene. lymphadenopathy. The cutaneous form does not have bone
PBD9 389  BP9 313, 829  PBD8 385–386  marrow involvement and splenic enlargement, so pancyto-
BP8 324, 337, 527–528 penia is not present. Borreliosis causes relapsing fever and is
transmitted via body lice. Brugia malayi is a nematode transmit-
50  C  This boy had cerebral malaria, the worst form of ted by mosquitoes that leads to filariasis involving l­ymphatics
malaria. After the infective mosquito bite, Plasmodium falci- to produce elephantiasis. Leishmania donovani is transmitted by
parum sporozoites invade liver cells and reproduce asexu- sand flies and leads to infection of macrophages, which pro-
ally. When the hepatocytes rupture, they release thousands duces hepatosplenomegaly, lymphadenopathy, and bone mar-
of merozoites that infect RBCs. The infected RBCs circulate row involvement with pancytopenia. ­Listeriosis is most often
and can bind to endothelium in the brain. Small cerebral ves- acquired via contaminated food or water. In most adults, it
sels become plugged with the RBCs, resulting in ischemia. produces mild diarrheal illness, but in some adults and chil-
The other listed options also could be secondarily involved dren, and in fetuses, it may produce meningitis or dissemina-
by vascular thromboses in the setting of malaria, but are not tion with microabscess (microgranuloma) formation. Mycobac-
extraerythrocytic sites for asexual reproduction. terium leprae causes Hansen disease (leprosy), with infection
PBD9 390–392  BP9 418–419  PBD8 386–388   BP8 326, 330, 433–434 of peripheral nerves and skin. In individuals with a strong
immune response, the tuberculoid form of this disease results
in granuloma formation; in individuals with a weak immune
51  A  This patient’s travel history suggests an insect-borne response, the lepromatous form occurs, characterized by large
disease. The figure shows the characteristic tetrad and ring numbers of macrophages filled with short, thin, acid-fast ba-
forms of Babesia microti, within erythrocytes. Babesiosis is, an cilli. African trypanosomiasis produces sleeping sickness.
uncommon malaria-like protozoan disease. The northeastern PBD9 392–393  BP9 310, 313  PBD8 388–390  BP8 324, 328
United States is an endemic area. The vector is the deer tick,
just as with Lyme disease from Borrelia burgdorferi, which
is a spirochete. Giardiasis typically produces self-limited, 54  E  The findings are consistent with African trypanoso-
watery diarrhea. Rickettsia rickettsii causes Rocky Mountain miasis, or sleeping sickness. The eradication of the tsetse fly
spotted fever, which occurs sporadically in the United States vector has been a priority for decades in many African coun-
in areas other than the Rocky Mountains and produces signs tries. Filarial worms endemic in parts of Central America,
and symptoms from damage to vascular endothelium and Southeast Asia, and Polynesia also can appear in blood, but
smooth muscle similar to a vasculitis. Wuchereria bancrofti is a are smaller in size and do not lead to chronic wasting. Fila-
form of filariasis that can cause elephantiasis, owing to lym- riasis is not endemic in Europe.
phatic obstruction in the presence of an inflammatory reac- PBD9 394  BP9 313   PBD8 390  BP8 322, 325, 328
tion to the adult filarial worms.
PBD9 392  PBD8 388  BP8 321, 328
55  C  This child is infected with Trypanosoma cruzi, result-
ing in Chagas disease, endemic to Central and South Ameri-
52  C  Visceral leishmaniasis (kala-azar) is caused by pro- ca. The vector is the reduviid (triatomid) bug. The organisms
tozoa in the Leishmania donovani complex. Of these, only can damage the heart by direct infection or by inducing an
L. donovani infantum is endemic to southern Europe and autoimmune response that affects the heart because of the
the Mediterranean area. It is transmitted to humans by the existence of cross-reactive antigen. Acute myocarditis rarely
sand fly (Phlebotomus). Pancytopenia implies bone marrow occurs, but most deaths from acute Chagas disease are due to
involvement, possibly enhanced by the enlarged spleen, heart failure. In 20% of infected individuals, cardiac failure
and the liver function abnormalities suggest liver involve- can occur 5 to 15 years after the initial infection. The affected
ment. Borreliosis causes relapsing fever and is transmitted heart is enlarged, and all four chambers are dilated. A cere-
via body lice. Echinococcal disease is caused by ingestion of bral abscess or acute meningitis is typically a complication
tapeworm eggs and can lead to cyst formation in visceral or- of a bacterial infection with septicemia. Chronic arthritis can
gans. Borrelia burgdorferi infection is transmitted via ticks and be seen in Lyme disease, which is transmitted by deer ticks.
can cause Lyme disease, characterized by erythema chroni- Mucocutaneous ulcers may be seen in Leishmania braziliensis
cum migrans, meningoencephalitis, and chronic arthritis. infection, which is transmitted via sand flies. Paranasal sinus
Schistosomiasis, which is transmitted via snails, can produce infection may be caused by Mucor circinelloides.
hepatic cirrhosis (Schistosoma mansoni or Schistosoma japoni- PBD9 394–395  BP9 402  PBD8 391  BP8 322, 414–415
cum) or bladder disease (Schistosoma haematobium). Typhus
C H A P T E R 8   Infectious Diseases 113

56  D  The rhabditoid larvae of Strongyloides stercoralis can cardiomyopathy may occur with Chagas disease, in which
become invasive filariform from autoinfection in immuno- the Trypanosoma cruzi organisms are transmitted through the
compromised hosts, so-called hyperinfection with involve- reduviid (triatomid) bug. Elephantiasis is a complication of
ment of multiple organs. Immunocompetent hosts typically filariasis, which is transmitted via mosquitoes. Mucocutane-
have only diarrhea. Parasites, particularly worms, crawling ous ulcers may be seen in Leishmania braziliensis infection,
through tissues incite a marked eosinophilia. Cysticercosis which is transmitted via sand flies. Squamous cell carcino-
from eating uncooked pork can result in the release of larvae mas may be seen in the bladder in chronic Schistosoma haema-
that penetrate the gut wall and disseminate hematogenously, tobium infection.
often settling in gray and white cerebral tissue, where they PBD9 397–398  BP9 325  PBD8 393–394  BP8 326, 329
develop into cysts. Onchocerciasis occurs as a result of infec-
tion with the filarial nematode Onchocerca volvulus and leads
to formation of a subcutaneous nodule. Schistosoma mansoni 60  E  The marked soft tissue enlargement and deformity
or Schistosoma japonicum infections have adult female worms is called elephantiasis, which results from lymphatic obstruc-
in the portal venous system that release eggs that can pro- tion in the presence of an inflammatory reaction to the adult
duce hepatic fibrosis; Schistosoma haematobium worms live filarial worms Wuchereria bancrofti. Echinococcus produces
in veins near the bladder and release eggs that result in he- hydatid disease of the liver, lungs, or bone. Leishmania trop-
maturia. Eating infected meat, typically uncooked pork, can ica can involve the skin, causing ulceration, and can enlarge
lead to trichinosis; Trichinella encysts in striated muscle to parenchymal organs. Schistosomiasis from Schistosoma man-
produce fever and myalgias. soni may affect the liver most severely. Trichinella larvae from
PBD9 395  PBD8 391–392 ingested, poorly cooked meat encyst in striated muscle.
PBD9 398–399  BP9 316  PBD8 395  BP8 322

57  E  Acute muscle pain with fever and eosinophilia sug-


gests a parasitic infestation of the skeletal muscles, most like- 61  E  Onchocerciasis is caused by Onchocerca volvulus
ly trichinosis; this results from ingesting poorly cooked meat with inflammation induced by the microfilaria. Both in-
infected with Trichinella spiralis larvae. Hemorrhagic fever sect abatement programs and campaigns to treat affected
can be a mild disease with myalgia, but can be severe with populations with ivermectin have helped to reduce the
extensive vascular endothelial damage. Influenza A and B prevalence of this disease. Though ivermectin kills micro-
infection may produce myositis in childhood, with more se- filariae, it does not kill the adult worms, and treatment
vere, focal, and later onset than the diffuse myalgias of typi- with doxycycline will eliminate the worm that symbiotic
cal flu. Poliomyelitis can lead to muscle weakness, but via Wolbachia bacteria need for reproduction. Acid-fast Myco-
neurogenic atrophy, through loss of motor neurons. Scrub bacterium leprae organisms may be seen in skin lesions of
typhus caused by Orientia tsutsugamushi can have myalgia lepromatous leprosy, but the eye is not involved. Elemen-
along with eschar and lymphadenopathy in the region of a tary bodies can be identified in conjunctival scrapings with
chigger bite. Chlamydia trachomatis infection, but additional eye com-
PBD9 396–397  BP9 314   PBD8 393  BP8 327 ponents are not involved, and there are no skin lesions.
Intracellular diplococci of Neisseria gonorrhoeae can cause
neonatal blindness. Intranuclear inclusions can be seen
58  B  Schistosoma haematobium is a parasitic infection most with herpes simplex keratitis, which can cause perforation
often seen in Africa, particularly the Nile Valley, in areas through corneal ulcerations.
where irrigation has expanded the range of the host snails. PBD9 399–400  PBD8 395–396
The adult worms live in veins adjacent to the bladder and
release eggs with a sharp spine to cut their way through the
wall of the urinary bladder, causing severe granulomatous 62  B  The Centers for Disease Control and Prevention
inflammation, fibrosis, and calcification. Mycobacterial in- has classified microbes into several categories based on the
fections of the urinary tract are uncommon and do not cause danger they pose as agents for bioterrorism on the basis
bladder fibrosis. Trichinella spiralis infects striated muscle. of their ease of production, dissemination, and production
Ascariasis involves the lower gastrointestinal tract, and the of serious illness. Variola major is the causative agent for
worms reside in the lumen. Cysticercosis from the pork tape- smallpox and has a mortality rate of 30%. Francisella tular-
worm Taenia solium can have a wide tissue distribution, but ensis is very infectious; only 10 to 50 organisms can cause
the brain is most often affected. disease. As a weapon, the bacteria can be made airborne
PBD9 397–398  BP9 312, 314  PBD8 393–394  BP8 335–336 for exposure by inhalation. Infected individuals experi-
ence life-threatening pneumonia. Chlamydia psittaci can
cause psittacosis, which also can produce pneumonitis,
59  C  These farmers are infected with either Schistosoma but the course is more variable. Hantavirus can produce
mansoni or Schistosoma japonicum. Female worms reside in a severe pneumonia, but the prodrome is longer, and the
the portal venous system and release eggs that cut their vector is the deer mouse. Mycobacterium kansasii produces
way into the liver and incite a granulomatous inflammatory findings similar to Mycobacterium tuberculosis. Rickettsia ty-
reaction. With time, the portal granulomas undergo fibro- phi is the causative agent for murine typhus with headache
sis, compressing the portal veins. This gives rise to severe and rash.
portal hypertension, splenomegaly, and ascites. A dilated PBD9 401  BP9 315  PBD8 337  BP8 321

You might also like

pFad - Phonifier reborn

Pfad - The Proxy pFad of © 2024 Garber Painting. All rights reserved.

Note: This service is not intended for secure transactions such as banking, social media, email, or purchasing. Use at your own risk. We assume no liability whatsoever for broken pages.


Alternative Proxies:

Alternative Proxy

pFad Proxy

pFad v3 Proxy

pFad v4 Proxy